You are on page 1of 53

Article III Bill of Rights

operate in compliance with legal requirements, within the limited area resulting from the joint operations
and the scale down program.

I. Section 1
The Sangguniang Panlungsod ratified the MOU in Resolution No. 97. In the same resolution, the
Sanggunian declared that the MOU was effective only for a period of six months starting July 25, 2002.
Thereafter, on January 30, 2003, the Sanggunian adopted Resolution No. 13 extending the validity of
Resolution No. 97 to April 30, 2003 and authorizing Mayor Atienza to issue special business permits to
the oil companies. Resolution No. 13, s. 2003 also called for a reassessment of the ordinance.
Petitioners filed this original action for mandamus on December 4, 2002 praying that Mayor Atienza be
compelled to enforce Ordinance No. 8027 and order the immediate removal of the terminals of the oil
companies.

MMDA v. Bel-Air Village Association, etc GR No. 135962, March 27, 2000 Maria Fatima
Orbecido
FACTS: Petitioner MMDA is a goverment agency tasked with the delivery of basic services in Metro
Manila and the herein respondent BAVA is a non-stock, non-profit corporation whose members are
homeowners in BEL Air Village which is a private subdivision in Makati City and also said
respondent is a registered owner of Neptune st. a road inside Bel Air Village. Moreover, on December
30,1995 petitioner sent a letter addressed through its chairman requesting respondent to open Neptune
st. to public vehicular traffic and likewise the perimeter wall separating the subdivision from the adjacent
Kalayan Avenue would be demolished. This notice was sent in pursuant to the mandate of the MMDA
Law or RA 7924 which requires the authority to use of roads as thoroughfares for the safe and
convenient movement of persons and vehicular traffic effective January 2, 1996. However, respondent
instituted against petitioner before RTC Branch 136 Makati cITY for injunction and prayed for a TRO
and preliminary injunction enjoining the opening of Neptune st. and prohibiting the demolition of the
perimeter wall.The RTC issued TRO the following day. On January 23,1996 after due hearing thr RTC
denied issuance of preliminary injunction.On the other hand, the Court of Appeals conducted the
inspection of Neptune st. and issued writ of preliminary injunction enjoining the implementation of the
MMDA' s proposed action.

Issue: Whether or not respondent has the mandatory legal duty to enforce Ordinance No. 8027 and
order the removal of the Pandacan Terminals.
Ruling: Under Rule 65, Section 3 of the Rules of Court, a petition for mandamus may be filed when
any tribunal, corporation, board, officer or person unlawfully neglects the performance of an act which
the law specifically enjoins as a duty resulting from an office, trust or station. Mandamus is an
extraordinary writ that is employed to compel the performance, when refused, of a ministerial duty that
is already imposed on the respondent and there is no other plain, speedy and adequate remedy in the
ordinary course of law. The petitioner should have a well-defined, clear and certain legal right to the
performance of the act and it must be the clear and imperative duty of respondent to do the act required
to be done.

ISSUE: Whether or not MMDA has the mandate to open the Neptune st. to public traffic pursuant to its
regulatory and police power?

When a mandamus proceeding concerns a public right and its object is to compel a public duty, the
people who are interested in the execution of the laws are regarded as the real parties in interest and
they need not show any specific interest. Besides, as residents of Manila, petitioners have a direct
interest in the enforcement of the city ordinances. Respondent never questioned the right of petitioners
to institute this proceeding.

HELD: NO, There is no syllable in RA 7924 that grants the MMDA police power, let alone legislative
power. It is noteworthy to say that the MMDA has limited powers such as the formulation, coordination,
regulation, implementation, preparation, management, monitoring,setting of policies, installation of
system and administration. Hence, the MMDA is, as termed in the charter itself, a "development
authority. It bears stressing that police power is lodged primarily in the National Legislature. It cannot be
exercised by any group or body of individuals not possessing legislative power. It is thus beyond doubt
that the MMDA is not a local goverment unit or a public corporation endowed with legislative power and
is not a political unit of goverment. Hence, it was illegal that they proposed the opening of Neptune st
for public traffic.

On the other hand, the Local Government Code imposes upon respondent the duty, as city mayor, to
enforce all laws and ordinances relative to the governance of the city. One of these is Ordinance No.
8027. As the chief executive of the city, he has the duty to enforce Ordinance No. 8027 as long as it has
not been repealed by the Sanggunian or annulled by the courts. He has no other choice. It is his
ministerial duty to do so.
Tupas v. CA 193 SCARA 597 Danilo Dinopol

Philippine Blooming Mills Employees Organization v. Philippine Blooming Mills Co. Inc., 51 SCRA 189
Jhemhar Ombra

Social Justice Society, et al v. Atienza, Jr., GR No. 156052, February 13, 2008 Analyn
Bongabong

Banco Espanol Filipino v. Palanca 37 P 921 Julius Camaingking

Facts: Plaintiff Banco Espanol Filipino accepted a mortgage from original defendant Engracio Palanca
Tanquenying y Limquingco on June 16, 1906 for lending P 218, 294.10. After the execution of the
instrument he returned to China which appears to be his native country; and he died upon January 29,
1910, without again returning to the Philippine Island. On March 31, 1908 Banco Espanol Filipino file a
petition to foreclose the said property. Since defendant is a non-resident, it was necessary to give
notice by publication. The Clerk of Court was also directed to send copy of the summons to the
defendants last known address, which is in Amoy, China. It is not shown whether the Clerk complied
with this requirement. Nevertheless, after publication in a newspaper of the City of Manila, the cause
proceeded and judgment by default was rendered. The decision was likewise published and afterwards
sale by public auction was held with the bank as the highest bidder. On August 7, 1908, this sale was
confirmed by the court. However, about seven years after the confirmation of this sale, a motion was
made by Vicente Palanca, as administrator of the estate of the original defendant, wherein the applicant
requested the court to set aside the order of default and the judgment, and to vacate all the

Facts: On November 20, 2001, the Sangguniang Panlungsod of Manila enacted Ordinance No. 8027,
which ordinance became effective on December 28, 2001, after its publication. Ordinance No. 8027
reclassified the area described therein from industrial to commercial and directed the owners and
operators of businesses disallowed under Section 1 to cease and desist from operating their
businesses within six months from the date of effectivity of the ordinance. Among the businesses
situated in the area are the so-called Pandacan Terminals of the oil companies Caltex, Petron and
Shell.
However, on June 26, 2002, the City of Manila and the Department of Energy (DOE) entered into a
memorandum of understanding (MOU) with the oil companies in which they agreed that the scaling
down of the Pandacan Terminals [was] the most viable and practicable option. Under the MOU, the City
of Manila and the DOE committed, among others, to enable the OIL COMPANIES to continuously

proceedings subsequent thereto. The basis of this application was that the order of default and the
judgment rendered thereon were void because the court had never acquired jurisdiction over the
defendant or over the subject of the action.

brought suiton several grounds, one of which is that said ordinance is arbitrary, oppressive and
unreasonable, being applied only to aliens who are thus, deprived of their rights to life, liberty and
property and therefore, violates the due process and equal protection clauses of the Constitution. Hiu
Chiong obtained judgment from the CFI declaring the ordinance null and void.

Issue: Whether or not due process of law was observed?


ISSUE:
Rulings: On Jurisdiction
The word jurisdiction is used in several different, though related, senses since it may have reference
(1) to the authority of the court to entertain a particular kind of action or to administer a particular kind of
relief, or it may refer to the power of the court over the parties, or (2) over the property which is the
subject to the litigation.
The sovereign authority which organizes a court determines the nature and extent of its powers in
general and thus fixes its competency or jurisdiction with reference to the actions which it may entertain
and the relief it may grant.
How Jurisdiction is Acquired
Jurisdiction over the person is acquired by the voluntary appearance of a party in court and his
submission to its authority, or it is acquired by the coercive power of legal process exerted over the
person.
Jurisdiction over the property which is the subject of the litigation may result either from a seizure of the
property under legal process, whereby it is brought into the actual custody of the law, or it may result
from the institution of legal proceedings wherein, under special provisions of law, the power of the court
over the property is recognized and made effective. In the latter case the property, though at all times
within the potential power of the court, may never be taken into actual custody at all. An illustration of
the jurisdiction acquired by actual seizure is found in attachment proceedings, where the property is
seized at the beginning of the action, or some subsequent stage of its progress, and held to abide the
final event of the litigation. An illustration of what we term potential jurisdiction over the res, is found in
the proceeding to register the title of land under our system for the registration of land. Here the court,
without taking actual physical control over the property assumes, at the instance of some person
claiming to be owner, to exercise a jurisdiction in rem over the property and to adjudicate the title in
favor of the petitioner against all the world.
In the terminology of American law the action to foreclose a mortgage is said to be a proceeding quasi
in rem, by which is expressed the idea that while it is not strictly speaking an action in rem yet it
partakes of that nature and is substantially such. The expression "action in rem" is, in its narrow
application, used only with reference to certain proceedings in courts of admiralty wherein the property
alone is treated as responsible for the claim or obligation upon which the proceedings are based. The
action quasi rem differs from the true action in rem in the circumstance that in the former an individual is
named as defendant, and the purpose of the proceeding is to subject his interest therein to the
obligation or lien burdening the property. All proceedings having for their sole object the sale or other
disposition of the property of the defendant, whether by attachment, foreclosure, or other form of
remedy, are in a general way thus designated. The judgment entered in these proceedings is
conclusive only between the parties.
It is true that in proceedings of this character, if the defendant for whom publication is made appears,
the action becomes as to him a personal action and is conducted as such. This, however, does not
affect the proposition that where the defendant fails to appear the action is quasi in rem; and it should
therefore be considered with reference to the principles governing actions in rem.

Whether or not the ordinance violates the due process clause of the Constitution?

RULING: YES. The Supreme Court affirmed the decision appealed from and held that the ordinance in
question violates the due process of law and equal protection rule of the Constitution. Requiring a
person before he can be employed to get a permit from the City Mayor of Manila who may withhold or
refuse it at will is tantamount to denying him the basic right of the people in the Philippines to engage in
a means of livelihood. While it is true that the Philippines as a State is not obliged to admit aliens within
its territory, once an alien is admitted, he cannot be deprived of life without due process of law. This
guarantee includes the means of livelihood. The shelter of protection under the due process and equal
protection clause is given to all persons, both aliens and citizens.

Galvez v. Court of Appeals, 237 SCRA 685 Alnashrip Arola

FACTS:
On November 12, 1993 petitioners Honorato Galvez, the incumbent mayor of San
Ildefonso, Bulacan, and one Godofredo Diego were charged in three separate informations with
homicide and two counts of frustrated homicide under Judge felipe N. Villajuan of RTC Malolos,
Bulacan and both accused posted their respective bail bonds for their temporary released. On the
process of their cases respondent prosecutor Dennis M. Villa-Ignacio filed an ex parte motion to
withdraw informations in said cases and subsequently filed four new infromations for murder, two
counts of frustrated murder and illegal possession of firearms and no bail having been recommended
under the sala of Judge Victoria Fornillos and ordered the warrant of arrest for the petitioners.
ISSUE:
Whether the Ex parte motion to withdraw the original information is null and void and
the filing of four new informations are valid under Article 3 section 1 of the 1987 Philippine Constitution?
HELD:
Contrary to petitioners submission, the absence of notice and hearing does not divest a
trial court of authority to pass on the merits of the motion. It has been held that the order of the court
granting the motion to dismiss despite absence of a notice of hearings, or proof of service thereof, is
merely an irregularity in the proceedings. It cannot deprive a competent court of jurisdiction over the
case. The court still retains its authority to pass on the merits of the motion. The remedy of the
aggrieved party in such cases is either to have the order set aside or the irregularity otherwise cured by
the court which dismissed the complaint, or to appeal from the dismissal and not certiorari.

State Prosecutors v. Muro, 236 SCRA 505 Sherlyn Saladaga

FACTS: In a letter-complaint, respondent Judge Manuel T. Muro of the Regional Trial Court (RTC) of
Manila, Branch 54, was charged by State Prosecutors Nilo C. Mariano, George C. Dee and Paterno V.
Tac-an with ignorance of the law, grave misconduct and violations of Rules 2.01, 3.01 and 3.02 of the
Code of Judicial Conduct. Respondent judge issued an Order dismissing eleven cases filed by the
undersigned complainant prosecutors, members of the DOJ Panel of Prosecutors against the accused
Mrs. Imelda Romualdez Marcos, for Violation of Central Bank Foreign Exchange Restrictions, as
consolidated in CB Circular No. 960, in relation to the penal provisions of Sec. 34 of R.A. 265, as
amended, such order was made on the basis of newspaper reports concerning the announcement by
the President of the Philippines of the lifting by the government of all foreign exchange restrictions and

Aspects of the Proceedings


Villegas v. Hui Chiong, - Lynette Latip
FACTS: An ordinance of the City of Manila signed by the petitioner Mayor Villegas prohibited the
employment of aliens in any occupation or business unless they first secured a permit from him(then
Mayor of Manila) and paid a fee of P50.00. Respondent HiuChiong, an alien, employed in Manila,

the arrival at such decision by the Monetary Board as per statement of Central Bank Governor Jose
Cuisia. Respondent Judge did not even have the prudence of requiring first the comment of the
prosecution on the effect of aforesaid Central Bank Circular/Monetary Board resolution on the pending
cases
before
dismissing
the
same.

eedings in the case.- The fault or error tainting the order of dismissal of the lower court consists in its
failure to observe procedural due process and to exercise its discretion properly and judiciously.- The
dismissal was based merely on the findings of the Acting Secretary
of Justice that no libel wascommitted. The trial judge did not make anindependent evaluation or
assessment of the merits of the case.
- The grant of the motion to dismiss was based upon considerations other than the
judge's own personal individual conviction that there was no case
againstthe accused. The trial judge must himself beconvinced that there was indeed no sufficienteviden
ce against the accused, and this conclusion can be arrived at only after an assessment of the evidence
in the possession of the prosecution.

ISSUE: Whether or not the action of the Respondent Judge, of not requiring first the comment of the
prosecution,
amount
to
denial
of
due
process
to
the
Government?
HELD: The court held that the very act of respondent judge in altogether dismissing sua sponte the
eleven criminal cases without even a motion to quash having been filed by the accused, and without at
least giving the prosecution the basic opportunity to be heard on the matter by way of a written
comment or on oral argument, is not only a blatant denial of elementary due process to the
Government but is palpably indicative of bad faith and partiality, regardless of how carefully he may
have evaluated changes in the factual situation and legal standing of the cases, as a result of the
newspaper report, the fact remains that he gave the prosecution no chance whatsoever to show or
prove that it had strong evidence of the guilt of the accused. He thereby effectively deprived the
prosecution of its right to due process. The Court finds respondent Judge Manuel T. Muro guilty of
gross ignorance of the law. He is hereby DISMISSED from the service, such dismissal to carry with it
cancellation of eligibility, forfeiture of leave credits and retirement benefits, and disqualification from
reemployment in the government service.

Petitioners Claim
- Martinez moved to dismiss the appeal on the ground that no appeal lies from the dismissal of a
criminal case, and certainly not by the private complainant, particularly where dismissal was at the
instance of the City Prosecutor upon orders of the Department of Justice.- If any remedy was available
to private complainant ,it was a petition for certiorari, not an appeal.
ISSUE: WON complainant is allowed to file an appeal?
HELD: YES. The right to appeal from a final judgment or order in a criminal case is granted to "any
party", except when the accused is placed thereby in double jeopardy.
Reasoning

Martinez v. CA, 237 SCRA 575 Richard Atamosa


Section 2, Rule 122 RCP" Who may appeal. Any party may appeal from a final judgment or order,
except if the accused would be placed thereby in double
jeopardy.- Court ruled that the word "party" must be understood to mean not only the government and
the accused, but also other persons who may be affected by the judgment rendered in the criminal
proceeding. Thus, the party injured by the crime has been held to have the right to appeal from a
resolution of the court which is derogatory to his right to demand civil liability arising from the offense. (
People v. Guido)

FACTS: - Manuel P. Martinez actually seeks the dismissal of the information for libel filed against him in
the Trial Court.- On complaint of then Vice-President Salvador H. Laurel, an Information was filed
before the RTC of Manila by Assistant Prosecutor Antonio J. Ballena, charging Manuel P. Martinez with
libel arising from the allegedly derogatory and scurrilous imputations
and insinuations against Laurel contained in Martinez' article entitled "The Sorrows of Laurel" published
on January 8, 1990 in his Manila Times column.- Martinez filed a "Motion for Reinvestigation" which
was denied by Judge Manuel E. Yuzon. The case wasset for arraignment and pre-trial conference on
July31, 1990, but this setting was cancelled in view of Judge Yuzon's retirement.- Martinez filed a
petition with the DOJ seeking review of the resolution of the City Prosecutor finding a prima facie case
of libel against him. Accordingly, 3rd Asst. City Prosecutor Lourdes C. Tabanag filed before
the trial court a motion to suspend proceedings pending resolution by the DOJ of Martinez' petitionfor
review, which was granted by Judge Pepito.- Complainant Laurel attempted once more to have the
case set for arraignment and trial. No action was taken on his said motion.- August 16, 1991: Acting
Justice Secretary Silvestre H. Bello III declared inter alia that while the language used in the article may
be unsavory and unpleasant to complainant, the same was not actionable as libel, as it embodied
merely an opinion protected as a privileged communication under Article 354 of the RPC. The appealed
resolution was set aside and the City Prosecutor was directed to cause the dismissal of the information
filed against Manuel F. Martinez. Consequently, a motion to dismiss was filed on August 26, 1991 and
set for hearing on December17, 1991. At the hearing, upon manifestation of complainant's counsel, as
private prosecutor, that he had received no copy of the motion to dismiss, the trial court directed
the case prosecutor to furnish said counsel the desired copy, giving the latter ten(10) days to respond
thereto.- Motion to dismiss was granted by Judge RobertoBarrios.- Laurel went to CA ascribing error to
the lower court.- CA issued a Resolution granting the appeal and remanding the case for arraignment
of the accused and trial on the merits. The Appellate Court ruled that private complainant had "sufficient
personality and a valid grievance against the order of dismissal before arraignment" and that
the remedy of
appealwas property available because the order of dismissal was a final order which terminated allproc

- Offended parties in criminal cases have sufficient interest and personality as 'person(s)
aggrieved' tofile the special civil action of prohibition andcertiorari under Sections 1 and 2 of Rule 65 in
line with the underlying spirit of the liberal construction of the Rules of Court- The procedural
recourse of appeal taken by private complainant Laurel is correct because the order of dismissal was a
final order. It finally disposed of the pending action so that nothing more could be done with it in the
lower court.- The remedy against such a judgment is an appeal, regardless of the questions sought to
be raised on appeal, whether of fact, or of law, whether involving jurisdiction or grave abuse
of discretion of the Trial Court. . . . (T)he party aggrieved . . . did not have the
option to substitute the special civil action of certiorari under Rule 65 for the remedy of appeal provided
for in Rule 41. Indeed, the existence and availability of the right of appeal are antithetical to the
availment of the special civil action of certiorari.- The rule therefore in this jurisdiction is that once
acomplaint or information is filed in Court anydisposition of the case as its dismissal or the conviction or
acquittal of the accused rests in the sound discretion of the Court. Although the fiscal retains the
direction and control of the prosecution of criminal cases even while the case is already in Cour the
cannot impose his opinion on the trial court. The Court is the best and sole judge on what to do with the
case before it.
Espeleta v. Avelino, 62 SCRA 396 Feliciano Beldad

Facts: Herein petitioner filed petition for Certiorari for trial court under Hon. Celso Avelino, Presiding
Judge, CFI, Cebu City for failure to follow the requirements of procedural due process where it orders
the testimony of a vital witness stricken off the records when it appears that the failure of said witness to
appear on the set for continuation of her testimony was due to justifiable circumstances. The petitioner
complains that he was denied procedural due process when respondent Judge acceded to the plea of
plaintiff-respondent that the testimony of a witness deemed by him as vital to his case be ignored and
disregarded in its entirety for failure to be present in court on the day set for her cross-examination,
despite due notice given in open court. However, it appears that said witness who is a new employee of
the Office of the Department of Local Government and Community Development at the time cannot
challenge the order of his new office and the urgency of her job and informed petitioners counsel of her
unavailability so that the latter could make the proper representation with the trial court, for possible
deferment.

damages docketed as Civil Case No. 1312-A of the same Regional Trial Court, against private
respondents . On December 6, 1988, the trial court in Civil Case No. 1311-A issued a restraining order
directing herein private respondents and the other defendants in said case to refrain from implementing
the alias writ of demolition. Private respondents Adora Cruz, et al. filed a petition for certiorari and
prohibition with the Court of Appeals docketed therein as CA-G.R. SP No. 16527, praying that the order
dated December 6, 1988 issued in Civil Case No. 1311-A be annulled and that the Honorable Patricio
Patajo, the Presiding Judge of the Regional Trial Court of Antipolo, Rizal, be prohibited from conducting
further proceedings in Civil Cases No. 1243 and No. 1311. This was the third time the case was brought
before the Court of Appeals. On August 30, 1990, the Court of Appeals rendered its decision in SP No.
16527 declaring the order dated December 6, 1988 null and void. On September 25, 1990, the Court of
Appeals denied herein petitioners motion for reconsideration.
Issue:
Whether the writ of execution issued in Civil Cases No. 630 and No. 631 may be enforced
against petitioners?

ISSUE: Whether or not there is abuse of discretion on the part of the respondent Judge of CFI, Cebu
City when he ordered the stricken off from the records the witness testimony which is vital to the case.

Held: No, Clearly, the second requirement afore-mentioned does not obtain in Civil Cases No. 630 and
631, for the trial court in said cases did not acquire jurisdiction over the persons of petitioners as they
were not impleaded therein and were consequently not summoned to appear and present their
defenses to resist the claims of private respondents. The fact that petitioners filed, as aforementioned,
an opposition to the issuance of a writ of demolition in Civil Cases No. 630 and No. 631 does not signify
that they had voluntarily submitted themselves to the jurisdiction of the trial court, precisely because the
very purpose of said opposition is that no writ of demolition should be issued or enforced against them
as they are not parties to said case. Furthermore, it cannot be said that they had intervened in said
cases by the filing of the afore-mentioned opposition for the reason that there can be no intervention in
a case already terminated by final judgment (Lorenzana vs. Cayetano, supra, p. 492). The right to due
process is one of the building blocks of the edifice of our democratic form of government, and courts
must ever be vigilant in safeguarding it, otherwise persons mights be dragooned, to jail without so much
as the filing of a complaint, or they may one morning wake up with all their hard-earned property
suddenly gone without so much they being aware of the cause therefor. WHEREFORE, the petition is
GRANTED. The assailed decision dated August 30, 1990 of respondent Court of Appeals is hereby
ANNULLED and SET ASIDE and the writs of execution and demolition in Civil Cases No. 630 and 631
are hereby declared null and void and unenforceable in so far as petitioners are concerned.

HELD: Yes, the court ruled that there was indeed abuse of discretion on the part of the Presiding Judge
of the CFI of Cebu City. Respondent Judge would justify the aforesaid order by characterizing the
request for postponement as tantamount to delaying the administration of justice. He was not exactly
mindful of a 1916 admonition of this Court, when it warned that a sound discretion in this regard should
be exercised by the trial judge, and the highly commendable desire for the dispatch of business should
not be permitted to turn the scales of justice rather than accede to a reasonable request for a
continuance. It is even more deplorable that there appeared to be a total lack of awareness of the due
process
implications
of
the
petition.
WHEREFORE, the petition for certiorari is granted and the order of April 4, 1974, issued by respondent
Judge is set aside, nullified and considered as totally devoid of any force or effect. The case is
remanded to the lower court for further proceedings in accordance with law and in consonance with this
decision, more specifically that the testimony of Miss Montao remains in the records subject to the test
of
cross-examination,
if
any,
by
private
respondent.
Doctrine .The mere filing of a motion to postpone hearing, even if the motion is not entirely groundless,
confers on the movant no right either to assume that the motion for postponement would be granted or
to be absent at, and shy away from, the hearing.

Gonzales v. CSC, 226 SCRA 66 Kinang

Rabino v. Cruz, 222 SCRA 493 Allan Tamayao

Facts:
This is a petition for review seeking the annulment of the decision of respondent Court of
Appeals in CA-G.R. SP No 16527. The first one was SP No. 05934, aforementioned. On December 24,
1986, the Court of Appeals promulgated its decision in SP No. 08124 dismissing the petition (Annex E
of Memorandum for Private Respondents). A motion for reconsideration was filed by petitioners therein
but on December 15, 1988, the Court of Appeals issued a resolution denying the motion for
reconsideration (Annex F, Memorandum for Private Respondents). Petitioners filed in Civil Cases No.
630 and No. 631 an opposition dated November 27, 1986 (CA-G.R. SP No. 16527), to the issuance of
a writ of demolition. On December 1, 1988, the other occupants of the subject parcel of land in Civil
Cases No. 630 and 631, petitioners herein, filed an action for damages, docketed as Civil Case No.
1243, of the Regional Trial Court of Antipolo, Rizal, against respondents (1) Marina, Antonio, Eduardo,
Concepcion, Teresita Cruz, Mario, Hector Jose, Mariano Gongora, Juan S.P. Herros and Pio E.
Martinez, in his capacity as Ex-Officio Sheriff of the Regional Trial Court of Antipolo, Rizal (See
Decision of Court of Appeals in CA-G.R. SP No. 16527). On December 1, 1988, petitioners filed
another action docketed as Civil Case No. 1311-A with the Regional Trial Court of Antipolo, Rizal, for
annulment of the writ of demolition with damages against private respondents. On the same date,
David Palmenco, et al., the defendants in Civil Cases No. 630 and 631 filed an action for injunction with

FACTS: At stake is the 36 year career service of the petitioner in government. For losing his job without
proper notice, petitioner seeks the protection of due process, the guaranty against acts of arbitrariness
of government. 1 His plea is not without merit.
It took petitioner Inocencio Gonzales thirty six (36) years to ascend to his position of
Administrative Officer III of the Agricultural Training Institute (ATI), an agency of the Department of
Agriculture. On his 25th year of service, he received a merit award recognizing his continuous,
dedicated, and faithful service in the government. On his 30th year, he repeated the feat. His record of
service is without any wart of malfeasance or misfeasance in office.
Early in 1990, certain problems beset his two (2)children in the United States. He had to fly to
the United States to attend to his children. He applied for and was granted leaves with pay from
February 2, to July 16, 1990. After six (6) months, however, the family problem had not gone away and
he decided to spend more time with his children.
On June 25, 1990, petitioner wrote to the Director of ATI requesting approval of a leave
without pay starting from the second week of July to December 31,1991. He cited as additional reason
his desire to take advantage of ". . . a physical check-up free of charge due to my childrens' medical

plan benefits". The letter was personally delivered by petitioner's wife. It carried his address in the
United States at 149 Declaration Way, San Jose, California, 95116.
For unknown reason, the Director of ATI did not act on the letter-request. It was neither
approved nor disapproved. Three (3) months later, ATI started acting adversely on petitioner's request.
On September 5, 1990, Atty. Ildefonso del Rosario, ATI's Assistant Director and OIC, wrote to petitioner
declaring him absent without official leave for more than thirty (30) days and warning him that should he
not report within five (5) days from receipt of the letter, he would be dropped from the rolls. The letter
was addressed at 30 Ventura St., BF Homes, Quezon City, petitioner's house. The letter, however, was
returned to sender (ATI) on September 27, 1990. 2
What ATI did was to publish a notice of similar import in the October 4, 11 and 18, 1990
issues of the Philippine Journal, a newspaper, of general circulation. On October 24, 1990, ATI dropped
petitioner from its rolls. He was not furnished a copy of the order. When he came back from the United
States and reported for work on November 19, 1990, he found out that Mercedes Puruganan had been
appointed to his position. By himself, he protested to the Civil Service Commission on December 14,
1990. His letter-protest was endorsed to the Merit Systems Protection Board (MPSB) for appropriate
action. On April 30, 1991, the Board ruled that petitioner was duly notified before he was dropped from
the roll. His appeal was dismissed. His motion for reconsideration was rejected.
Petitioner appealed to the Civil Service Commission. Again, petitioner lost. In its Resolution
No. 92-640, dated May 7, 1992, the Commission held:
(1) that the requirement of notice was "substantially" complied with by the ATI, and (2) that the failure of
ATI to act on his request for leave without pay was of "no moment".
Undaunted by his legal setbacks, petitioner filed this petition for certiorari.

Facts: The National Bureau of Investigation (NBI) filed with the Department of Justice a letter-complaint
charging petitioners with the crime of Rape and Homicide of Vizconde women in their home on June
30, 1991.
Forthwith, the Department of Justice formed a panel of prosecutors to conduct the preliminary
investigation.
Petitioners fault the DOJ Panel for its finding of probable cause. They assail the credibility of Jessica
Alfaro as inherently weak and uncorroborated due to the inconsistencies between her April 28, 1995
and May 22, 1995 sworn statements. They criticize the procedure followed by the DOJ Panel when it
did not examine witnesses to clarify the alleged inconsistencies.
Petitioners complain about the denial of their constitutional right to due process and violation of their
right to an impartial investigation. They also assail the prejudicial publicity that attended their
preliminary investigation.
Issues: 1. Whether or not the DOJ Panel gravely abused its discretion in holding that there is probable
cause to charge them with the crime of rape and homicide
2. Whether or not the DOJ Panel denied them their constitutional right to due process during their
preliminary investigation
Held: 1. NO. The Court ruled that the DOJ Panel did not gravely abuse its discretion when it found
probable cause against the petitioners. A probable cause needs only to rest on evidence showing that
more likely than not, a crime has been committed and was committed by the suspects. Probable cause
need not be based on clear and convincing evidence of guilt, neither on evidence establishing guilt
beyond reasonable doubt and definitely, not on evidence establishing absolute certainty of guilt.
2. NO. Petitioners were given all the opportunities to be heard.

ISSUE: Whether or not there is a violation of due process?


HELD: It is the ruling of the respondent Civil Service Commission that the sending of the said notice to
the residence of petitioner constitutes "substantial" compliance with the demands of due process. The
ruling would have some allure if the address of petitioner in the United States was not known to the
officials of ATI and if his Philippine address was his last known address. But as stressed above, they
knew of petitioner's exact address in the United States and there appears no impediment for them to
send the notice in this correct address. Petitioner, be it noted, was not moving from one residence to
another, to avoid service of legal notices. They are aware that petitioner was not momentarily staying in
his address in Quezon City where he could receive said notice. Under the circumstances, it is grave
abuse of discretion for the respondent Commission to hold that there was "substantial" compliance with
the notice requirement of due process. The disputed ruling cuts too deeply on petitioner's right to
continue his employment in the government and unduly dilutes the protection of due process. To be
sure, the cavalier attitude of respondent Commission is deplorable considering that on line is the thirty
six (36) long years of faithful and dedicated, service to the government of the petitioner. Nothing less
than strict compliance with the demands of due process should have been demanded by the
respondent Commission from the officials of ATI in light of the equities of the case. Nor can we give our
concurrence to the further ruling of the respondent Commission that the denial of due process to the
petitioner was cured by the publication of said notice in three (3) issues of the Philippine Journal. Notice
by publication might have been proper if the address of petitioner were unknown. Since the officials of
ATI knew the whereabouts of petitioner, they have no legal warrant to notify him thru the newspapers.

Facts: Three (3) separate Informations were filed against accused Claudio Teehankee, Jr. for the
shooting of Roland John Chapman, Jussi Olavi Leino and Maureen Hultman. Initially, he was charged
with: MURDER for the killing of ROLAND CHAPMAN, and two (2) FRUSTRATED MURDER for the
shooting and wounding of JUSSI LEINO and MAUREEN HULTMAN. When Hultman died on October
17, 1991, during the course of the trial, the Information for Frustrated Murder against accused was
amended to MURDER.
Issue: Whether or not the publicity given the case against the appellant was massive, overwhelming,
and prejudicial as to effectively deprive the accused of right to impartial trial.
Held : Pervasive publicity is not per se prejudicial to the right of an accused to fair trial. The mere fact

The petition for certiorari is granted and Resolution No. 92-640 dated May 7, 1992 of the
respondent Civil Service Commission is reversed and set aside. The Director of the Agricultural Training
Institute is ordered to reinstate petitioner to his position as Administrative Officer III or its equivalent
without loss of any right or privilege.

People v. Teehankee, GR No. 111206, Oct. 6, 1995 Engr. Sali

that the trial of appellant was given a day-to-day, gavel-to-gavel coverage does not by itself prove that
the publicity so permeated the mind of the trial judge and impaired his impartiality. For one, it is
impossible to seal the minds of members of the bench from pre-trial and other off-court publicity of
sensational criminal cases.

Webb v. Hon. Raul De Leon, 247 SCRA 652 Ria Jajurie

instruction that it re-open the case, receive all such evidence as may be relevant, and otherwise
proceed in accordance with the requirements set forth in the decision. Moreover, it was found no
substantial evidence to indicate that the exclusion of the 89 laborers here was due to their union
affiliation or activity. The whole transcript taken contains what transpired during the hearing and is more
of a record of contradictory and conflicting statements of opposing counsel, with sporadic conclusion
drawn to suit their own views. It is evident that these statements and expressions of views of counsel
have no evidentiary value.

At best, the accused can only conjure possibility of prejudice on the part of the trial judge due to the
barrage of publicity that characterized the investigation and trial of the case. The SC had previously
rejected this standard of possibility of prejudice and adopted the test of actual prejudice as we ruled
that to warrant a finding of prejudicial publicity, there must be allegation and proof that the judges have
been unduly influenced, not simply that they might be, by the barrage of publicity. In the case at bar, the

Essential requirements of due process in trials and investigations of an administrative character


a. There must be a hearing, where a party may present evidence in support of his case.
b. The tribunal must consider the evidence presented by a party.
c. While the tribunal has no duty to decide the case correctly, its decision must be supported by
evidence.
d. The evidence supporting the decision must be substantial. Substantial evidence is such relevant
evidence as a reasonable mind might accept as adequate to support a conclusion.
e. The evidence must have been presented at the hearing or at least contained in the record and
known to the parties affected.
f. The tribunal must rely on its own independent consideration of evidence, and not rely on the
recommendation of a subordinate.
g. The decision must state the facts and the law in such a way that the parties can know the issues
involved and the reasons for the decision.

records do not show that the trial judge developed actual bias against appellant as a consequence of
the extensive media coverage of the pre-trial and trial of his case. The totality of circumstances of the
case does not prove that the trial judge acquired a fixed opinion as a result of prejudicial publicity which
is incapable of change even by evidence presented during the trial. The accused has the burden to
prove this actual bias and he has not discharged the burden. There is no evidence showing that the trial
judge allowed the proceedings to turn into a carnival. Nor did he consent to or condone any
manifestation of unruly or improper behavior or conduct inside the courtroom during the trial of the case
at bar.

Ang Tibay v. CIR 69 PHIL 635 Maria Katrina Tanjusay

FACTS: The petitioner Rural Bank of Buhi, Inc. is a juridical entity existing under the laws of the
Philippines. Buhi is a rural bank that started its operations only on December 26, 1975.
In 1980, an examination of the books and affairs of Buhi was ordered conducted by the Rural
Banks and Savings and Loan Association (DRBSLA), Central Bank of the Philippines, which by law, has
charge of the supervision and examination of rural banks and savings and loan associations in the
Philippines. However, said petitioner refused to be examined and as a result thereof, financial
assistance was suspended.
On January 10, 1980, a general examination of the banks affairs and operations was
conducted and there were found by DRBSLA represented by herein respondent Consolacion V. Odra,
Director of DRBSLA, among others, massive irregularities in its operations consisting of loans to
unknown and fictitious borrowers, where the sum of P1,704,782.00 was past due and another sum of
P1,130,000.00 was also past due in favor of the Central Bank.
Respondent, Consolacion V. Odra, submitted a report recommending to the Monetary Board
of the Central Bank the placing of Buhi under receivership in accordance with Section 29 of Republic
Act No. 265, as amended, the designation of the Director, DRBSLA, as receiver thereof.

FACTS: The Solicitor-General Ozaeta in behalf of the respondent Court of Industrial Relations has filed
a motion for reconsideration, and the respondent National Labor Union, Inc., on the other hand, prays
for the vacation of the judgment rendered by the majority of the Court and the remanding of the case to
the Court of Industrial Relations for a new trial. The latter alleged the following and to wit: (a) Toribio
Teodoro's claim that on September 26, 1938, there was shortage of leather soles in ANG TIBAY making
it necessary for him to temporarily lay off the members of the National Labor Union Inc., is entirely false
and unsupported by the records of the Bureau of Customs and the Books of Accounts of native dealers
in leather; (b) That Toribio Teodoro's letter to the Philippine Army dated September 29, 1938, was a
scheme to systematically prevent the forfeiture of this bond despite the breach of his CONTRACT with
the Philippine Army; (c) That the National Workers' Brotherhood of ANG TIBAY is a company or
employer union dominated by Teodoro, the existence and functions of which are illegal; (d) That the
employer Teodoro was guilty of unfair labor practice for discriminating against the NLU, Inc., and
unjustly favoring the NWB (e) That in the exercise by the laborers of their rights to collective bargaining,
majority rule and elective representation are highly essential and indispensable. (Sections 2 and 5,
Commonwealth Act No. 213.)
ISSUE:

Rural Bank of Buhi v. CA, 162 SCRA 288 Cyril Joy Casil

ISSUE: UNDER SEC. 29, R.A. 265, AS AMENDED, MAY THE MONETARY BOARD (MB) OF THE
CENTRAL BANK (CB) PLACE A RURAL BANK UNDER RECEIVERSHIP WITHOUT PRIOR NOTICE
TO SAID RURAL BANK TO ENABLE IT TO BE HEARD ON THE GROUND RELIED UPON FOR
SUCH RECEIVERSHIP?

Whether or not notice and hearing are always required in administrative proceedings?

RULING: The court ruled in the affirmative and in favor of the respondents motion. The interest of
justice would be better served if the movant is given opportunity to present at the hearing the
documents referred to in his motion and such other evidence as may be relevant to the main issue
involved. The legislation which created the Court of Industrial Relations and under which it acts is new.
The failure to grasp the fundamental issue involved is not entirely attributable to the parties adversely
affected by the result. Accordingly, the motion for a new trial should be, and the same is hereby,
granted, and the entire record of this case shall be remanded to the Court of Industrial Relations, with

RULING: YES. There is no requirement whether express or implied that a hearing be first conducted
before a banking institution may be placed on receivership.
The appointment of a receiver may be made by the Monetary Board without notice and
hearing but its action is subject to judicial inquiry to insure the protection of the banking institution.
Stated otherwise, due process does not necessarily require a prior hearing; a hearing or an opportunity

to be heard may be subsequent to the closure. One can just imagine the dire consequences of a prior
hearing: bank runs would be the order of the day, resulting in panic and hysteria. In the process,
fortunes may be wiped out, and disillusionment will run the gamut of the entire banking community.
Closure and liquidation of a bank may be considered as an exercise of police power. Such
exercise may, however, be subject to judicial inquiry and could be set aside if found to be capricious,
discriminatory, whimsical, arbitrary, unjust or a denial of the due process and equal protection clauses
of the Constitution.
Courts may interfere with the Central Banks exercise of discretion in determining whether or
not a distressed bank shall be supported or liquidated.

Minimum standards to be satisfied in the imposition of disciplinary sanctions in academic institutions,


such as Petitioner University herein, thus:
1.
2.
3.
4.
5.

ADMU v. Capulong 222 SCRA 644 Roel Marcial

FACTS: Leonardo H. Villa, a first year law student of Petitioner University, died of serious physical
injuries at Chinese General Hospital after the initiation rites of Aquila Legis. Bienvenido Marquez was
also hospitalized at the Capitol Medical Center for acute renal failure occasioned by the serious
physical injuries inflicted upon him on the same occasion. Petitioner Dean Cynthia del Castillo created a
Joint Administration-Faculty-Student Investigating Committee which was tasked to investigate and
submit a report within 72 hours on the circumstances surrounding the death of Lennie Villa. Said notice
also required respondent students to submit their written statements within twenty-four (24) hours from
receipt. Although respondent students received a copy of the written notice, they failed to file a reply. In
the meantime, they were placed on preventive suspension. The Joint Administration-Faculty-Student
Investigating Committee, after receiving the written statements and hearing the testimonies of several
witness, found a prima facie case against respondent students for violation of Rule 3 of the Law
School Catalogue entitled "Discipline." Respondent students were then required to file their written
answers to the formal charge. Petitioner Dean created a Disciplinary Board to hear the charges against
respondent students. The Board found respondent students guilty of violating Rule No. 3 of the
Ateneo Law School Rules on Discipline which prohibits participation in hazing activities. However, in
view of the lack of unanimity among the members of the Board on the penalty of dismissal,
the Board left the imposition of the penalty to the University Administration. Accordingly, Fr. Bernas
imposed the penalty of dismissal on all respondent students. Respondent students filed with RTC
Makati a TRO since they are currently enrolled. This was granted. A TRO was also issued enjoining
petitioners from dismissing the respondents. A day after the expiration of the temporary restraining
order, Dean del Castillo created a Special Board to investigate the charges of hazing against
respondent students Abas and Mendoza. This was requested to be stricken out by the respondents and
argued that the creation of the Special Board was totally unrelated to the original petition which alleged
lack of due process. This was granted and reinstatement of the students was ordered.

The students must be informed in writing of the nature and cause of any accusation against
them;
That they shall have the right to answer the charges against them with the assistance of
counsel, if desired:
They shall be informed of the evidence against them
They shall have the right to adduce evidence in their own behalf; and
The evidence must be duly considered by the investigating committee or official designated
by the school authorities to hear and decide the case.

Non v. Judge Dames, 185 SCRA 523 Kirsty Sarita

Facts: Petitioners, students in private respondent Mabini Colleges, Inc. in Daet, Camarines Norte, were
not allowed to re-enroll by the school for the academic year 1988-1989 for leading or participating in
student mass actions against the school in the preceding semester. The subject of the protests is not,
however, made clear in the pleadings.
Petitioners filed a petition in the court seeking their readmission or re-enrollment to the school, but the
trial court dismissed the petition. They now petition the court to reverse its ruling in Alcuaz vs. PSBA1,
which was also applied in the case. The court said that petitioners waived their privilege to be admitted
for re-enrollment with respondent college when they adopted, signed, and used its enrollment form for
the first semester of school year 1988-89, which states that: The Mabini College reserves the right to
deny admission of students whose scholarship and attendance are unsatisfactory and to require
withdrawal of students whose conduct discredits the institution and/or whose activities unduly disrupts
or interfere with the efficient operation of the college. Students, therefore, are required to behave in
accord with the Mabini College code of conduct and discipline.
Issue: Whether or Not the students right to freedom of speech and assembly infringed?
Held: Yes. The protection to the cognate rights of speech and assembly guaranteed by the Constitution
is similarly available to students is well-settled in our jurisdiction. However there are limitations. The
permissible limitation on Student Exercise of Constitutional Rights within the school presupposes that
conduct by the student, in class or out of it, which for any reason whether it stems from time, place, or
type of behavior should not materially disrupt class work or must not involve substantial disorder or
invasion of the rights of others.

ISSUE: Was there denial of due process against the respondent students?
HELD: There was no denial of due process, more particularly procedural due process. Dean of the
Ateneo Law School notified and required respondent students to submit their written statement on the
incident. Instead of filing a reply, respondent students requested through their counsel, copies of the
charges. The nature and cause of the accusation were adequately spelled out in petitioners' notices.
Present is the twin elements of notice and hearing.

Respondent students argue that petitioners are not in a position to file the instant petition under Rule 65
considering that they failed to file a motion for reconsideration first before the trial court, thereby by
passing the latter and the Court of Appeals. It is accepted legal doctrine that an exception to the
doctrine of exhaustion of remedies is when the case involves a question of law, as in this case, where
the issue is whether or not respondent students have been afforded procedural due process prior to
their dismissal from Petitioner University.

Guzman v. NU, 142 SCRA 699 Sali / Marines

FACTS: Petitioners Diosdado Guzman, Ulysses Urbiztondo and Ariel Ramacula, students of
respondent National University, sought relief from what they describe as their school's "continued and
persistent refusal to allow them to enroll."

Under the Education Act of 1982, 5 the petitioners, as students, have the right among others
"to freely choose their field of study subject to existing curricula and to continue their course therein up
to graduation, except in case of academic deficiency, or violation of disciplinary regulations." 6
Petitioners were being denied this right, or being disciplined, without due process, in violation of the
admonition in the Manual of Regulations for Private Schools 7 that "(n)o penalty shall be imposed upon
any student except for cause as defined in ... (the) Manual and/or in the school rules and regulations as
duly promulgated and only after due investigation shall have been conducted."

HELD: No. It is beyond dispute that a student once admitted by the school is considered enrolled for
one semester, It is provided in Paragraph 137 Manual of Regulations for Private Schools, that when a
college student registers in a school, it is understood that he is enrolling for the entire semester.
Likewise, it is provided in the Manual that the written contracts required for college teachers are for
one semester. It is thus evident that after the close of the first semester, the PSBA-QC no longer has
any existing contract either with the students or with the intervening teachers. Such being the case, the
charge of denial of due process is untenable.
The petition is hereby DISMISSED

ISSUE: Are disciplinary cases involving students entail proceedings and hearings similar to those
prescribed for actions and proceedings in courts of justice?
Malabanan v. Ramento, 129 SCRA 359 Allan R. Taga-oc
RULING: No. The imposition of disciplinary sanctions requires observance of procedural due process.
The proceedings in student discipline cases may be summary. There are withal minimum standards
which must be met to satisfy the demands of procedural due process, and these are, that (1) the
students must be informed in writing of the nature and cause of any accusation against them; (2) they
shag have the right to answer the charges against them, with the assistance of counsel, if desired; (3)
they shall be informed of the evidence against them; (4) they shall have the right to adduce evidence in
their own behalf; and (5) the evidence must be duly considered by the investigating committee or official
designated by the school authorities to hear and decide the case.
WHEREFORE, the petition is granted and the respondents are directed to allow the petitioners
to re-enroll or otherwise continue with their respective courses, without prejudice to any disciplinary
proceedings to which any or all of them may be subjected in accordance with the standards herein set
forth.

FACTS:
Petitioners were officers of the Supreme Student Council of the Gregorio Araneta University
Foundation. They were granted a permit to hold a meeting to protest the merger of two units of the
university. On the scheduled date, the students continued their meeting beyond the scheduled time and
held it in a different place from that indicated in the permit. They expressed in a vehement language
their opposition to the merger and as a result, classes and office work was disturbed. Petitioners were
placed under preventive suspension. On appeal, they were found guilt of holding an illegal assembly
and oral defamation. They were suspended for one academic year. They filed a petition for certiorari in
the SC.
HELD: The petititon may be considered moot and academic considering that the TRO issued by the
SC allowed the students to enroll. But there is a need to pass squarely on the constitutional question.
Respect for the constitutional rights of peaceable assembly and free speech calls for the setting aside
of the order of suspension. Suspending them for one year is out of proportion considering that the
vigorous presentation of views was expected. The excitement of the occasion, the propensity of
speakers to exaggerate and the exuberance of the youth should be taken into consideration.

Alcuaz v. PSBA, 161 SCRA 7 Hasel Daluz

UP Board of Regents v. Telan, GR 110280, Oct. 21, 1996 Manalo

FACTS: Petitioners are all bonafide students of the Philippine School of Business Administration,
Quezon City. The students of the respondent school and the respondent PSBA, Q.C. had already
agreed on certain matters which would govern their activities within the school. On the exercise of
students democratic rights, it has been agreed that protest actions can be conducted any day as long
as they meet the following requirements: a) that they be held at the PSBA quadrangle from 12:30 pm
to 1:00 pm only; b) that the protest action be removed to the PSBA parking lot if it will exceed the 1:00
time limit; c) that if the protest move exceeds 1:00 it will be limited only up to 2:30 pm; d) However,
before any action is taken the organizers of the protest action should secure a permit 6 days before, or
if on the same day, it still be under the first-come-first-served basis in the use of facilities, volume of
sound system shall be adjusted so as not to disturb classes.It is the firm stand of the administration of
PSBA that it will not allow the students to directly participate in the policy-making body of the school. In
spite of the above-stated agreement, petitioners felt the need to hold dialogues and demanded the
negotiation of a new agreement, which demand was turned down by the school, resulting in mass
assemblies and barricades of school entrances. During the regular enrolment period, petitioners and
other students similarly situated were allegedly blacklisted and denied admission for the second
semester of school year 19861987 which prompted the President of the Student Council to file a
complaint with the Director of the MECS against the PSBA for barring the enrolment of the Student
Council Officers and student leaders.

FACTS: THE UP Board of Regents imposed on Nadal the penalties of suspension for one year, nonissuance of any certificate of good moral character during the suspension and/or as long as Nadal has
not reimbursed the STFAP benefits he had received with 12% interest per annum and non issuance of
his transcript of records until he has settled his financial obligations with the university. The disciplinary
action is meted after finally rendering a guilty verdict on Nadals alleged willfull withholding of the
following information in his application for scholarship tantamount to acts of dishonesty, viz: (1) that he
has and maintains a car and (2) the income of his mother in the USA in support of the studies of his
brothers. Nadal complained that he was not afforded due process when, after the Board Meeting on his
case on March 28, 1993 that resulted in a decision of NOT GUILTY in his favor, the Chairman of the
UP Board of Regents, without notice to the petitioner, called another meeting the following day to
deliberate on the Chairmans Motion for Reconsideration, which this time resulted in a decision of
GUILTY. Upon petition, Nadal was granted his action for mandamus with preliminary injunction.
ISSUE: WON Nadal was denied due process.
HELD: No. It is gross error to equate due process in the instant case with the sending of notice of the
March 29, 1993 BOR meeting. University rules do not require the attendance in BOR meetings of
individuals whose cases are included as items on the agenda of the Board. At no time did respondent
complain of lack of notice given to him to attend any of the regular and special BOR meetings where his
case was up for deliberation. Let it not be forgotten that respondent aspires to join the ranks of
professionals who would uphold truth at all costs so that justice may prevail. Nadal has sufficiently
proven to have violated his undertaking to divulge all information needed when he applied for the
benefits of the STFAP. Unlike in criminal cases which require proof beyond reasonable doubt as basis

ISSUE: whether or not there has been deprivation of due process for petitioners-students who have
been barred from re-enrollment and for intervenors-teachers whose services have been terminated
as faculty members, on account of their participation in the demonstration or protest charged by
respondents as anarchic rallies, and a violation of their constitutional rights of expression and
assembly?

for a judgment, in administrative or quasi-judiciall proceedings, only substantial evidence is required,


that which means a reasonable mind might accept a relevant evidence as adequate to support a
conclusion.

Issue:

Whether or not the respondent Commission had committed grave abused of discretion in
issuing

a provisional authority in favor of PLDT, without prior notice to the petitioners.


Lao Gi Alias Chia, Jr. v. CA 180 SCRA 756 Guisadio
Ruling: The Public Service Commission found that the application involved in the present petition is
Facts:
On September 3 1958, the secretary of Justice rendered a decision opinion no. 191
series of 1958 finding Filomeno Chia, Jr alias sian Pieng hu to be a Filipino citizen born on November
28, 1899 being the legitimate son of Inosencio Chia and Ma. Layug og Guagua, Pampanga. However
on October 3, 1980, the Minister of justice rendered opinion no. 147 series of 1980, cancelling opinion
no 191 series of 1958 and setting aside the citizenship oj filomino Chia, Sr on the ground that it was
founded on fraud and misinterpretation. A motion for reconsideration of said opinion was denied by
minister of justice on February 13, 1981.

actually an application for approval of rates for digital transmission service facilities, which it
may approve provisionally, and without the necessity of any notice and hearing as provided in
the above-quoted provision of law. Therefore, a careful study of the records yields no cogent
reason to disturb the findings and conclusions of the NTC.
Maceda v. ERB 199 SCRA 454 Abdurajak

Issue:
Whether or not the petitioners citizenship was secured by fraud is precisely the subject
matter of the proceedings before the Commission on Immigration and Deportation, in which no
evidence had been presented yet in support of the charge of fraud in the acquisition of petitioners
citizenship.

FACTS: In G.R. No. 96266, petitioner Maceda seeks nullification of the Energy Regulatory Board (ERB)
Orders dated December 5 and 6, 1990 on the ground that the hearings conducted on the second
provisional increase in oil prices did not allow him substantial cross-examination, in effect, allegedly, a
denial of due process. Upon the outbreak of the Persian Gulf conflict on August 2, 1990, private
respondents oil companies filed with the ERB their respective applications on oil price increases
(docketed as ERB Case Nos. 90-106,90-32 and 90-384, respectively. On September 21, 1990, the ERB
issued an order granting a provisional increase of P 1.42 per loter. Petitoner Maceda filed a petition foe
prohibition on September 26, 1990 (E. Maceda v. ERB, ET AL., G.R. No. 95203), seeking to nullify the
provisional increase. The petition was dismissed on December 18, 1990, reaffirming ERBS authority to
grant provisional increase even without prior hearing, pursuant to Sec. 8 of E.O. No. 172. Petitioner
Maceda together with petitioner Original ( G.R. No. 96349) also claim that the provisional increase
involved amounts over and above that sought by the petitioning oil companies.

Ruling:
Deportation proceeding does not partake of the nature of a criminal action, the
constitutional right of a person should not be denied: the rules on criminal procedure in the rules of
court are applicable to deprtation.
Commission on Immigration and deportation; Citizenship: the Commission has the authority and
jurisdiction to hear and determine the question of citizen raised.
PHILCOMSAT v. Alcuaz 180 SCRA 218 Ottong
Radio Communicattions v. NTC 184 SCRA 517 Lagbas

ISSUE: Whether ERB will grant provisional increases sought by the oil companies.

This case is a Petition for certiorari and prohibition with preliminary injunction to review the order of the

HELD: In G.R. No. 96349, petitioner Original additionally claims that if the price increase will be used to
augment the OPSF this will constitute illegal taxation. In the Maceda case, (G.R. Nos. 95203-05, supra)
this Court has already ruled that "the Board Order authorizing the proceeds generated by the increase
to be deposited to the OPSF is not an act of taxation but is authorized by Presidential Decree No. 1956,
as amended by Executive Order No. 137.The petitions of E.O. Original et al. (G.R. No. 96349) and C.S.
Povedas, Jr. (G.R. No. 96284), insofar as they question the ERB's authority under Sec. 8 of E.O. 172,
have become moot and academic. We lament Our helplessness over this second provisional increase
in oil price. We have stated that this "is a question best judged by the political leadership" (G.R. Nos.
95203-05, G.R. Nos. 95119-21, supra). We wish to reiterate our previous pronouncements therein that
while the government is able to justify a provisional increase, these findings "are not final, and it is up to
petitioners to demonstrate that the present economic picture does not warrant a permanent increase."
In this regard, we also note the Solicitor General's comments that "the ERB is not averse to the idea of
a presidential review of its decision," except that there is no law at present authorizing the same.
Perhaps, as pointed out by Justice Padilla, our lawmakers may see the wisdom of allowing presidential
review of the decisions of the ERB since, despite its being a quasi-judicial body, it is still "an
administrative body under the Office of the President whose decisions should be appealed to the
President under the established principle of exhaustion of administrative remedies," especially on a
matter as transcendental as oil price increases which affect the lives of almost an Filipinos.
ACCORDINGLY, the petitions are hereby DISMISSED.
SO ORDERED.

National Telecommunications Commission.


Facts: On January 4, 1984, private respondent PLDT filed an application with respondent Commission
for the Approval of Rates for Digital Transmission Service Facilities under NTC Case No. 84
003. At the hearing, petitioner PT & T Co., along with other petitioners, came to know of the
pending petition through the former, appeared and moved for some time within which to file an
opposition or reply to said application and they alleged that neither respondent commission nor
private respondent PLDT informed them of the existence of this provisional authority. Also,
alleging that the application filed by the PLDT is not for approval as its caption misleadingly
indicates but for authority to engage in new services not covered by private respondents
franchise and certificate of public convenience and necessity. Petitioners further claimed that
PLDT is limited by its legislative franchise to render only radiotelephonic services, exclusive
of radiotelegraphic or record services.

On 16 October 1979, Evelyn Kintanar requested another and separate investigation from the
Department of Investigation, GSIS, Manila to which she was never considered for investigation
although she had requested such re-investigation. From the basis of the affidavits, it was concluded
that the loss of the two (2) checks occurred while they were in the custody of Evelyn Kintanar. As such,
Private respondent was dismissed from the service.
On 3 June 1980, Evelyn Kintanar filed an appeal with the Civil Service Commission,
contending that her summary dismissal was a violation of her right to security of tenure and of the
constitutional guarantee of due process. The CSC indorsed the appeal to GSIS President and General
Manager which informed that respondents dismissal was mandated under paragraph a of Section 40
Presidential Decree No. 807.
The lower court rendered a decision in favor of the Private Respondent. On Appeal, the Court
of Appeals affirmed. Petitioner moved for reconsideration, without success.

Abalos v. Civil Service Commission, 196 SCRA 81 Amilbahar


Facts: In its resolution dated January11, 1990, the Civil Service Commission affirmed the order of Civil
Service Regional Office No. 12 directing the reinstatement in the provincial engineers Office, Lanao del
Sur. Of Sergio Villabona and Eduardo Yap, Jr on the ground that they had been illegally dismissed.
Petioner Francisco A. Abalos is now before us and prays that the resolution be reversed it was issued
with grave abuse of discretion.
Private respondent Villabona and Yap, who allegedly executed on September 6, 1986,
affidavits in supports of the complaint. The sworn statements were disowned in a joint affidavits
executed on September 25, 1986, by the private respondent, who claim that the earlier statements
were issued by the petitioner to Vllabona and Yap on September 25, 1987, reading as follows:
You are hereby ordered to explain in writing within 72 hours from receip hereof why no
disciplinary action be taken against you for having recanted your sworn statement which was made the
basis for the filing of a criminal complaint against the late Governor Arsenio A. Quibranza. Thereby
causing embarrassment to the Office of the Provincial Governor.
In the meantime you are hereby suspended from work effective upon receipt thereof, until
after the termination of the investigation to be conducted in accordance with law.
On October 14, 1987, the private respondent were informed in another memorandum that the
formal changes for dishonesty and intentionally making false statements in material facts had been
filed against them which they should answer within 72hours.
The petitioner stresses that on September 28, 1987, the private respondents admitted their
guilt in affidavits which he said he submitted later to the Civil Service Commission as an annex to his
formal appeal. By pleading guilty, respondents waived whatever rights to a formal investigation they
had.
The private respondents have rejected their supposed confession as having been extracted
from them without benefit of legal assistance. They invoke article III, section 17, of the Constitution,
which to gather with section 12(1), and might have been the reason for the dismissal by the
Ombudsman on February 1, 1990, of the complaint for perjury filed against them by the petitioner.
The petitioner also invokes Section 40 of P.D.No. 807 and argues that in view of the private
respondents admission of the charge against them,

ISSUE: Whether or not Section 40 of Presidential Decree No. 907 constitutes deprivation of procedural
due process.
HELD: Yes. The employee must be informed of the charges against him and that he must have a
reasonable opportunity to present his side of the matter. The burden was upon petitioner of showing at
least a prima facie case that Evelyn Kintanar had indeed committed acts of dishonesty. In the case at
bar, records show that petitioner failed in producing convincing evidence of respondents guilt.
Petitioner has adduced no basis for overturning the factual conclusions reached by the trial court and
affirmed by the Court of Appeals.
Wherefore, premises considered, the present petition for review is hereby denied due course
for lack of merit. The decision of Court of Appeals is hereby affirmed.

FACTS: Challenged in this petition is the decision of the respondent NLRC holding Hellenic Philippine
Shipping Company liable for the illegal dismissal of Capt. Epifanio Siete, herein private respondent, and
awarding him salaries and other benefits corresponding to the unexpired portion of his employment
contract. Enforcement of this decision has meanwhile been held in abeyance pursuant to our temporary
restraining
order
dated
August
3,
1988.
Siete was employed on May 22, 1985, as Master of M/V Houda G by Sultan Shipping Co., Ltd., through
its crewing agent, herein petitioner. He boarded the vessel on May 24, 1985, at Cyprus. From there, it
sailed on June 1, 1985, to El Ferrol, Spain, where it loaded cargo that it subsequently discharged at
Tripoli, Lebanon, from June 25-29, 1985. It then proceeded back to Cyprus, arriving there on June 30,
1985.
On July 8, 1985, Capt. Wilfredo Lim boarded the vessel and advised Siete that he had instructions from
the owners to take over its command. These instructions were confirmed by a telex sent by Sultan
Shipping to Siete on July 10, 1985. Neither Lim nor the telex indicated the reason for his relief. The
private respondent claims this information was also withheld from him by the petitioner upon his
repatriation
to
Manila.
On July 12, 1985, Siete filed a complaint against the petitioner for illegal dismissal and non-payment of
his salary and other benefits under their employment contract. On September 6, 1985, the petitioner
alleged in its answer that the complainant had been dismissed because of his failure to complete with
the instruction of Sultan Shipping to erase the timber load line on the vessel and for his negligence in
the discharge of the cargo at Tripoli that endangered the vessel and stevedores. Siete denied these
averments in his reply dated September 23, 1985, and reiterated that he had not earlier been informed
of the cause of his dismissal and repatriation, either in Cyprus or later in Manila.
After considering the position papers and documentary evidence of the parties, Administrator Tomas D.
Achacoso of the Philippine Overseas Employment Administration (POEA) dismissed the complaint,

Issue: Whether or Not Section 40 could be validly applied against the private respondents to justify
their summary dismissal?
Ruling: The court had earlier entertained serious misgivings about the constitutionality of Section 40 as
against strong protest that it was violative of due process in so far as it deprived the Civil servant of the
right to defend himself against the ex parte decision to dismiss him.

Hellenic Philippine Shipping Inc v. Siete, 195 SCRA 179 Bagasina

GSIS v. CA, 201 SCRA 661 Florendo

FACTS: Respondent Evelyn T. Kintanar was employed by petitioner Government Service Insurance
System at its Cebu Branch, initially as a casual employee, and later as records clerk, and then as
control clerk. As a control clerk, she was assigned to the Records and Communication (mailing) section,
Information and Services Division, where her principal duties and functions were to receive checks
(specifically, policy loan, educational assistance loan, and miscellaneous checks) coming from the
accounting section and list down the checks personally by requiring them to secure release papers to
be signed by the division head, or to mail the checks to the borrower-members.
Sometime in 1979, two (2) members of the GSIS in separate affidavits complained that they
had not received their policy loan checks. As a result, two (2) investigations were conducted separately
of GSIS employees who might have been involved in the loss and fraudulent encashment of the two (2)
checks. All the declarations of the employees investigated were reduced to writing and duly subscribed
and sworn to.

10

This is a petition for review on certiorari of the Decision 1 rendered in NLRC Case No. 4-1272-85 dated
July 26, 1989, affirming the dismissal of the petitioner by the respondent bank, and reversing thereby
the Decision 2 of Labor Arbiter Benigno C. Villarente, Jr. of March 29, 1988 which declared the
petitioner's dismissal as illegal and ordered his reinstatement with backwages and benefits.

holding that there was valid cause for Siete's removal. 2 The decision placed much value on the various
communications presented by the petitioner to show that Siete was indeed guilty of the charges that
justified
his
separation.
On January 4, 1988, the private respondent appealed to the NLRC, contending that the records
presented by the petitioner were prepared long after his dismissal and were especially suspect because
they came from persons in the employ of Sultan Shipping. He insisted that he was dismissed without
even being informed of the charges against him or given an opportunity to refute them. He added that,
even assuming he was negligent in the unloading of the cargo at Tripoli, this shortcoming did not
warrant
such
a
severe
penalty
as
his
dismissal.
In its decision dated June 27, 1988, 3 the public respondent reversed the POEA Administrator, holding
that the dismissal violated due process and that the documents submitted by the petitioner were
hearsay,
self-serving,
and
not
verified.
Accordingly,
it
disposed
as
follows:
A new decision is entered finding the dismissal of complainant as illegal. Respondent is hereby ordered
to pay to the complainant his salaries, wages and other benefits corresponding to the unexpired portion
of his employment contract with Sultan Shipping Company, Ltd., dated May 22, 1985.

Espero Santos Salaw, petitioner, was employed as a credit investigator-appraiser by the herein private
respondent Associated Bank. After having been accused of selling banks foreclosed machines and
electric generators in the total amount of P60,000.00 and allegedly in cahoots with his supervisor as
found in the extracted sworn statement of the Criminal Investigative Service (CIS) of the Philippine
Constabulary, on April 1, 1985 herein private respondent terminated the employment of Salaw
effective March 27, 1985 for alleged serious misconduct or willful disobedience and fraud or willful
breach of the trust reposed on him by the private respondents. Emphasized on both investigations
conducted by the Philippine Constabulary and Personnel Discipline and Investigation Committee
(PDIC) was the absence or non- avail of legal counsel for the petitioner.
In the illegal dismissal filed by the petitioner against respondent bank, the labor arbiter decided in favor
the Salaw, but quickly overturned by the NLRC when the case was appealed. Hence, this case at bar.

ISSUE: Whether or not that NLRC committed grave abuse of discretion in reversing the findings of
POEA?

Issue: Whether or not petitioner was deprive of due process for not being represented by counsel
during investigation.

HELD: The Court notes that the reports submitted by the petitioner to prove its charges were all
prepared after the fact of Siete's dismissal and were signed by its own employees. 4 Their motives are
necessarily suspect. The mere fact that they have made such reports does not itself prove the charges,
which were investigated ex parte, if at all. It is not denied that Siete was not informed of the charges
beforehand or that he was given an opportunity to refute them. Even after his arrival in Manila, he was
kept in the dark about the reason for his dismissal. The excuse of the petitioner that it itself did not know
why he was dismissed, being only a crewing agent of Sultan Shipping, deserves no comment.
The
Labor
Code
provides
as
follows:
Sec. 1. Security of tenure and due process. No worker shall be dismissed except for a just or
authorized
cause
provided
by
law
and
after
due
process.
Sec. 2. Notice of dismissal. Any employer who seeks to dismiss a worker shall furnish him a written
notice stating the particular acts or omission constituting the grounds for his dismissal. In cases of
abandonment of work, the notice shall be served at the worker's last known address.

Held: The Supreme Court held that the investigation of petitioner Salaw violated his constitutional right
to due process, petitioner was perfunctorily denied the assistance of counsel during investigation. The
right to counsel, a very basic requirement of substantive due process, has to be observed as
guaranteed by the 1987 Constitution to person under investigation, be the proceeding administrate civil,
or criminal. These rights cannot be waived except in writing in the presence of counsel.
WHEREFORE, premises considered, judgment is hereby rendered SETTING ASIDE the appealed
decision of the NLRC REINSTATING the decision of the labor arbiter.

Sec. 5. Answer and hearing. The worker may answer the allegations stated against him in the notice
of dismissal within a reasonable period from receipt of such notice. The employer shall afford the
worker ample opportunity to be heard and to defend himself with the assistance of his representative, if
he
so
desires.
Sec. 6. Decision to dismiss. The employer shall immediately notify a worker in writing of a decision to
dismiss
him
stating
clearly
the
reasons
therefor.

Macayayong v. Ople 204 SCRA 372 Ynawat

FACTS: Petitioner Macayayong had been detailed with the various offices in the Office of the President
(the last one being the Board of Liquidators) from 1968 up to the time he was dropped from the roster
of the Department of Labor effective January 31, 1976. On January 5 and 19, 1976, the Secretary of
Labor notified Macayayong that his services would be terminated effective January 31, 1976 by reason
of his failure to report back to his Office which urgently needed his services. And on January 23, 1976,
the Secretary issued an order dropping Macayayong from the roster of the Department of Labor for
Abandonment of Post. The order is anchored on the alleged fact that Macayayong blatantly defied
repeated recall orders.
On February 12, 1976, petitioner appealed to the Civil Service Commission from the order
rendered by the Secretary of Labor. On April 26, 1976, the Civil Service Commission chairmen by
respondent Clave rendered Resolution No. 351 affirming the appealed order of respondent Ople. On
May 19, 1977, the petitioner filed another appeal with the Office of the President from Resolution No.
351 of the Civil Service Commission. On June 20, 1979, after the lapse of almost two (2) years,
respondent Clave, as Presidential Assistant, affirmed his own appealed resolution rendered in his role

We are not persuaded that the NLRC committed grave abuse of discretion in reversing the findings of
the POEA sustaining the petitioner and dismissing the private respondent's complaint. On the contrary,
we agree that the private respondent was illegally dismissed because, first, he was not accorded a fair
investigation as required by law, and second, because the grounds invoked for his separation have not
been
proved
by
the
petitioner.
WHEREFORE, the challenged decision as above modified is AFFIRMED and the petition DISMISSED,
with costs against the petitioner. The temporary restraining order dated August 3,1988, is LIFTED.
SO ORDERED.
Salaw v. NLRC, 202 SCRA 7 Isnani

11

as Chairman of the Civil Service Commission. Hence, this petition for certiorari and mandamus. In a
resolution dated February 13, 1980, the Court gave due course to the petition and required both parties
to submit their respective memoranda. After the required memoranda were filed, the Court declared the
case submitted for decision in the resolution dated April 30, 1980.

CHR v. CSC, 227 SCRA 42 Hassan


People v. Nazario 165 SCRA 136 Ariel Larete

ISSUE: Whether or not petitioner's summary dismissal is in violation of the due process of law?
FACTS: Eusebio Nazario was charged in violation of refusal and failure to pay his municipal taxes
amounting to Php 362.62 because of his fishpond operation provided under Ordinance 4, Series of
1955, as amended. He is a resident of Sta. Mesa Manila and just leases a fishpond located at Pagbilao,
Quezon with the Philippine Fisheries Commission. The years in question of failure to pay were for 1964,
1965, and 1966. Nazario did not pay because he was not sure if he was covered under the ordinance.
He was found guilty thus this petition.
ISSUES:
1. Whether or not Ordinance 4, Series of 1955, as amended null and void for being ambiguous and
uncertain
2. Whether or not the ordinance was unconstitutional for being ex post facto
HELD:
1. No, the coverage of the ordinance covers him as the actual operator of the fishpond thus he comes
with the term Manager. He was the one who spent money in developing and maintaining it, so despite
only leasing it from the national government, the latter does not get any profit as it goes only to Nazario.
The dates of payment are also clearly stated Beginning and taking effect from 1964 if the fishpond
started operating in 1964.
2. No, it is not ex post facto. Ordinance 4 was enacted in 1955 so it cant be that the amendment under
Ordinance 12 is being made to apply retroactively. Also, the act of non-payment has been made
punishable since 1955 so it means Ordinance 12 is not imposing a retroactive penalty.
The appeal is DISMISSED with cost against the appellant.

HELD: Petitioner alleged that the order of summary dismissal on the ground of abandonment of post
was rendered without observing the due process requirement of the law and in violation of his security
of tenure. On the other hand, respondent alleged that petitioner was never denied due process. In fact,
he was sent the necessary notifications and was given an ultimatum to report back to his home office
which he disregarded. This may be considered as substantial compliance with the due process rule.
The petition is without merit. Petitioner contends that he was denied due process because he was not
given enough time to obey said recall. He alleges that the January 5 and 19 letters sent to him were
received together with the order of the Secretary of Labor on January 30, 1976, which was the eve of
the effectively of his dismissal on January 31, 1976. Hence, he was not given the opportunity to be
heard. Petitioner's contention is untenable. The records show that before petitioner was dropped from
the roster of the Department of Labor, he was notified twice by his home office to report back to work,
on January 5 and on January 19, 1976. In the aforesaid notices, the petitioner was urged to come back
to his home office because his services were badly needed, with a warning that in case he fails to
report, he would be dropped from the roster of the Department of Labor.

Alonzo v. Capulong, 244 SCRA 80 Cabanlong

Facts
This is a petition for certiorari to set aside the order dated March 31, 1993 of respondent judge
Ignacio Capulong, which granted a writ of preliminary injunction enjoining petitioner from preventively
suspending private respondent Juliet Fajardo as manager of the Administrative Services Department of
the Home Development Mutual Fund.
On December 17, 1992 Celeste G. Al-Jawazneh, who had been supplying office uniforms and providing
transportation service to Pag-ibig Fund employees, wrote a letter to the Chief Executive Officer, herein
petitioner Zorayda Alonzo, complaining against private respondent.
On December 28, 1992, a formal charge was filed against private respondent for dishonesty,
misconduct, disgraceful and immoral conduct, contracting of loans of money or other property from
persons with whom the office of the employee concerned had business relations, and conduct
prejudicial to the best interest of the service.

Estate of Francisco v. CA 199 SCRA 595 Jimenez

Facts: Litigated herein is a Quonset building situated in Port Area, Strong Boulevard, Isabela, Basilan,
constructed by the American Liberation Forces in 1944 and was purchased by Gregoria Francisco in
1946, said lot stands on a lot owned by the Philippine Ports Authority and was declared for the
exclusive use of port facilities by virtue of Proclamation No. 83 issued by President Elpidio Quirino.
But in the year 1989, Tan Gin San (surviving spouse of Gregoria) was issued a permit to
occupy the lot where the building stands for a period of 1 year to expire on December 31 of the same
year by the Philippine Ports Authority of Zamboanga. The permitee was using the Quonset for the
storage of copra.

Issue:
Whether or not the suspension of the private respondent without first giving her the right
to be heard in her defense is valid?
Ruling:
The preventive suspension of a civil service employee or officer can be ordered even
without a hearing because such suspension is not a penalty but only a preliminary step in an
administrative investigation. The purpose is to prevent the accused from using his position or office to
influence prospective witnesses or tamper with the records which may be vital in the prosecution of the
case.
WHEREFORE, the petition for certiorari is GRANTED; the writ of preliminary injunction dated April 1,
1993 is ANNULLED and SET ASIDE; and Special Civil Action No. 93-033 of the respondent court is
DISMISSED.
SO ORDERED.

It was ordered demolished by respondent Municipal Mayor, Benjamin Valencia citing Zoning
Ordinance No. 147 of the municipality, noting its antiquated and dilapidated structure and stressing the
clean-up campaign on illegal squatters and unsanitary surroundings along Strong Boulevard.
Petitioner sought a Writ of Prohibition with Injunction and Damages but was denied. It was
reversed by the Court of Appeals on Jan 25, 1990 and ruled that the respondent Mayor was not vested
with power to order summarily, and without any judicial proceeding, the demolition of the Quonset
building which was not a nuisance per se and that petitioner is in legal possession of the land on which
the building stands by virtue of the permit issued by the Philippine Ports Authority of Zamboanga but

12

the Respondent Court reversed itself quashing the writ of prohibition and setting aside the order of
restitution and payment of attorneys fees, stating that although the respondent Mayor initially issued an
order of demolition without juridical process, the deficiency was remedied when the appellant filed a
petition for prohibition and injunction and was heard on oral argument after appellees filed their answer.

charged with the generation of funds and the assessment, levy and collection of taxes and other
imposts.
It alleges that prior to the issuance of Revenue Memorandum Circular (RMC) 47-91 on June
11, 1991, which implemented Value Added Tax (VAT) Ruling 190-90, copra was classified as
agricultural food product under Section 103(b) of the National Internal Revenue Code and, therefore,
exempt from VAT at all stages of production or distribution.

Issue: Whether or not Respondent Mayor could summarily, without juridical process, order the
demolition of petitioners Quonset building.
Held: Ordinance No. 147, enacted on 27 December 1977, and relied upon by respondents, is entitled
"An Ordinance Establishing Comprehensive Zoning Regulations for the Municipality of Isabela . . ." It is
not disputed that the Quonset building, which is being used for the storage of copra, is located outside
the zone for warehouses. It is referred to in Ordinance as a non-conforming structure, which should be
relocated. And in the event that an immediate relocation of the building can not be accomplished,
Section 16 of the Ordinance provides:

The petitioner contends that the Bureau of Food and Drug of the Department of Health and
not the Bureau of Internal Revenue (BIR) is the competent government agency to determine the proper
classification of food products. It cites the opinion of Dr. Quintin Kintanar of the Bureau of Food and
Drug to the effect that copra should be considered "food" because it is produced from coconut which is
food and 80% of coconut products are edible. The respondents, on the contrary, argue that the opinion
of the BIR, as the government agency charged with the implementation and interpretation of the tax
laws, is entitled to great respect.

A certificate of non-conformance for all non-conforming uses shall be applied for by


the owner or agent of the property involved within twelve (12) months from the
approval of this Ordinance; otherwise the non-conforming use may be condemned
or removed at the owner's expense.

Likewise, petitioner claims that RMC No. 47-91 is discriminatory and violative of the equal
protection clause of the Constitution because while coconut farmers and copra producers are exempt,
traders and dealers are not, although both sell copra in its original state. Petitioners add that oil millers
do not enjoy tax credit out of the VAT payment of traders and dealers.

Even granting that petitioner failed to apply for a Certificate of Non-conformance, the
foregoing provision should not be interpreted as authorizing the summary removal of a non-conforming
building by the municipal government. For if it does, it must be struck down for being in contravention of
the requirements of due process, as originally held by the respondent Court.

Thus, the present petition for prohibition and injunction seeking to nullify Revenue
Memorandum Circular No. 47-91 and enjoin the collection by respondent revenue officials of the Value
Added Tax (VAT) on the sale of copra by members of petitioner organization.

Petitioner was in lawful possession of the lot and Quonset building by virtue of a permit from
the Philippine Ports Authority (Port of Zamboanga) when demolition was effected. It was not squatting
on public land. Its property was not of trifling value. It was entitled to an impartial hearing before a
tribunal authorized to decide whether the Quonset building did constitute a nuisance in law. There was
no compelling necessity for precipitate action. It follows then that respondent public officials of the
Municipality of Isabela, Basilan, transcended their authority in abating summarily petitioner's Quonset
building. They had deprived petitioner of its property without due process of law. The fact that petitioner
filed a suit for prohibition and was subsequently heard thereon will not cure the defect, as opined by the
Court of Appeals, the demolition having been a fait accompli prior to hearing and the authority to
demolish without a judicial order being a prejudicial issue.

Issues: 1. Whether or not due process is observed?


Held: No hearing was
WHEREFORE, the petition is DISMISSED.

The judgment of respondent Court of Appeals, dated 13 June 1990, quashing the writ of
prohibition and setting aside the order of restitution and payment of attorneys fees is SET ASIDE; its
original Decision, promulgated on 25 January 1990, is REINSTATED. The case was REMANDED to
the Regional Trial Court of Basilan, Branch 2, for the determination of the just compensation due
petitioner for the demolition of its Quonset building.

Medenilla v. Civil Service Commission 194 SCRA 278 Tubo

Facts: Ardeliza Medenilla, a contruactual employee working as Public Relations Officer II for the
Department of Public Works and Highways (DPWH). Later in 1987, Medenilla was detailed as
Technical Assistant in the Office of the Assistant Secretary for Administration and Manpower
Management. On January 30, 1987, pursuant to Executive Order No. 124, a reorganization ensued
within the DPWH and all the positions therein were abolished. A revised staffing pattern together with
the guidelines on the selection and placement of personnel was issued, included in the revised staffing
pattern is the contested position of Supervising Human Resource Development Officer. On January 2,
1989, the petitioner was appointed to the disputed position. On January 27, 1989, respondents Amparo
Dellosa, Rosalinda Juria and Marita Burdeos together with others all of whom are employees in the
Human Resource Training and Material Development Division, Administrative and Manpower
Management Service of the DPWH, jointly lodged a protest contesting the appointment of the petitioner
to the position. The protestants alleged that since they are next-in-rank employees, one of them should
have been appointed. On August 2, 1989 the protest was dismissed. The private respondents appealed

Misamis Occidental Association V. DOF 238 SCARA 63 Delatado

Facts:
Petitioner Misamis Oriental Association of Coco Traders, Inc. is a domestic corporation
whose members, individually or collectively, are engaged in the buying and selling of copra in Misamis
Oriental. On the other hand, respondents represent departments of the executive branch of government

13

the decision to the Civil Service Commission. On February 28, 1990, the Commission promulgated a
resolution to disapprove the promotional appointment of Ardeliza Medenilla. The petitioner on March 23,
1990 filed a motion for reconsideration of the resolution. However, the Commission on May 23, 1990
denied the petitioner's motion for reconsideration, for lack of experience and eligibility is in question.

satisfactorily rated for his performance. On September 21, 1988, the Commission resolved the case in
petitioner's favor and held that the appellants were not accorded due process of law and were
separated from the service not in accordance with the prescribed rules on reorganization. The
appellants' separation from service is, therefore, considered illegal and hereby ordered that appellants
be reappointed to their previous positions or to positions of comparable or equitable rank without loss of
seniority and that they be paid back salaries reckoned from the dates of their termination. On
September 12, 1989, petitioner filed with the Commission a letter informing the latter of the Bureau's
refusal to reinstate him and requesting for the taking of remedial action by the Commission. These
actions on the part of petitioner allegedly remained unheeded. And on July 26, 1990, petitioner found
out that the Bureau filed on October 27, 1988 a motion for reconsideration of the September 21, 1988
resolution. On February 1, 1989, the Commission gave due course to the motion for reconsideration
thereby setting aside its September 21, 1988 resolution.

Issue: Whether or not Ardeliza Medenilla is eligible for the position of Supervising Human Resource
Development Officer.
Decision: Yes. The petitioner possesses the appropriate civil service eligibility and requisite educational
background. Hence, the resolution of the Commission on May 23,1990 considered petitioners PD No.
907 eligibility. As to the petitioners experience, the Commission failed to consider it. The petitioner has
plenty of experience in the field of Human Resource Department and her one year and seven months
with Guthrie-Jensen who was engaged in research relating to performance appraisal systems and merit
promotion systems which duties are all related to Human Resource Development proves the petitioner
possesses these skills in more than the private respondents. Petition granted, the resolutions issued by
Civil Service Commission dated February 28, 1990 and May 23, 1990 are set aside.

ISSUE Whether or not the respondent Commission erred in giving due course to the motion for
reconsideration of respondent Bureau.
HELD With respect to petitioners contention that he was denied due process when the
Commission heard the Bureaus motion for reconsideration without notice to him, We agree
with respondent Bureaus argument that the defect was cured by the filing by petitioner of his
Omnibus Motion on July 30, 1990. Thus, in Medenilla v. Civil Service Commission, We said that
the lack of notice to petitioner regarding the pending appeal and the hearing of said appeal was
cured by the filing of a motion for reconsideration. Denial of due process cannot be
successfully invoked where a party was given the chance to be heard on his motion for
reconsideration

Mendiola v. CSC, 221 SCRA 295 Fernandez

FACTS Petitioner has been an employee of the Bureau since May 21, 1973. In 1987, President
Corazon Aquino issued Executive Order No. 127 mandating the reorganization of the Department of
Finance. In accordance with the said Executive Order, the Commissioner of the Bureau issued a
memorandum dated January 19, 1988 4 to streamline the Bureau. The said memorandum provided the
priority for the separation of personnel, to wit:

"CATEGORY I Personnel with administrative, criminal and/or patently undesirable


personnel.

Rodriguez v. Project 6 Market Service Coop., Inc et al, GR 79968 San Luis

Facts:
Petitioner Rodriguez is a vendor occupying and leasing one of the stalls at the Project 6
Market, Project 6, Quezon City from respondent Project 6 Market Service Cooperative, inc..

CATEGORY II Personnel above 60 yrs. of age as of January 1, 1988, and NOT occupying
sensitive of key supervisory/managerial positions.

The instant petition stems from Civil Case No. Q-45781 filed in 1984 by the Cooperative
against petitioner with Metropolitan Trial Court (MTC), Branch 39, Quezon City. The judgment of the
MTC having become final and executory, the Cooperative moved for its execution with the RTC and a
writ of execution was issued accordingly. Petitioner however moved to quash the writ on the ground that
the situation of the parties had changed after trial that would now rendered execution inequitable.

CATEGORY III Personnel with family relations within the Second Degree of consanguinity
or affinity.
CATEGORY IV Personnel directly or indirectly in the management or control of any private
enterprise which may be affected by the functions of EIIB, and

Issued: Whether the filling of Civil Case No. Q-49819 would render execution of judgment in Civil
Case No. Q-47100 unjust and justify execution.

CATEGORY V Personnel other than the above but willing to be separated to take
advantage of gratuity pursuant to Executive Order No. 127."

Held:
In Civil Case No. Q-49819 was filed by petitioner against the Cooperative on January 21,
1987. petitioner alleged that the lease over his market stall was a verbal lease-purchase agreement,
and that when Civil Case No. Q-47100, he discovered that this contract was legally untenable since the
cooperative was not the owner but mere lessee of the land on which the market building was
constructed.

On March 30, 1988, petitioner received a notice of termination from service effective at the close of
office hours of April 30, 1988. Alleging that he was not informed of the cause of his dismissal, petitioner
appealed his case to the chairman of the Appeals Board. His appeal was denied. Subsequently, he
appealed to the Commission and averred that he was denied due process when he was dismissed from
the service. Furthermore, he claimed that he could have been included in Category I of the January 19,
1988 memorandum. However, he contested such inclusion because he had been commended and

Clearly , the existence of Civil Case No. Q-49819 does not in any way effect Civil Case No. Q-97100.
for one, the issue in Civil Case No Q-49819 was the validity of the lease-purchase agreement between

14

petitioner and the Cooperative while that in Civil Case No. Q-47100 was the non-payment of rent and
the fact of physical possession of the leased property.
However, petitioner admits that when he moved for reconsideration of said order, he received his copy
of the opposition and respondent judge conducted a hearing on his motion.
IN VIEW WHEREOF, the petition is dismissed, the temporary restraining order is lifted and the order
dated August 31, 1987 and September 18, 1987 of respondent judge are affirmed. Costs against
petitioner.
SO ORDERED.

Stronghold Insurance v. CA 205 SCRA 605 Aripin

FACTS: The petitioner STRONGHOLD INSURANCE COMPANY, INC invokes due process to escape
liability on a surety bond executed for the protection of a Filipino seaman.
Lazo v. CSC, 236 SCRA 469 Hussin
Acting on behalf of its foreign principal, Qatar National Fishing Co., Pan Asian Logistics and Trading, a
domestic recruiting and placement agency, hired Adriano Urtesuela as captain of the vessel M/V Oryx
for the stipulated period of twelve months. The required surety bond, in the amount of P50,000.00,
was submitted by Pan Asian and Stronghold Insurance Co., Inc., the herein petitioner, to answer for the
liabilities of the employer. Urtesuela assumed his duties on April 18, 1982, but three months later his
services were terminated and he was repatriated to Manila. He thereupon filed a complaint against Pan
Asian and his former employer with the Philippine Overseas Employment Administration for breach of
contract and damages.

Facts:
Then case involves a question on the validity of the decision of the Civil Service Commission revoking
herein petitioners eligibility for being null and void on the ground that it violates the petitioners right to
due process of law. According to the public respondent, the revocation was premised on a letter stating
that petitioner's actual score was 34.48%, not 76.46% as indicated in his certificate of eligibility. The
Civil Service Commission Regional office charged petitioner with dishonesty, grave misconduct and

POEA rendered a decision in his favor, representing his salaries for the unexpired portion of his contract
and the cash value of his unused vacation leave, plus attorney's fees and costs, which the respondents
were required to pay. The judgment eventually became final and executory, not having been appealed
on time. Pursuant thereto, a writ of execution was issued against Pan Asian then Urtesuela filed a
complaint with the Insurance Commission against Stronghold on the basis of the aforementioned surety
bond and prayed for the value thereof plus attorney's fees and litigation costs.

conduct prejudicial to the best interests of the service, and ordered the Regional Office to conduct anew
a formal investigation of the case. Petitioner asked for reconsideration, alleging that Resolution No. 92837 was issued in violation of his right to due process and that the CSC had found him to have failed
the Civil Service Examinations without evidence being presented to support the finding.
Issue:
Whether the petitioner was denied due process of law when the CSC revoked his certificate of eligibility
Held:
The Supreme Court held that Under the Constitution, the Civil Service Commission is the central

After hearing, the Insurance Commission held that the complaint should be reformed because the
provisions in the surety bond were not stipulations pour autrui to entitle Urtesuela to bring the suit
himself. It held that the proper party was the POEA. 1 This ruling was reversed on appeal by the
respondent court in its decision that, as the actual beneficiary of the surety bond, Urtesuela was
competent to sue Stronghold, which as surety was solidarily liable with Pan Asian for the judgment
rendered against the latter by the POEA.

personnel agency of the government charged with the duty of determining questions of qualifications of
merit and fitness of those appointed to the civil service. Its power to issue a certificate of eligibility
carries with it the power to revoke a certificate for being null and void.
The argument is made, however, that the CSC cannot motu propio revoke a certificate of eligibility

The petitioner asks for reversal of the Court of Appeals. It submits that the decision of the POEA is
not binding upon it because it was not impleaded in the complaint; it was not notified thereof nor did it
participate in the hearing; and it was not specifically directed to pay the damages awarded to the
complainant.

without notice and hearing to the examinees concerned. While this is true as a general proposition, in
the context of this case, which simply involves the rechecking of examination papers and nothing more
than a reevaluation of documents already in the records of the CSC according to a standard answer

The Court has gone over the decision and finds that the petitioner is "hoist by its own petard." For as
the quoted excerpt itself says, the case is "different from those in which the surety, by law and/or by the
terms of his contract, has promised to abide by the judgment against the principal and renounced the
right to be sued or cited." In the surety bond, the petitioner unequivocally bound itself:

key previously set by it, notice and hearing was not required. The question before the CSC did not
require any evidentiary hearing. Instead, what applied was the rule of res ipsa loquitur. Petitioner could
have examined the rechecking of his examination papers and, if he found anything wrong, he could
have asked for reconsideration. But, while he filed one in this case, he did not show that his score was

The petitioner contends, however, that the said stipulation is unconstitutional and contrary to public
policy, because it is "a virtual waiver" of the right to be heard and "opens wide the door for fraud and
collusion between the principal and the bond obligee" to the prejudice of the surety.

really 76.46%. He simply argued that he should not be made to answer for an irregularity in which he
had no participation and, on this basis, asked the CSC for a formal investigation.

15

The Court cannot agree. The circumstance that the chance to be heard is not availed of does not
disparage that opportunity and deprive the person of the right to due process. This Court has
consistently held in cases too numerous to mention that due process is not violated where a person is
not heard because he has chosen, for whatever reason, not to be heard.

Ruling: No, the petitioner, which is a corporate entity, has no personality to invoke the right to be
presumed innocent which right is available only to an individual who is an accused in a criminal case.

In the proceedings before the Commission, the petitioner was given full opportunity (which it took) to
present its side, in its answer with counterclaim to the complaint, in its testimony at the hearings, in its
motion to dismiss the complaint, and in its 10-page memorandum. There is absolutely no question that
in that proceeding, the petitioner was actually and even extensively heard.

Alba v. Hon. Deputy Ombudsman, GR 120223 March 13, 1996 Maria Fatima Orbecido
FACTS: The instant motion for reconsideration has its origin in an administrative case filed with the
office of the OMBUDSMAN for Mindanao by private respondents Jesiela Antiporta and Aida Salmeo
(aggrieved graduating students of AIMSFI ) against petitioner Dr. Ramon Alba in his capacity as
Director III of the DECS accusing the latter of violating certain provisions of the Code of Conduct and
Ethical Standard for Public Officials and employees (RA 6713). That the herein petitioner acted against
the interest of the aggrieved graduating students and was partial to the AIMSFI owners. For such gross
misconduct, petitioner was meted a suspension of 30 days without pay and warned that any other
instance of non-observance of the Code of Conduct will result in graver punishment. When petitioner's
motion for reconsideration of the foregoing resolution was denied he filed an "Appeal/Petition for
certioari or prohibition with prayer for TRO and Writ of Preliminary Injunction" petition with this court.

ISSUE: Whether or not due process was not violated?


HELD: Right to be heard; Due process is not violated where a person is not heard because he has
chosen, for whatever reason, not to be heard; If he opts to be silent where he has a right to speak, he
cannot later complain that he was unduly silenced.The Court cannot agree. The argument assumes
that the right to a hearing is absolute and may not be waived in any case under the due process clause.
This is not correct. As a matter of fact, the right to be heard is as often waived as it is invoked, and
validly as long as the party is given an opportunity to be heard on his behalf. The circumstance that the
chance to be heard is not availed of does not disparage that opportunity and deprive the person of the
right to due process. This Court has consistently held in cases too numerous to mention that due
process is not violated where a person is not heard because he has chosen, for whatever reason, not
to be heard. It should be obvious that if he opts to be silent where he has a right to speak, he cannot
later be heard to complain that he was unduly silenced. [Stronghold Insurance Co., Inc. vs. Court of
Appeals, 205 SCRA 605(1992)]

ISSUE: Whether or not petitioner was deprived of due process and denial of his constitutional right to
property. Whether or not the 30-day suspension of petitioner without pay and "unappealable" imposed
by herein respondent was valid and in accordance with due process.
HELD: No, he was not deprived of due process and there wasn't any denial of his constitutional right to
property. The constitutional requirement of due process may be satisfied notwithstanding the denial of
the right to appeal for the essence of due process is simply the opportunity to be heard and to present
evidence in support of one's case. Moreover, as the court consistently held that the right to appeal is
not a natural right and a part of due process, it is merely a statutory privilege, and may be exercised
only in the manner and in accordance with the provision of the law. Hence, a party was given all the
opportunity to be heard, albeit through pleadings, where the office of the Ombudsman not only gave
due course and consideration to his counter-affidavit but also entertained and resolved his motion for
reconsideration, which is not ordinarily allowed in the adjudication of administrative cases where the
penalty imposed is suspension of not more than one month. Thus, contrary to petitioner's claim, he was
in fact given all opportunity to be heard, albeit through pleadings.

Feeder International Line v. CA 197 SCRA 842 Joan Solatorio

Facts: The M/T ULU WAI a foreign vessel of Honduran registry, owned and operated by Feeder
International Shipping Lines of Singapore, left Singapore on May 6, 1986 carrying 1,100 metric tons of
gas oil and 1,000 metric tons of fuel oil consigned to Far East Synergy Corporation of Zamboanga,
Philippines.
On May 14, 1986, the vessel anchored at the vicinity of Guiuanon Island in Iloilo without notifying the
Iloilo customs authorities. The presence of the vessel only came to the knowledge of the Iloilo
authorities by information of the civilian informer in the area. Acting on said information, the Acting
District Collector of Iloilo dispatched a Customs team on May 19, 1986 to verify the report. The
Customs team found out that the vessel did not have on board the required ship and shipping
documents, except for a clearance from the port authorities of Singapore clearing the vessel for
Zamboanga.
The District Collector issued his decision finding the said vessel and her cargo in violation of the Tariff
and Customs Code of the Philippines (PD 1464). The same was affirmed by both Custom
Commissioner
and
Court
of
Tax
Appeals.

Telan v. CA 202 SCRA 534 - Ombra

Aris (Phils) Inc. v. NLRC 200 SCRA 246 Analyn Bongabong

Facts: On 11 April 1988, private respondents, who were employees of petitioner, aggrieved by
management's failure to attend to their complaints concerning their working surroundings which had
become detrimental and hazardous, requested for a grievance conference. As none was arranged, and
believing that their appeal would be fruitless, they grouped together after the end of their work that day
with other employees and marched directly to the management's office to protest its long silence and
inaction
on
their
complaints.
On 12 April 1988, the management issued a memorandum to each of the private respondents, who
were identified by the petitioner's supervisors as the most active participants in the "rally", requiring
them to explain why they should not be terminated from the service for their conduct. Despite their
explanation, private respondents were dismissed for violation of company rules and regulations, more
specifically of the provisions on security and public order and on inciting or participating in illegal strikes
or
concerted
actions.

Issue: Whether or not the petitioner was deprived of property without due process of law in that its right
to be presumed innocent was not recognized?

16

quashal of the TRO on the ground that it did not comply with Sec. 29 of R.A. 265. On July 19, 1985 the
trial court granted the relief sought and denied TSB for injunction. TSB filed a petition for certiorari
under Rule 65 of the Rules of Court dated July 25, 1985 seeking to enjoin the continued
implementation of the questioned MB resolution. On August 9, 1985 Central bank and Ramon V.
Tiaoqui file a motion to dismiss the complaint before the RTC for failure to state a cause of action. On
September 9, 1985 TSB filed an urgent motion in the RTC to direct receiver Ramon V. Tiaoqui to
restore TSB to its private management. On November 11, 1985 RTC denied petitioners motion to
dismiss and ordered receiver Tiaoqui to restore the management of TSB to its elected Board of
Directors and officers, subject to CB comptrollership. Petitioners elevated the twin orders of RTC to the
court of appeals on a petition for certiorari under Rule 65. On September 26, 1986 the court upheld the
orders of the trial court. On October 15, 1986, Central Bank and its appointed receiver, Ramon V.
Tiaoqui, filed a petition under Rule 45 of the Rules of Court praying that the decision of the court of
appeals in CAG.R. SP No. 07867 be set aside and that the civil case pending before the RTC of
Quezon City be dismissed. The Supreme Court affirmed the decision of the Court of Appeals except
insofar as it upholds the order of the trial court of November 11, 1985 directing Ramon V. Tiaoqui to
restore the management of Triumph Savings Bank to its elected Board of directors and Officers, which
is SET ASIDE.

Private respondents lost no time in filing a complaint for illegal dismissal against petitioner and Mr.
Gavino Bayan with the regional office of the NLRC at the National Capital Region, Manila. After due trial
the labor arbiter ordered Aris (Phils.), Inc. to reinstate Leodegario de Guzman and company to their
former respective positions or any substantial equivalent positions if already filled up, without loss of
seniority
right
and
privileges.
On 19 July 1989, de Guzman and company filed a Motion For Issuance of a Writ of Execution pursuant
to Section 12 of R.A. No. 6715 which provides that In any event, the decision of the Labor Arbiter
reinstating a dismissed or separated employee, in so far as the reinstatement aspect is concerned,
shall immediately be executory, even pending appeal. The employee shall either be admitted back to
work under the same terms and conditions prevailing prior to his dismissal or separation or, at the
option of the employer, merely reinstated in the payroll. The posting of a bond by the employer shall not
stay
the
execution
for
reinstatement
provided
therein."
On 21 July 1989, petitioner filed its Appeal. On 26 July 1989, the complainants, except Flor Rayos del
Sol, filed a Partial Appeal. On 10 August 1989, complainant Flor Rayos del Sol filed a Partial Appeal.
On 29 August 1989, petitioner filed an Opposition to the motion for execution alleging that Section 12 of
R.A. No. 6715 on execution pending appeal cannot be applied retroactively to cases pending at the
time of its effectivity. Petitioner submitted a Rejoinder to the Reply on 5 September 1989. On 5 October
1989, the Labor Arbiter issued an Order granting the motion for execution and the issuance of a partial
writ of execution "as far as reinstatement of herein complainants is concerned in consonance with the
provision
of
Section
2
of
the
rules
particularly
the
last
sentence
thereof."

Issue:May a Monetary Board resolution placing a private bank under receivership be annulled on the
ground of lack of prior notice and hearing?
Rulings:In sum, appeal to procedural due process cannot just outweigh the evil sought to be
prevented; hence, we rule that Section 29 of R.A. 265 is a sound legislation promulgated in accordance
with the Constitution in the exercise of police power of the state. Consequently, the absence of notice
and hearing is not a valid ground to annul a Monetary Board resolution placing a bank under
receivership. The absence of prior notice and hearing cannot be deemed acts of arbitrariness and bad
faith. Thus, an MB resolution placing a bank under receivership, or conservatorship for that matter, may
only be annulled after a determination has been made by the trial court that its issuance was tainted
with arbitrariness and bad faith. Until such determination is made, the status quo shall be maintained,
the bank shall continue to be under receivership.

Unable to accept the above Order, petitioner filed the instant petition on October 1989.
Issue:
Ruling: Police Power; Provision concerning the mandatory and automatic reinstatement of an
employee whose dismissal is found unjustified by the labor arbiter is a valid exercise of the
police power of the state and the contested provision is then a police legislation.
Respondent NLRC, through the Office of the Solicitor General, filed its Comment on 20
November 1989. Meeting squarely the issues raised by petitioner, it submits that the provision
concerning the mandatory and automatic reinstatement of an employee whose dismissal is
found unjustified by the labor arbiter is a valid exercise of the police power of the state and the
contested provision is then a police legislation. [Aris (Phil.) Inc. vs. NLRC, 200 SCRA
246(1991)]

Philippine Merchant Marine School v. CA L 112844 June 2, 1995 Lynette Latip


FACTS:
PHILIPPINE MERCHANT MARINE SCHOOL, INC. (PMMSI), was established in
Manila in 1950 to train and produce competent marine officers. For several times prior to 1985
respondent Department of Education, Culture and Sports (DECS) disapproved petitioner's requests for
renewal permit/recognition and from 1985-1988 PMMSI operated without such permit. Communications
from the DECS were blatantly disregarded by the school and repeated DECS inspection, surveys,
evaluations and reevaluations from 1988-1990 revealed that petitioner scored points for the courses
they offer way below the DECS requirements.
In 1989 a memorandum from the Bureau of Higher Education recommended gradual phasing
out of the schools maritime programs;if the school can come up with the DECS minimum standard
within the phasing out period, suspension order may be lifted otherwise, closure order will be issued.
After repeated re-inspection and dialogue with the Bureau of Higher Education, the matter was elevated
to the Office of the President on appeal of PMMSI.
In letters filed with the Office of the President in 1992 petitioner alleged compliance with
DECS requirements.During the pendency of the appeal, the DECS issued a Closure Order. On
November 1992 the Office of the President dismissed petitioners appeal. PMMSI moved for
reconsideration but the Office of the President did not set aside its previous resolution.Petitioner
assailed both resolutions of the Office of the President before respondent Court of Appeals by way of
certiorari. It alleged that the resolutions failed to meet the constitutional requirement of due process
because the basis for affirming the DECS phase-out and closure orders was not sufficiently disclosed.

Rivera v. CSC, 240 SCRA 43 Danilo Dinopol

Central Bank v. CA, 220 SCRA 536 Julius Camaingking

Facts: Based on the report submitted by Supervision and Examination Sector that Triumph Savings
banks financial condition is one of insolvency, Central Bank through Monetary Board on May 31, 1985
issued a resolution ordering the closure of TSB and appointing Ramon V. Tiaoqui as receiver. On June
11, 1985 TSB filed a complaint with RTC to annul MB resolution issued by Central Bank with prayer for
injunction on the ground that it was issued without prior notice and hearing. On July 1, 1985, the trial
court temporarily restrained petitioners for implementing MB resolution prompting them to move for the

17

Furthermore, its letters dated 2 and 3 October 1992 which presented incontrovertible proof that it had
introduced substantial improvements on its facilities for the past two and a half years while its appeal
was pending were not taken into account, thereby gravely abusing its discretion.

FACTS: Respondent GLORIOUS was found guilty of dollar-salting and misdeclaration of importations
by the Garments and Textile Export Board (GTEB) and, as a result of which, the export quotas allocated
to it were cancelled. Its quotas were given to two newly-formed corporationsDe Soleil Apparel
Manufacturing Corporation (De Soleil) and the American Inter-Fashion Corporation (AIFC). These two
corporations were joint ventures of the Hongkong investors and majority stockholders of Glorious Sun
on one hand and, allegedly, a member of the family and a crony of President Marcos on the other. The
Office of the President set aside the GTEB decision and remanded the case for genuine hearings
where due process would be accorded both parties. The petitioner now alleges that the GTEB decision
is res judicata and that Glorious Sun was given every opportunity to be heard by the Board. Before the
cancellation in 1984, the private respondent Glorious had been enjoying export quotas granted to it
since 1977

ISSUE: Was PMMSI deprived of its right to a hearing and did the resolutions of the Office of the
President fail to meet the constitutional requirement of due process because the basis for affirming the
DECS phase-out and closure orders was not sufficiently disclosed?
RULING: NO. The Supreme Court held that the assertion of petitioner that it was deprived of its right to
a hearing or any opportunity whatsoever to correct the alleged deficiencies is untenable. The earlier
narration of facts clearly demonstrates that before the DECS issued the phase-out and closure
orders, petitioner was duly notified, warned and given several opportunities to correct its
deficiencies and to comply with pertinent orders and regulations since 1986.
Petitioner has gone all the way up to the Office of the President to seek a reversal of the
phase-out and closure orders. There is thus no reason to complain of lack of opportunity to explain its
side as well as to comply with the alleged deficiencies. As long as the parties were given opportunity to
be heard before judgment was rendered, the demands of due process were sufficiently met (Lindo v.
COMELEC, 194 SCRA 25). The SC also noted that petitioner repeatedly sought reconsideration of the
various orders of respondent DECS and its motions were duly considered by respondent DECS to the
extent of allowing and granting its request for re-inspection of its premises. In connection therewith, it
has been ruled that the opportunity to be heard is the essence of procedural due process and that any
defect is cured by the filing of a motion for reconsideration (Medenilla v. Civil Service Commission, 194
SCRA 278).

ISSUE: Whether or not the cancellation of the export quotas of the private respondent GTEB violated
the private respondents constitutional right to due process?
HELD: The court held that the record clearly manifests that in cancelling the export quotas of the private
respondent GTEB violated the private respondents constitutional right to due process and the fact that
before the cancellation in 1984, the private respondent had been enjoying export quotas granted to it
since 1977 such in effect, the private respondents export quota allocation which initially was a privilege
evolved into some form of property right which should not be removed from it arbitrarily and without due
process only to hurriedly confer it on another. These privileges have been accorded to petitioner for so
long that they have become impressed with property rights.

Concerned Officials of MWSS v. Vasquez, 240 SCRA 502 Arola


Estrada v. Sandiganbayan, GR 148560 (Nov. 19, 2001) Richard Atamosa

FACTS: The ombudsman in its 19th October 1992 order directed the board of trustees of MWSS to a.)
set aside the recommendation of its pre-qualifications, bids and awards committee for construction
services and technical equipment (PBAC-CSTE) that contract no. APM-01 be given to a contract
offering fiberglass pipes and b.) to instead award the contract to a complying and responsive bidder
pursuant to the provision of the P.D 1594. The subsequent motion for reconsideration was denied by
the ombudsman in its order of 01 March 1993. These two orders are now sought to be annulled in this
petition for certiorari with prayer for preliminary injunction or a restraining order, lodged by the
concerned officials of the MWSS led by its former Administrator Teofilo I. Asuncion.

Facts: Petitioner Joseph Ejercito Estrada, the highest-ranking official to be prosecuted under RA 7080
(An Act Defining and Penalizing the Crime of Plunder), 1 as amended by RA 7659, 2 wishes to impress
upon us that the assailed law is so defectively fashioned that it crosses that thin but distinct line which
divides the valid from the constitutionally infirm. He therefore makes a stringent call for this Court to
subject the Plunder Law to the crucible of constitutionality mainly because, according to him, (a) it
suffers from the vice of vagueness; (b) it dispenses with the "reasonable doubt" standard in criminal
prosecutions; and, (c) it abolishes the element of mens rea in crimes already punishable under The
Revised Penal Code, all of which are purportedly clear violations of the fundamental rights of the
accused to due process and to be informed of the nature and cause of the accusation against him.

ISSUES: a. Whether or not due process had been properly observed in the issuance of the assailed
orders by the ombudsman.

ISSUES: WON the Plunder Law requires less evidence for providing the predicate crimes of plunder
and therefore violates the rights of the accused to due process.

HELD: The court are more than convinced that after a scrutiny of the records of the case, that
petitioners have been amply accorded the opportunity to be heard, that the essence of due process is
an opportunity to be heard not solely by verbal presentation but also, and perhaps even many times
more creditably and practicable that oral arguments, through pleadings. In administrative proceedings,
moreover technical rules of procedure and evidence are not strictly applied; administrative due process
cannot be fully equated to due process in its strict judicial sense. Wherefore, the petition is granted. The
questioned of 19th October 1992 order of the ombudsman as well as its March 01, 1993 order are
hereby annulled, and set aside.

Held: No. As long as the law affords some comprehensible guide or rule that would inform those who
are subject to it what conduct would render them liable to its penalties, its validity will be sustained. The
amended information itself closely tracks the language of law, indicating w/ reasonable certainty the
various elements of the offense w/c the petitioner is alleged to have committed.

No. Sec. 4 (Rule of Evidence) states that: For purposes of establishing the crime of plunder, it shall not
be necessary to prove each and every criminal act done by the accused in furtherance of the scheme
or conspiracy to amass, accumulate or acquire ill-gotten wealth, it being sufficient to establish beyond
reasonable doubt a pattern of overt or criminal acts indicative of the overall unlawful scheme or
conspiracy.

American Inter-Fashion Corp. v. Office of the President, 197 SCRA 409 Saladaga

18

Held: PREMISES CONSIDERED, this Court holds that RA 7080 otherwise known as the Plunder Law,
as amended by RA 7659, is CONSTITUTIONAL. Consequently, the petition to declare the law
unconstitutional is DISMISSED for lack of merit

coterminous with the effectivity of the contract executed by and between Corfarm and MERALCO for
the management of the latters commissary (hereinafter referred to as the management contract).On
31 December 1992, said management contract between Corfarm and MERALCO expired. However,
Corfarm continued to operate the MERALCO commissary despite the non-renewal of said contract. On
13 January 1993, petitioners received a memorandum, dated 12 January 1993, from private
respondents terminating their services effective on said date, allegedly for two reasons: 1) the
expiration of their employment contracts, these being coterminous with the management contract
between Corfarm and MERALCO; and, 2) the on-going evaluation of their past performances, and
investigation of the internal auditor of Corfarm of certain anomalous transactions involving them
(petitioners). On 2 February 1993, petitioners filed with the arbitration branch of the NLRC a complaint
for illegal dismissal against private respondents. On 20 June 1994, Labor Arbiter Facundo L. Leda
rendered a decision declaring the complainants to have been illegally dismissed and the respondents
ordered to reinstate them immediately to their former or substantially equivalent positions and to pay
them jointly and severally, the total amount of One Hundred Thirty-three Thousand Four Hundred Sixty
Pesos and 70/100 (P133,460.70) representing back wages and attorneys fees. Private respondents
appealed the aforementioned decision of the labor arbiter. On 24 November 1994, the NLRC
promulgated a decision setting aside the labor arbiters order and dismissing herein petitioners
complaint for lack of merit. Petitioners filed a motion for reconsideration of the NLRC decision, which
motion was denied. Hence, this petition where petitioners allege that public respondent NLRC gravely
abused its discretion in 1) reversing the labor arbiters decision finding the petitioners dismissal to have
been illegal for lack of due notice and hearing as required by law; and, 2) in ignoring the documents
and testimony contained in the record which support the labor arbiters decision finding the petitioners
without fault on the alleged acts attributed to them.

CIR v. CA 261 SCRA 236 Feliciano Beldad


FACTS: Fortune Tobacco Corporation ("Fortune Tobacco") is engaged in the manufacture of different
brands of cigarettes. The Philippine Patent Office issued to the corporation separate certificates of
trademark registration over "Champion," "Hope," and "More" cigarettes. The initial position of the CIR
was to classify 'Champion,' 'Hope,' and 'More' as foreign brands since they were listed in the World
Tobacco Directory as belonging to foreign companies. However, Fortune Tobacco changed the names
of 'Hope' to 'Hope Luxury ' and 'More' to 'Premium More,' thereby removing the said brands from the
foreign brand category.
RA No. 7654, was enacted and became effective on 03 July 1993. It amended Section 142(c)(1) of the
NIRC. About a month after the enactment and two (2) days before the effectively of RA 7654, Revenue
Memorandum Circular No. 37-93 ("RMC 37-93") Reclassification of Cigarettes Subject to Excise Tax,
was issued by the BIR. Fortune Tobacco requested for a review, reconsideration and recall of RMC 3793. The request was denied on 29 July 1993. The following day, or on 30 July 1993, the CIR assessed
Fortune Tobacco for ad valorem tax deficiency amounting to P9, 598, 334. 00.
On 03 August 1993, Fortune Tobacco filed a petition for review with the CTA. The CTA upheld the
position of Fortune Tobacco and adjudged RMC No. 37-93 as defective.

Issue:
Whether or not NLRC gravely abused its discretion in reversing the labors arbiters decision
finding the dismissal to have been illegal for lack of due notice and hearing as required by law?

ISSUE: Whether or not there is a violation of the due process of law.


Held: Due Process; The twin requirements of notice and hearing constitute essential elements of due
process in the dismissal of employees.This Court has consistently held that the twin requirements of
notice and hearing constitute essential elements of due process in the dismissal of employees. As to
the requirement of notice, it has been held that the employer must furnish the worker with two written
notices before termination of employment can be legally effected: (a) notice which apprises the
employee of the particular acts or omissions for which his dismissal is sought; and, (b) subsequent
notice which informs the employee of the employers decision to dismiss him.
The essence of due process is simply an opportunity to be heard, and not that an actual hearing should
always and indispensably be held.With regard to the requirement of a hearing, this Court has held
that the essence of due process is simply an opportunity to be heard, and not that an actual hearing
should always and indispensably be held.
Note.Due process was designed to afford an opportunity to be heard, not that an actual hearing
should always and indis-pensably be held. (Pamantasan ng Lungsod ng Maynila vs. Civil Service
Commission, 241 SCRA 506 [1995])

HELD: A reading of RMC 37-93, particularly considering the circumstances under which it has been
issued, convinces us that the circular cannot be viewed simply as a corrective measure or merely as
construing Section 142(c)(1) of the NIRC, as amended, but has, in fact and most importantly, been
made in order to place "Hope Luxury," "Premium More" and "Champion" within the classification of
locally manufactured cigarettes bearing foreign brands and to thereby have them covered by RA 7654.
In so doing, the BIR not simply interpreted the law; verily, it legislated under its quasi-legislative
authority. The due observance of the requirements of notice, of hearing, and of publication should not
have been then ignored. The Court is convinced that the hastily promulgated RMC 37-93 has fallen
short of a valid and effective administrative issuance.
DOCTRINE: Uniformity of taxation merely requires that all the subjects or objects of taxation, similarly
situated are to be treated alike both in privileges and liabilities. (Tan vs. Del Rosao, Jr., 237 SCRA 324
[1994])

Conti v. NLRC 271 SCRA 114 Allan Tamayao

Facts:
Petitioners Amor Conti and Leopoldo Cruz seek to annul the decision of the National Labor
Relations Commission setting aside the labor arbiters decision, declaring that herein petitioners were
illegally dismissed from employment; Petitioner Leopoldo Cruz was employed by the same respondent
corporation as a warehouseman on 16 May 1991. Both Amor Conti and Leopoldo Cruz were
subsequently promoted to the positions of Head of Commissary and Store Supervisor, respectively. In
their respective employment contracts with Corfarm, it was stipulated that their employment shall be

Joson v. Executive Secretary 290 SCRA 279 Kinang

FACTS The case at bar involves the validity of the suspension from office of petitioner Eduardo Nonato
Joson as Governor of the province of Nueva Ecija. Private respondent Oscar C. Tinio is the ViceGovernor of said province while private respondents Loreto P. Pangilinan, Crispulo S. Esguerra, Solita
C. Santos, Vicente C. Palilio and Napoleon G. Interior are members of the Sangguniang Panlalawigan.

19

Private respondents prayed for the suspension or removal of petitioner; for an emergency
audit of the provincial treasury of Nueva Ecija; and for the review of the proposed loan in light of the
financial condition of the province,
ISSUE

Garchitorena not waiting for Amores manifestation. Such procedural flaws committed by respondent
Sandiganbayan are fatal to the validity of its decision convicting petitioner. Garchitorena had already
created the Special Division of five (5) justices in view of the lack of unanimity of the three (3) justices in
the First Division. At that stage, petitioner had a vested right to be heard by the five (5) justices,
especially the new justices in the persons of Justices Amores and del Rosario who may have a different
view of the cases against her. At that point, Presiding Justice Garchitorena and Justice Balajadia may
change their mind and agree with the original opinion of Justice Atienza but the turnaround cannot
deprive petitioner of her vested right to the opinion of Justices Amores and del Rosario. It may be true
that Justice del Rosario had already expressed his opinion during an informal, unscheduled meeting in
the unnamed restaurant but as aforestated, that opinion is not the opinion contemplated by law. But
what is more, petitioner was denied the opinion of Justice Amores for before it could be given, Presiding
Justice Garchitorena dissolved the Special Division.

Whether or not the suspension from office of the petitioner Eduardo Nonato Joson is valid.

HELD: Rejection of petitioners right to a formal investigation denied him procedural due process.
The rejection of petitioners right to a formal investigation denied him procedural due process. Section 5
of A.O. No. 23 provides that at the preliminary conference, the Investigating Authority shall summon the
parties to consider whether they desire a formal investigation. This provision does not give the
Investigating Authority the discretion to determine whether a formal investigation would be conducted.
The records show that petitioner filed a motion for formal investigation. [Joson vs. Torres, 290 SCRA
279(1998)]
Petitioner's right to a formal investigation was not satisfied when the complaint against him
was decided on the basis of position papers. There is nothing in the Local Government Code and its
Implementing Rules and Regulations or in A.O. No. 23 that provide that administrative cases against
elective local officials can be decided on the basis of position papers. A.O. No. 23 states that the
Investigating Authority may require the parties to submit their respective memoranda but this is only
after formal investigation and hearing. A.O. No. 23 does not authorize the Investigating Authority to
dispense with a hearing especially in cases involving allegations of fact which are not only in contrast
but contradictory to each other. These contradictions are best settled by allowing the examination and
cross-examination of witnesses. Position papers are often-times prepared with the assistance of
lawyers and their artful preparation can make the discovery of truth difficult. The jurisprudence cited by
the DILG in its order denying petitioner's motion for a formal investigation applies to appointive officials
and employees. Administrative disciplinary proceedings against elective government officials are not
exactly similar to those against appointive officials. In fact, the provisions that apply to elective local
officials are separate and distinct from appointive government officers and employees. This can be
gleaned from the Local Government Code itself.

Pafianco v. Moral, 322 SCRA 439 Engr. Sali

Facts: SECRETARY ERLINDA C. PEFIANCO of the Department of Education, Culture and Sports
(DECS) seeks to nullify through this petition dismissing the petition filed by then DECS Secretary
Ricardo T. Gloria for lack of merit, as well as its Resolution dated 13 January 1998 denying
reconsideration thereof.
Issues : Whether the Court of Appeals erred in dismissing the petition for certiorari for failure of
petitioner to file a motion for reconsideration of the order denying the motion to dismiss, and in holding
that the trial court did not commit grave abuse of discretion in denying the motion to dismiss.
Held: More importantly, the DECS resolution is complete in itself for purposes of appeal to the Civil
Service Commission, that is, it contains sufficient findings of fact and conclusion of law upon which
respondents removal from office was grounded. This resolution, and not the investigation report,
should be the basis of any further remedies respondent might wish to pursue, and we cannot see how
she would be prejudiced by denying her access to the investigation report.

Marcos v. Sandiganbayan 12 LR 6 N98 Ria Jajurie

In fine, the trial courts Order of 23 April 1997 denying petitioners motion to dismiss is not a mere error
of judgment as the Court of Appeals held, but a grave abuse of discretion amounting to lack or excess
of jurisdiction because, to capsulize, the order is a patent nullity for failure to comply with the provisions
of the rules requiring that a resolution on a motion to dismiss should clearly and distinctly state the
reasons therefor; and, respondent is clearly not entitled to the writ of mandamus as she did not appeal
the DECS resolution dismissing her from service, and there is no law or rule which imposes a
ministerial duty on petitioner to furnish respondent with a copy of the investigation report, hence her
petition clearly lacked a cause of action. In such instance, while the trial courts order is merely
interlocutory and non-appealable, certiorari is the proper remedy to annul the same since it is rendered
with grave abuse of discretion.

Facts: Imelda was charged together with Jose Dans for Graft & Corruption for a dubious transaction
done in 1984 while they were officers transacting business with the Light Railway Transit (LRT). The
case was raffled to the 1st Division of the Sandiganbayan. The division was headed by Justice
Garchitorena with J Balajadia and J Atienza as associate justices. No decision was reached by the
division by reason of Atienzas dissent in favor of Imeldas innocence. Garchitorena then summoned a
special division of the SB to include JJ Amores and Cipriano as additional members. Amores then
asked Garchitorena to be given 15 days to send in his manifestation. On the date of Amores request,
Garchitorena received manifestation from J Balajadia stating that he agrees with J Rosario who further
agrees with J Atienza. Garchitorena then issued a special order to immediately dissolve the special
division and have the issue be raised to the SB en banc for it would already be pointless to wait for
Amores manifestation granted that a majority has already decided on Imeldas favor. The SB en banc
ruled against Imelda.

ISSUE: Whether or not due process has been observed.

Roxas v. CA 321 SCRA 106 Maria Katrina Tanjusay

FACTS: This case involves three (3) haciendas


validity of the acquisition of these haciendas by
Comprehensive Agrarian Reform Law of 1988.
haciendas, namely, Haciendas Palico, Banilad

HELD: The SC ruled that the ruling of the SB is bereft of merit as there was no strong showing of
Imeldas guilt. The SC further emphasized that Imelda was deprived of due process by reason of

20

in Nasugbu, Batangas owned by petitioner and the


the government under Republic Act No. 6657, the
Petitioner Roxas is the registered owner of three
and Caylaway, all located in the Municipality of

Nasugbu, Batangas. The events of this case occurred during the incumbency of then President
Corazon C. Aquino. In February 1986, President Aquino issued Proclamation No. 3 promulgating a
Provisional Constitution. As head of the provisional government, the President exercised legislative
power until a legislature is elected and convened under a new Constitution. In the exercise of this
legislative power, the President signed on July 22, 1987, Proclamation No. 131 instituting a
Comprehensive Agrarian Reform Program and Executive Order No. 229 providing the mechanisms
necessary to initially implement the program.
On July 27, 1987, the Congress of the Philippines formally convened and took over legislative power
from the President. This Congress passed Republic Act No. 6657, the Comprehensive Agrarian Reform
Law (CARL) of 1988. The Act was signed by the President on June 10, 1988 and took effect on June
15, 1988.
Before the laws effectivity, on May 6, 1988, petitioner filed with respondent DAR a voluntary offer to sell
Hacienda Caylaway pursuant to the provisions of E.O. No. 229. Haciendas Palico and Banilad were
later placed under compulsory acquisition by respondent DAR in accordance with the CARL. For a valid
implementation of the CAR Program, two notices are required: (1) the Notice of Coverage and letter of
invitation to a preliminary conference sent to the landowner; and (2) Notice of Acquisition. The
implementation of the CARL is an exercise of the States police power and the power of eminent
domain. To the extent that the CARL prescribes retention limits to the landowners, there is an exercise
of police power for the regulation of private property in accordance with the Constitution. But where, to
carry out such regulation, the owners are deprived of lands they own in excess of the maximum area
allowed, there is also a taking under the power of eminent domain. The taking contemplated is not a
mere limitation of the use of the land. What is required is the surrender of the title to and physical
possession of the said excess and all beneficial rights accruing to the owner in favor of the farmer
beneficiary.

later found respondent to have committed a breach of internal discipline by taking alcoholic drinks while
on duty.
ISSUE: WON the conviction of an offense for which he was not charged, which conviction is a nullity
because of the lack of procedural due process of law is legal?
RULING: Premises considered, we hold that the Court of Appeals correctly found that the decision of
the petitioners Board was rendered without or in excess of jurisdiction, as respondent Torcita was found
guilty of an offense for which he was not properly charged. A decision is void for lack of due process if,
as a result, a party is deprived of the opportunity of being heard (Palu-ay vs. CA, 293 SCRA 358). A
void judgment never acquires finality (Heirs of Mayor Nemencio Galvez vs. CA, 255 SCRA 672; Fortich
vs. Corona, 298 SCRA 678). Hence, aforementioned decision cannot be deemed to have become final
and executory.
WHEREFORE, the assailed decision dated September 1, 1997 of the Court of Appeals is
AFFIRMED and the instant petition is DISMISSED.

Nature: Petition for review of a decision of the Manila RTC


FACTS: On June 18, 1999 the Department of Justice received from the Department of Foreign Affairs a
request for the extradition of private respondent Mark Jimenez to the U.S. The Grand Jury Indictment,
the warrant for his arrest, and other supporting documents for said extradition were attached along with
the request. Charges include:

ISSUE:
Whether or not the procedure in sending of notices in the implementation of the
CAR program is necessary to comply with the requisites of due process
RULING: YES. The Court ruled in favor of the petitioner since it governs the extraordinary method of
expropriating private property, the CARL should be strictly construed. Consequently, faithful compliance
with its provisions, especially those which relate to the procedure for acquisition of expropriated lands,
should be observed. Therefore, the service by respondent DAR of the notices of acquisition to petitioner
by ordinary mail, not being in conformity with the mandate of RA 6657, is invalid and ineffective. With
more reason, the compulsory acquisition of portions of Hacienda Palico, for which no notices of
acquisition were issued by the DAR, should be declared invalid.

1. Conspiracy to commit offense or to defraud the US


2. Attempt to evade or defeat tax
3. Fraud by wire, radio, or television
4. False statement or entries
5. Election contribution in name of another
The Department of Justice (DOJ), through a designated panel proceeded with the technical evaluation
and assessment of the extradition treaty which they found having matters needed to be addressed.
Respondent, then requested for copies of all the documents included in the extradition request and for
him to be given ample time to assess it.

The petition is granted in part and the acquisition proceedings over the three haciendas are nullified for
respondent DARs failure to observe due process when it issued Certificates of Land Ownership Award
(CLOAs) to farmer beneficiaries over portions of petitioners land without just compensation to the
petitioner. In accordance with the guidelines set forth in this decision and the applicable administrative
procedure, the case is hereby remanded to respondent DAR for proper acquisition proceedings and
determination of petitioners application for conversion.

Sec. of Justice v. Lantion, 343 SCRA 377, GR. 139465, Oct. 17, 2000 Roel Marcial

The Secretary of Justice denied request on the ff. grounds:


1. He found it premature to secure him copies prior to the completion of the evaluation. At that point in
time, the DOJ is in the process of evaluating whether the procedures and requirements under the
relevant law (PD 1069Philippine Extradition Law) and treaty (RP-US Extradition Treaty) have been
complied with by the Requesting Government. Evaluation by the DOJ of the documents is not a
preliminary investigation like in criminal cases making the constitutionally guaranteed rights of the
accused in criminal prosecution inapplicable.

Summary Dismissal Board v. Toricta 330 SCRA 153, GR 130442, April 6, 2000 Cyril Joy
Casil

2. The U.S. requested for the prevention of unauthorized disclosure of the information in the
documents.

FACTS: A verified complaint was filed against C/Insp. LazaroTorcita, herein respondent, by Manuel
Puey, Jesus Puey, Alex Edwin del Rosario. Respondent was charged with 12 administrative complaints
into one major complaint, which is conduct unbecoming of a police officer. The Summary Dismissal
Board suspended respondent from the service for 20 days for simple irregularity of service. The Board

3. Finally, country is bound to Vienna convention on law of treaties such that every treaty in force is
binding upon the parties.

21

The respondent filed for petition of mandamus, certiorari, and prohibition. The RTC of NCR ruled in
favor of the respondent. Secretary of Justice was made to issue a copy of the requested papers, as
well as conducting further proceedings.

reversed its earlier Decision. It held that private respondent was bereft of the right to notice and
hearing during the evaluation stage of the extradition process. This Resolution has become final and
executory.
Finding no more legal obstacle, the Government of the United States of America, represented by the
Philippine DOJ, filed Petition for Extradition. The Petition alleged, that Jimenez was the subject of an
arrest warrant issued by the United States District Court for the Southern District of Florida on April 15,
1999. The warrant had been issued in connection with the following charges (1) conspiracy to defraud
the United States (2) tax evasion, (3) wire fraud, (4) false statements, (5) illegal campaign contributions.
In order to prevent the flight of Jimenez, the Petition prayed for the issuance of an order for his
immediate arrest pursuant to Section 6 of PD No. 1069.
Before the RTC could act on the Petition, Respondent Jimenez filed before it an Urgent
Manifestation/Ex-Parte Motion, which prayed that petitioners application for an arrest warrant be set
for hearing.
In its assailed May 23, 2001 Order, the RTC granted the Motion of Jimenez and set the case for hearing
on June 5, 2001. In that hearing, petitioner manifested its reservations on the procedure adopted by
the trial court allowing the accused in an extradition case to be heard prior to the issuance of a warrant
of arrest.
After the hearing, the court a quo required the parties to submit their respective memoranda. In his
Memorandum, Jimenez sought an alternative prayer: that in case a warrant should issue, he is allowed
to post bail in the amount of P100, 000.
The alternative prayer of Jimenez was also set for hearing on June 15, 2001. Thereafter, the court
below issued its questioned July 3, 2001 Order, directing the issuance of a warrant for his arrest and
fixing bail for his temporary liberty at one million pesos in cash. After he had surrendered his passport
and posted the required cash bond, Jimenez was granted provisional liberty via the challenged Order
dated July 4, 2001.

ISSUES: 1. WON private is respondent entitled to the two basic due process rights of notice and
hearing.
Held: Extradition; Court holds now that private respondent is bereft of the right to notice and hearing
during the evaluation stage of the extradition process.We now hold that private respondent is bereft
of the right to notice and hearing during the evaluation stage of the extradition process. There is no
provision in the RP-US Extradition Treaty and in P.D. No. 1069 which gives an extraditee the right to
demand from the petitioner Secretary of Justice copies of the extradition request from the US
government and its supporting documents and to comment thereon while the request is still undergoing
evaluation. We cannot write a provision in the treaty giving private respondent that right where there is
none. It is well-settled that a court cannot alter, amend, or add to a treaty by the insertion of any
clause, small or great, or dispense with any of its conditions and requirements or take away any
qualification, or integral part of any stipulation, upon any motion of equity, or general convenience, or
substantial justice.
Private respondents plea to due process collides with important state interests which cannot also be
ignored for they serve the interest of the greater majority.To be sure, private respondents plea for
due process deserves serious consideration involving as it does his primordial right to liberty. His plea
to due process, however, collides with important state interests which cannot also be ignored for they
serve the interest of the greater majority. The clash of rights demands a delicate balancing of interests
approach which is a fundamental postulate of constitutional law. The approach requires that we take
conscious and detailed consideration of the interplay of interests observable in a given situation or type
of situation. These interests usually consist in the exercise by an individual of his basic freedoms on
the one hand, and the governments promotion of fundamental public interest or policy objectives on the
other.

ISSUES: (1) Whether Jimenez is entitled to notice and hearing before a warrant for his arrest can be
issued.
(2) Whether he is entitled to bail and to provisional liberty while the extradition proceedings are pending.
RULING: The Petition is GRANTED. The assailed RTC Order dated May 23, 2001 is hereby declared
NULL and VOID, Trial Court of Manila is directed to conduct the extradition proceedings before it, with
all deliberate speed pursuant to the spirit and the letter of our Extradition Treaty with the United States
as well as our Extradition Law. No costs.
SO ORDERED

Government of the USA v. Purganan 389 SCRA 623 Sept. 24, 2002 Kirsty Sarita

FACTS: Pursuant to the existing RP-US Extradition Treaty the United States Government, through
diplomatic channels, request the extradition of Mark B. Jimenez, also known as Mario Batacan Crespo.
Upon receipt of the Notes and documents, the secretary of foreign affairs (SFA) transmitted them to the
secretary of justice (SOJ) for appropriate action, pursuant to Section 5 of Presidential Decree (PD) No.
1069, also known as the Extradition Law.
Upon learning of the request for his extradition, Jimenez sought and was granted a Temporary
Restraining Order (TRO) by the RTC of Manila, Branch 25. The TRO prohibited the Department of
Justice (DOJ) from filing with the RTC a petition for his extradition. The validity of the TRO was,
however, assailed by the SOJ in a Petition before this Court. Initially, the Court -- by a vote of 9-6 -dismissed the Petition. The SOJ was ordered to furnish private respondent copies of the extradition
request and its supporting papers and to grant the latter a reasonable period within which to file a
comment and supporting evidence.
Acting on the Motion for Reconsideration filed by the SOJ, this Court issued its October 17, 2000
Resolution. By an identical vote of 9-6 -- after three justices changed their votes -- it reconsidered and

Lazaro v. CA 330 SCRA 208 April 6, 2000 Sali / Marines

FACTS: Spouses Jose and Anita Alesna, private respondents herein, filed a civil action for annulment
of title, reconveyance (transfer back) and damages against Petitioners Gabriel Lazaro and the heirs of
Florencia Pineda and Eva Viernes.
After trial, the RTC rendered judgment in favor of the petitioners. Thereafter, the private
respondents filed a Notice of Appeal before the trial court.
In a Resolution dated June 17, 1998, the CA dismissed the appeal for failure of herein private
respondents to pay the required docket fees within the prescribed period. Thereafter, it issued its first
assailed Resolution dated July 31, 1998 granting their Motion for Reconsideration and reinstating the
appeal.
Subsequently, the petitioners also filed their own Motion for Reconsideration assailing the
said Resolution. As earlier stated, the CA denied their Motion.

22

ISSUE:
Whether or not petitioners failure to pay docket and other lawful fees within the
prescribed period deprived him of procedural due process?

ISSUE: Whether or not the said law is valid.


HELD: The SC ruled against Toribio. The SC explained that it is not a taking of the property for
public use, within the meaning of the constitution, but is a just and legitimate exercise of the
power of the legislature to regulate and restrain such particular use of the property as would be
inconsistent with or injurious to the rights of the publics. All property is acquired and held under
the tacit condition that it shall not be so used as to injure the equal rights of others or greatly impair the
public rights and interests of the community.
doctrine of the sovereign police power of the State, and confirm us in our opinion that the provision of
the statute in question being a proper exercise of that power is not in violation of the terms of section 5
of the Philippine Bill, which provide that "no law shall be enacted which shall deprive any person of life,
liberty, or property without due process of law," a provision which itself is adopted from the Constitution
of the United States, and is found in substance in the constitution of most if not all of the States of the
Union.

RULING: "Procedural rules are not to be belittled or dismissed simply because their non-observance
may have resulted in prejudice to a party's substantive rights. Like all rules, they are required to be
followed except only for the most persuasive of reasons when they may be relaxed to relieve a litigant
of an injustice not commensurate with the degree of his thoughtlessness in not complying with the
procedure prescribed." The Court reiterates that rules of procedure, especially those prescribing the
time within which certain acts must be done, "have oft been held as absolutely indispensable
(necessary) to the prevention of needless delays and to the orderly and speedy discharge of business.
The reason for rules of this nature is because the dispatch of business by courts would be impossible,
and intolerable delays would result, without rules governing practice. Such rules are a necessary
incident to the proper, efficient and orderly discharge of judicial functions." Indeed, in no uncertain
terms, the Court held that the said rules may be relaxed only in "exceptionally meritorious cases."
WHEREFORE, the Petition is hereby GRANTED. Bayombong, Nueva Vizcaya (Branch 27) in Civil
Case No.4058 is declared FINAL and EXECUTORY.

Preservation of the life of the carabao

Churchill v. Rafferty 32 Phil. 580 Manalo

Agabon v. NLRC 442 SCRA 573, GR 158693, Nov. 17, 2004 Hazel Dalus

Facts:
The case arises from the fact that defendant, Collector of Internal Revenue, would like to
destroy or remove any sign, signboard, or billboard, the property of the plaintiffs, for the sole reason
that such sign, signboard, or billboard is, or may be offensive to the sight. The plaintiffs allege
otherwise. Was there valid exercise of police power in this case?

FACTS: Private respondent Riviera Home Improvements, Inc. is engaged in the business of selling and
installing ornamental and construction materials. It employed petitioner Virgilio Agabon and Jenny
Agabon as gypsum board and cornice installers on January 2, 1992 until February 23, 1999 when they
were dismissed for abandonment of work. Petitioners then filed a complaint for illegal dismissal. The
Labor Arbiter rendered a decision declaring the dismissal illegal. On appeal, the NLRC reversed the
decision because it found that the petitioners had abandoned their work and were not entitled to
backwages and separation pay. The Court of Appeals in turn ruled that the dismissal of the petitioners
was not illegal because they had abandoned their employment.

Issue: Whether or not due process is observed?


Held: Yes. There can be no doubt that the exercise of the police power of the Philippine Government
belongs to the Legislature and that this power is limited only by the Acts of Congress and those
fundamentals principles which lie at the foundation of all republican forms of government. An Act of the
Legislature which is obviously and undoubtedly foreign to any of the purposes of the police power and
interferes with the ordinary enjoyment of property would, without doubt, be held to be invalid. But where
the Act is reasonably within a proper consideration of and care for the public health, safety, or comfort, it
should not be disturbed by the courts.
"The power vested in the legislature by the constitution to make, ordain, and establish all manner of
wholesome and reasonable laws, statutes, and ordinances, either with penalties or without, not
repugnant to the constitution, as they shall judge to be for the good and welfare of the commonwealth,
and of the subjects of the same."
"The police power of the State, so far, has not received a full and complete definition. It may be said,
however, to be the right of the State, or state functionary, to prescribe regulations for the good order,
peace, health, protection, comfort, convenience and morals of the community, which do not ... violate
any of the provisions of the organic law."
"It [the police power] has for its object the improvement of social and economic conditioned affecting
the community at large and collectively with a view to bring about "he greatest good of the greatest
number."Courts have consistently and wisely declined to set any fixed limitations upon subjects calling
for the exercise of this power. It is elastic and is exercised from time to time as varying social conditions
demand correction."
"It may be said in a general way that the police power extends to all the great public needs. It may be
put forth in aid of what is sanctioned by usage, or held by the prevailing morality or strong and
preponderant opinion to be greatly and immediately necessary to the public welfare."
"It is much easier to perceive and realize the existence and sources of this police power than to mark its
boundaries, or to prescribe limits to its exercise."

ISSUE: Whether or not procedures for dismissal were observed? (illegal dismissal)
HELD: No. Private respondent did not follow the notice requirements and instead argued that sending
notices to the last known addresses would have been useless because they did not reside there
anymore. Unfortunately for the private respondent, this is not a valid excuse because the law mandates
the twin notice requirements to the employees last known address. Thus, it should be held liable for
non-compliance with the procedural requirements of due process.
When the dismissal is for a just cause, the lack of statutory due process should not nullify the dismissal,
or render it illegal, or ineffectual. However, the employer should indemnify the employee for the violation
of his statutory rights.
9. Substantive Due Process
US v. Toribio 15 Phil. 85 Allan R. Taga-oc
Facts: Sometime in the 1900s, Toribio applied for a license to have his carabao be slaughtered. His
request was denied because his carabao is found not to be unfit for work. He nevertheless slaughtered
his carabao without the necessary license. He was eventually sued and was sentenced by the trial
court. His counsel in one way or the other argued that the law mandating that one should acquire a
permit to slaughter his carabao is not a valid exercise of police power.

23

Held: Due process; Standards of legal infirmity.There is no controlling and precise definition of due
process. It furnishes though a standard to which governmental action should conform in order
that deprivation of life, liberty or property, in each appropriate case, be valid. The standard of
due process which must exist both as a procedural and as substantive requisite to free the
challenged ordinance, or any governmental action for that matter, from imputation of legal
infirmity, is responsiveness to the supremacy of reason. obedience to the dictates of justice. It
would be an affront to reason to stigmatize an ordinance enacted precisely to meet what a
municipal lawmaking body considers an evil of rather serious pro portions as an arbitrary and
capricious exercise of authority. What should be deemed unreasonable and what would
amount to an abdication of the power to govern is inaction in the face of an admitted
deterioration of the state of public morals.
In favor of the operators. Right to property and to earn a living.

People v. Fajardo 104 Phil. 443 Guisadio


Facts: The municipal council of Baao, Camarines Sur stating among others that construction of a
building, which will destroy the view of the plaza, shall not be allowed and therefore be
destroyed at the expense of the owner, enacted an ordinance. Herein appellant filed a written
request with the incumbent municipal mayor for a permit to construct a building adjacent to
their gasoline station on a parcel of land registered in Fajardo's name, located along the
national highway and separated from the public plaza by a creek. The request was denied,
for the reason among others that the proposed building would destroy the view or beauty of
the public plaza. Defendants reiterated their request for a building permit, but again the
mayor turned down the request. Whereupon, appellants proceeded with the construction of
the building without a permit, because they needed a place of residence very badly, their
former house having been destroyed by a typhoon and hitherto they had been living on
leased property. Thereafter, defendants were charged in violation of the ordinance and
subsequently convicted.

Ynot v. Intermediate Court of Appeals 148 SCRA 659 Falcatan

Facts: Petitioner Transported six carabaos in a pump boat when they were confiscated by the station
commander of Nuevo, Iloilo, for violation of EO no. 626. The petitioner sued for recovery, and the RTC
issued a writ of replevin(recovery) upon his filing of a supersedeas bond of P 12, 000. Court sustained
confiscation of the carabaos and, and, since they could no longer be produced, ordered the
confiscation of the bond. Court declined the constitutionality of EO 626, as raised by petitioner for lack
of authority. Petitioner appealed to IAC, which upheld decision of the lower court. .

Issue: Whether or Not the ordinance is a valid exercise of police power?


Held: No. It is not a valid exercise of police power. The ordinance is unreasonable and
oppressive, in that it operates to permanently deprive appellants of the right to use their own
property; hence, it oversteps the bounds of police power, and amounts to a taking of
appellants property without just compensation. We do not overlook that the modern tendency
is to regard the beautification of neighborhoods as conducive to the comfort and happiness of
residents.

Issue: Whether or not executive order no. 626 is constitutional and within due process?
Held: The court finds that the challenged measures is an invalid exercise of the police power
because the method employed to conserve the carabaos is not reasonably necessary to the purpose of
the law and, worse is unduly oppressive. Due process is violated because the owner of the property
confiscated is denied the right to be heard in his defense and is immediately condemned and punished.
The strength of democracy lies not in the rights it guarantees but in the courage of the people to invoke
them whenever they are ignored or violated. Rights are but weapons on the wall if, like expensive
tapestry, all they do is embellish and impress. We agree with the respondent court, that the police
station commander who confiscated the carabaos is not liable in damages for enforcing the executive
order.
Wherefore Executive order No. 626 is hereby declared unconstitutional. Decision of Court of Appeals is
reversed. The supersedeas bond is cancelled and the amount thereof is ordered restored to the
petitioner.

As the case now stands, every structure that may be erected on appellants' land, regardless
of its own beauty, stands condemned under the ordinance in question, because it would
interfere with the view of the public plaza from the highway. The appellants would, in effect,
be constrained to let their land remain idle and unused for the obvious purpose for which it is
best suited, being urban in character. To legally achieve that result, the municipality must give
appellants just compensation and an opportunity to be heard.

Ermita-Malate Hotel & Operator vs. City of Manila 20 SCRA 849 Ottong
Agustin v. Edu, 88 SCRA 195 Lagbas
Facts: Petitioners assailed the constitutionality of Manila Ordinance No. 4760 regulating the operation
of hotels, motels and lodging houses on the ground that it is unreasonable and hence violative to the
due process clause, wherein it requires establishments to provide guest registration forms on the lobby
open for public view at all times.

Facts: Agustin is the owner of a Volkswagen Beetle Car. He is assailing the validity of Letter of
Instruction No 229, which requires all motor vehicles to have early warning devices particularly to equip
them with a pair of reflectorized triangular early warning devices. Agustin is arguing that this order is

Respondent City Mayor contends that the challenged ordinance was a valid and proper exercise of
police power measure for the proper purpose of curbing immorality. An explanatory note for the
challenged ordinance made mention of the alarming increase in the rate of prostitution, adultery and
fornication in Manila traceable in great part to the existence of motels and the like.

unconstitutional, harsh, cruel, and unconscionable to the motoring public. Cars are already equipped
with blinking lights which is already enough to provide warning to other motorists. And that the mandate
to compel motorists to buy a set of reflectorized early warning devices is redundant and would only
make manufacturers and dealers instant millionaires.

Issue: Whether or not Ordinance No. 4760 is violative of the due process clause.

24

ISSUE: Whether or not the said is EO is valid.

public policy or is unreasonable, oppressive, partial, discriminating or in derogation of a common right.


Ordinance No. 649 clearly invades the property rights of the petitioners for even if we could assume
that, on its face, the interference was reasonable, from the foregoing considerations, I has been fully
shown that it is an unwarranted and unlawful curtailment of the property and personal rights of citizens.
For being unreasonable and an undue restraint of trade, it cannot, under the guise of exercising police
power, be upheld as valid.
Wherefore, the decision of the Trial Court in Special Civil Case No. 237 is hereby Reversed and Set
Aside and a new judgement is hereby rendered declaring Ordinance No. 640 unconstitutional and,
therefore, null and void. This decision is immediately executory. So Ordered.

HELD: Such early warning device requirement is not an expensive redundancy, nor oppressive,
for car owners whose cars are already equipped with 1) blinking-lights in the fore and aft of said motor
vehicles, 2) battery-powered blinking lights inside motor vehicles, 3) built-in reflectorized tapes on
front and rear bumpers of motor vehicles, or 4) well-lighted two (2) petroleum lamps (the Kinke) . . .
because: Being universal among the signatory countries to the said 1968 Vienna Conventions,
and visible even under adverse conditions at a distance of at least 400 meters, any motorist from this

National Development Co. and New Agrix v. Phil. Vet. Bank 192 SCRA 257 amilbahar

country or from any part of the world, who sees a reflectorized rectangular early warning device

Facts: This case involves the Constitutionality of a presidential decree which, like all other issuances of
President Marcos during his regime, was at that time regarded as sacrosanct. The particular enactment
in question is Pres. Decree No. 1717, which ordered the rehabilitation of the Agrix Group of Companies
to be administered mainly by the National Development Company.
The Agrix Marketing, Inc. (AGRIX) had executed in favor of private respondent Philippine
Veterance Bank a real estate mortgage dated July 7, 1978, over three(parcels) of land situated in Los
Banos, Laguna. During the existence of the mortgage the AGRIX went Bankrupt.
The private respondent filed a claim with the AGRIX Claims Committee for the payment of its
loan credit. The New Agrix, Inc. and the National Development Company, petitioner herein, invoking
Section 4(1) of the decree, filed a petition with the Regional Trial Court of Calamba, Laguna, for the
mortgage lien in favor of the private respondent. For its part, the private respondents took steps to extra
judicially foreclose the Mortgage, prompting the petitioners to file a second case with the same court to
stop the foreclosure. The two cases were consolidated.
The Trial Court rendered the impugned decision. Judge Francisco Ma. Guerrero annulled not
only the challenged provision, Section 4(1), but the entire Pres. Decree No. 1717 on the grounds that
(1) the presidential exercise of legislative powers was a violation of the principle of separation of power.
(2) The law impaired the obligation of contracts. And (3) the decree violated the equal protection clause.
The motion for reconsideration of this decision having been denied.
On August 30, 1988, the court granted the petitioners prayer for a temporary restraining order
and instructed the respondent to cease and desist from conducting a public auction sale of the lands in
question.
The petitioner contend that the private respondent is now and automatic justification for every
act of the government depriving a person of his life, liberty, property.

installed on the roads, highways or expressways, will conclude, without thinking, that somewhere along
the travelled portion of that road, highway, or expressway, there is a motor vehicle which is stationary,
stalled or disabled which obstructs or endangers passing traffic. On the other hand, a motorist who
sees any of the aforementioned other built-in warning devices or the petroleum lamps will not
immediately get adequate advance warning because he will still think what that blinking light is all
about. Is it an emergency vehicle? Is it a law enforcement car? Is it an ambulance? Such confusion or
uncertainty in the mind of the motorist will thus increase, rather than decrease, the danger of collision.

Balacuit v. CFI 163 SCRA 182 Abdurajak


FACTS: Petitioners are Carlos Balacuit, Lamberto Tan, and Sergio Yu Carcel, managers of the Mays
and Dalisay Theaters, the Crown Theater, and the Diamond Theater, respectively. Aggrieved by the
effects of Ordinance No. 640, they filed a complaint in Court of First Instance of Agusan Del Norte and
Butuan City praying, that the subject ordinance be declared unconstitutional and, therefore, void and
unenforceable.
Upon motion of the petitioners, a temporary restraining order was issued on July 14, 1969 by the court
a quo enjoining the respondent City of Butuan and its officials from enforcing Ordinance No. 640. On
july 29, 1969, respondents filed their answer sustaining the validity of the ordinance. On January 30,
1973, the litigants filed their stipulation of facts. On June 4, 1973, the respondent rendered its decision.
Petitioners filed their motion for reconsideration of the decision of the court a quo which was denied in a
resolution of the said court dated NOVEMBER 10, 1973. Petitioners attack the validity and
constitutionality of Ordinance No. 640 on the grounds that it is ultra vires and an invalid exercise of
police power. Petitioners contend that Ordinance No. 640 is not within the power of the Municipal
Board to enact as provided for in Section 15(n) of Republic Act No. 523, the Charter of the City of
Butuan. Respondent City of Butuan, on the other hand, attempts to justify the enactment of the
ordinance by invoking the general welfare clause embodied in Section 15 (nn) of the cited law.

Issue: Whether or Not the right to property in all mortgages, liens, interests, penalties and charges
owing to the creditors of AGRIX is arbitrary destroyed?
Ruling: A mortgage lien is a property right derived from contract and so comes under the protection of
the Bill of Rights. So do interest on loan, as well as penalties and charges, which are also vested rights
once they accrue.
The court reaffirms and applies that ruling in the case at bar. Is that Pres.Decree No.1717 is an invalid
exercise of the police power, not being in conformity with the traditional requirements of a lawful subject
and a lawful method. The extinction of the mortgage and other liens and of the interest and other
charges pertaining to the legitimate creditors of AGRIX constitutes taking without due process of law,
and this is compounded by the reduction of the secured creditors to the category of unsecured creditors
in violation of the equal protection clause.
With the above pronouncements, we feel there is no more need to rule in the authority of President
Marcos to promulgate Pres.Decree No. 1717 under Amendment No.6 of the 1973 Constitution. Even if
he had such authority, the decree must fall just the same because of its violation of the Bill of rights.

ISSUE: Whether the Ordinance No. 640 is valid and constitutional.


HELD: Respondent City of Butuan argues that the presumption is always in favour of the validity of the
ordinance. This maybe the rule but it has already been held that although the presumption is always in
favour of the validity or reasonableness of the ordinance, such presumption must nevertheless be set
aside when the invalidity or unreasonableness appears on the face of the ordinance itself or is
established by proper evidence. The exercise of the police power by the local government is valid
unless it contravenes the fundamental law of the land, ran act of the legislature, or unless it is against

25

building belonging to Pryce


Properties Corporation Inc. for its casino.
On December 7, 1992, Sangguniang Panlungsod of CDO enacted ordinance 3353,
prohibiting the issuance of business permit and cancelling existing business permit to any
establishment for the using and allowing to be used its premises or portion thereof for the
operation
of
a
casino.
On January 4, 1993, it enacted Ordinance 3375-93, prohibiting the operation of casino and
providing
penalty
for
violation
therefore.
Pryce assailed the ordinances before the CA, where it was joined by PAGCOR as intervenor.
The Court found the ordinances invalid and issued the writ prayed for to prohibit their
enforcement. CDO City and its mayor filed a petition for review under Rules of Court with the
Supreme Court.

WHEREFORE, the petition is DISMISSED. Pres.Decree No. 17171 is declared UNCONSTITUTIONAL.


The temporary restraining order dated August 30, 1988, is lifted. Cost against the petitioner.

Maranaw Hotel v. NLRC 238 SCRA 190 florendo

FACTS: On 16 June 1990, Private Respondent Gina G. Castro was hired on a probationary basis for
six months as a guest relations officer of the Century Park Sheraton Hotel, owned by petitioner. On 10
November 1990, she was dismissed on the ground of failure to meet the standards set forth in her
probationary employment contract. She then filed on 13 November 1990 with the NLRC a complaint for
illegal dismissal with reinstatement back wages, and damages against the hotel and its former general
manager, Pieter Grieder.
On 23 December 1991, the Labor Arbiter rendered a decision in favor of the private
respondent.
On 7 February 1992, within the 10-day reglementary period, petitioner filed an appeal to the
NLRC alleging therein that the labor arbiter committed abuse of discretion and serious error in his
findings of fact and conclusions of law.

Issue: WON the Sangguniang Panlungsod can prohibit the establishment of casino operated by
PAGCOR through an ordinance or resolution.
Held: No. Gambling is not illegal per se. While it is generally considered inimical to the interests of the
people, there is nothing in the Constitution categorically proscribing or penalizing gambling or,
for that matter, even mentioning it at all. In the exercise of its own discretion, the Congress
may prohibit gambling altogether or allow it without limitation or it may prohibit some forms of
gambling and allow others for whatever reasons it may consider sufficient.
Under Sec. 458 of the Local Government Code, local government units are authorized to
prevent or suppress, among others, gambling and other prohibited games of chance.
Ordinances should not contravene a statue as municipal governments are only agents of the
national government. Local councils exercise only delegated powers conferred on them by
Congress as the national lawmaking body. The delegate cannot be superior to the principal or
exercise powers higher than those of the latter.
The tests of a valid ordinance are well established. A long line of decisions has held that to be
valid, an ordinance must conform to the following substantive requirements:
1) It must not contravene the constitution or any statute.
2) It must not be unfair or oppressive.
3) It must not be partial or discriminatory.
4) It must not prohibit but may regulate trade.
5) It must be general and consistent with public policy.
6) It must not be unreasonable.

ISSUE: Whether or not the NLRC acted with grave abuse of discretion in holding that the private
respondent should be considered as reinstated in the payroll from the filing of the motion for execution
on 13 March 1992 until the promulgation of its resolution and as a necessary consequence ordering the
petitioner to pay the private respondent her salaries corresponding to the period from 13 March 1992
up to 25 March 1993 when its resolution was promulgated.
HELD: Yes. petition is hereby GRANTED
We agree with the petitioner that the NLRC acted with grave abuse of discretion. The resolution of the
issue is found in the third paragraph of Article 223 of the Labor Code. It must be stressed, however, that
although the reinstatement aspect of the decision is immediately executory, it does not follow that it is
self-executory. There must be a writ of execution which may be issued motu proprio or on motion of an
interested party as provided by Article 224 of the Labor Code and Second Paragraph of Section 1, Rule
VIII of the New Rules of Procedure of the NLRC. In the instant case, the Labor Arbiter neither issued
motu proprio a writ of execution to enforce the reinstatement aspect of his decision nor acted on the
private respondents motion for execution filed on 13 March 1992. Since in the instant case no occasion
arose for the petitioner to exercise its option under Article 223 of the Labor Code with respect to the
reinstatement aspect of the decision of the Labor Arbiter, the NLRC acted with grave abuse of
discretion when it ordered that the private respondent should be considered reinstated in the payroll
from the filing of her motion for execution until the promulgation of its resolution on 25 March 1993.
Petition is hereby GRANTED.

Bennis v. Michigan No. 94-8729 March 4, 1996 Isnani


Substantive Due Process (abatement-end) Fifth amendment
Facts: Petitioner was a joint owner, with her husband, of an automobile in which her husband engaged
in sexual activity with a prostitute. A Michigan court ordered the automobile forfeited as a public
nuisance, with no offset for her interest, notwithstanding her lack of knowledge of her husband's activity.
Detroit police arrested John Bennis after observing him engaged in a sexual act with a prostitute in the
automobile while it was parked on a Detroit city street. Bennis was convicted of gross indecency. The
State then sued both Bennis and his wife, petitioner Tina B. Bennis, to have the car declared a public
nuisance and abated.

Magtajas v. Pryce Properties 234 SCRA 255 Bagasina

Facts: The Philippine Amusement and Gaming Corporation (PAGCOR) is a corporation created directly
by P.D. 1869 to help centralize and regulate all games of chance, including casinos on land
and sea within the territorial jurisdiction of the Philippines. In Basco v. Philippine Amusements
and Gaming Corporation, this Court sustained the constitutionality of the decree and even
cited the benefits of the entity to the national economy as the third highest revenue-earner in
the
government.
PAGCOR decided to expand its operations to Cagayan de Oro City by leasing a portion of a

Issues: Whether or not abatement scheme has deprived petitioner of her interest in the forfeited car
without due process.

26

Held: The gravamen of petitioner's due process claim is not that she was denied notice or an
opportunity to contest the abatement of her car; she was accorded both. Rather, she claims she was
entitled to contest the abatement by showing she did not know her husband would use it to violate
Michigan's indecency law. But a long and unbroken line of cases holds that an owner's interest in
property may be forfeited by reason of the use to which the property is put even though the owner did
not know that it was to be put to such use. "It has long been settled that statutory forfeitures of
property entrusted by the innocent owner to another who uses it in violation of the revenue
laws of the United States is not a violation of the due process clause of the Fifth Amendment.

illegal and "in gross violation of the constitutional right... to life liberty and property." Said Federation
consequently prayed for the issuance of a writ of preliminary injunction against the aforestated orders.
Issue:Whether or not the police power can abridge the right of the performing workers?
Ruling: In the case at bar, the challenged Department Order clearly applies to all performing artists and
entertainers destined for jobs abroad. These orders, we stressed hereinfore, further the Constitutional
mandate requiring government to protect our workforce, particularly those who may be prone to abuse
and exploitation as they are beyond the physical reach of government regulatory agencies. The tragic
incidents must somehow stop, but short of absolutely curtailing the right of these performers and
entertainers to work abroad, the assailed measures enable our government to assume a measure of
control.
WHEREFORE, finding no reversible error in the decision sought to be reviewed, petition is hereby
DENIED.
SO ORDERED.

Cruzan v. Dir. Missouri No. 88-1503 June 25 1990 Ynawat

FACTS: Nancy Beth Cruzan was left in a persistent vegetative state after sustaining
serious injuries in an automobile accident. She was supported with an artificial life support system. After
it was apparent that she had no hopes of recovery, her parents asked the hospital
authorities to withdraw the life support system. The hospital authorities wouldnt heed such request if
there is no court approval. The parents sought and received authorization from the court. The court
found that a person in Nancys condition had a fundamental right under the State and Federal
Constitutions to refuse and direct the withdrawal of death prolonging procedures. The court also
found that Nancys expressed thoughts at age 25 in somewhat a serious conversation with her
housemate friend that if sick or injured she would not wish to continue her life unless she could live at
least halfway normally suggests that given her present condition she would not wish to continue on with
her nutrition and hydration. This decision was reversed by the state Supreme Court.

Corona v. United Harbor 283 SCRA 31 Hassan

Facts: the PPA was created on July 11, 1974, by virtue of Presidential Decree No. 505. On December
23, 1975, Presidential Decree No. 857 was issued revising the PPAs charter. Pursuant to its power of
control, regulation, and supervision of pilots and the pilotage profession, the PPA promulgated PPA-AO03-85 on March 21, 1985, which embodied the Rules and Regulations Governing Pilotage Services,
the Conduct of Pilots and Pilotage Fees in Philippine Ports. These rules mandate, inter alia, that
aspiring pilots must be holders of pilot licenses and must train as probationary pilots in outports for
three months and in the Port of Manila for four months. It is only after they have achieved satisfactory
performance that they are given permanent and regular appointments by the PPA itself.

ISSUE: Whether or not the States refusal to terminate the life support system of Cruzan did violate her
right to due process and liberty interest rights to refuse medical treatment.

Issue: Is pilotage a property right? Is A.O. 04-92 violative of due process?

HELD:The US Supreme Court in a 6-3 ruling found that a person did have a liberty interest under the
due process clause of the 14th amendment to refuse medical treatment, provided that they are
competent and there was clear and convincing evidence that the person didnt want artificial support
to keep them alive. Petitioner Nancy Cruzan was rendered incompetent due to her injuries from
an automobile accident. Without this evidence, a state obligation to uphold human life overrules the
wishes of patient or parents. The court found that the statements of Nancy Cruzan under certain
conditions were unreliable for the purpose of determining her intent and thus insufficient to support coguardians claim to exercise substituted judgment on Nancys behalf. It rejected the argument that
Cruzans parents were entitled to order the termination of the life support system, concluding that no
person can assume that choice of an incompetent in the absence of formalities required under
Missouris Living Will Statutes or the clear and convincing, inherently reliable evidence absent here. In
this case, the Cruzans had no clear or convincing evidence like a living will to terminate the life
support system.

Ruling: Yes, The court a quo pointed out that the Bureau of Customs, the precursor of the PPA,
recognized
pilotage
as
a
profession
and,
therefore,
a
property
right
No, PPA-AO 04-92 does not constitute a wrongful interference with, let alone a wrongful
deprivation of, the property rights of those affected thereby. As may be noted, the issuance
aims no more than to improve pilotage services by limiting the appointment to harbor pilot
positions to one year, subject to renewal or cancellation after a rigid evaluation of the
appointees
performance.
PPA-AO 04-92 does not forbid, but merely regulates, the exercise by harbor pilots of their
profession
in
PPAs
jurisdictional
area

Kelly v. Johnson 425 US 238 Ariel Larete

FACTS:A county regulation limiting the length of county policemen's hair held not to violate any
right guaranteed respondent policeman by the Fourteenth Amendment Respondent sought the
protection of the Fourteenth Amendment, not as an ordinary citizen, but as a law enforcement
employee of the county, a subdivision of the State, and this distinction is one of considerable
significance, since a State has wider latitude and notably different interests in imposing
restrictive regulations on its employees than it does in regulating the citizenry at large.

JMM Promotion and Management Inc. v. CA 260 SCRA 319 Cabanlong

Facts
In Civil Case No. 95-72750, the Federation of Entertainment Talent Managers of the
Philippines (FETMOP), on January 27, 1995 filed a class suit assailing these department orders,
principally contending that said orders 1) violated the constitutional right to travel; 2) abridged existing
contracts for employment; and 3) deprived individual artists of their licenses without due process of law.
FETMOP, likewise, averred that the issuance of the Artist Record Book (ARB) was discriminatory and

27

Choice of organization, dress, and equipment for law enforcement personnel is entitled to the
same sort of presumption of legislative validity as are state choices to promote other aims
within the cognizance of the State's police power. Thus, the question is not whether the State
can "establish" a "genuine public need" for the specific regulation, but whether respondent can
demonstrate that there is no rational connection between the regulation, based as it is on the
county's method of organizing its police force, and the promotion of safety of persons and
property.
There is a regulation requiring police officers to keep hair at a certain length.

property right, nor does it create a vested right. In a more emphatic pronouncement, we held in Oposa
vs. Factoran, Jr. that:

Needless to say, all licenses may thus be revoked or rescinded by executive action. It is not a contract,
property or a property right protected by the due process clause of the Constitution.
xxx
In our jurisdiction, the PNP Chief is granted broad discretion in the issuance of PTCFOR. This is
evident from the tenor of the Implementing Rules and Regulations of P.D. No. 1866 which state that
the Chief of Constabulary may, in meritorious cases as determined by him and under such conditions
as he may impose, authorize lawful holders of firearms to carry them outside of residence. Following
the American doctrine, it is indeed logical to say that a PTCFOR does not constitute a property right
protected
under
our
Constitution.

ISSUE:Whether a state or local government's choice to have its police uniformed reflects a
desire to make police officers readily recognizable to the public or to foster the esprit de corps
that similarity of garb and appearance may inculcate within the police force itself, the
justification for the hair style regulation is sufficiently rational to defeat respondent's claim
based on the liberty guarantee of the Fourteenth Amendment

Consequently, a PTCFOR, just like ordinary licenses in other regulated fields, may be revoked any
time. It does not confer an absolute right, but only a personal privilege to be exercised under existing
restrictions, and such as may thereafter be reasonably imposed. A licensee takes his license subject to
such conditions as the Legislature sees fit to impose, and one of the statutory conditions of this license
is that it might be revoked by the selectmen at their pleasure. Such a license is not a contract, and a
revocation of it does not deprive the defendant of any property, immunity, or privilege within the
meaning of these words in the Declaration of Rights. The US Supreme Court, in Doyle vs. Continental
Ins. Co, held: The correlative power to revoke or recall permission is a necessary consequence of the
main power. A mere license by the State is always revocable.

HELD:No, there is no protected liberty. Unlike Roe and others, there is no substantial claim of
infringement on the individuals freedom of choice with respect to certain basic matters of
procreation, marriage, and family life.
Reasoning
Similarity in appearance of police officers is a desirable trait and serves as a rational reason for
the law.
Marshall Dissent:
By taking over appearance, the state forces him to sacrifice elements of his identity. This liberty
of appearance is on par with other protected interests like privacy, self-identity, autonomy and
personal integrity. Further, there is no rational relationship between the ends means (how does
appearance have to do with esprit de corps?)

Chavez v. Romulo 431 SCRA 534 (2004) Jimenez

A mere license is always revocable

10. Equal Protection of the Law

FACTS:

FACTS:
Theaccused Cayat wasconvicted undersections2and 3ofAct 1639 bya peace of court in Baguio City and was
sentenced to payfiftypesosorsufferSubsidiaryimprisonment in case of insolvency.Under theActin question, the possession of intoxicating
liquor otherthan native winesand liquorsbyanymemberofnon-Christian tribesin the Philippinesisconsidered unlawfuland punishable under
the same act. Sometime onJanuary 1967,the accused was foundunlawfully,ilegallyand wilfully received,acquire and havein his
possession a one bottle ofA-1-1 gin, an intoxicating liquor notconsidered asa native wine orliquor andisilegalunder the contemplation of
Act 1639.He admitted allofthe accusation against himhoweverpleaded notguilty during histrial.He questions the constitutionalityoftheActon
the ground thatit is discriminatory anddenies equal protection ofa law violative of adue processand animproper exercise of police power.

This case is about the ban on the carrying of firearms outside of residence in order to deter the rising
crime rates. Petitioner questions the ban as a violation of his right to property.
ISSUE:

HELD:

Whether or not the revocation of permit to carry firearms is unconstitutional

ISSUE:Whetherornotthe saidAct 1639 violatesthe equalprotection clause.

Whether or not the right to carry firearms is a vested property right

Petitioner

cannot

find

solace

to

the

above-quoted

Constitutional

People v. Cayat 68 PHIL. 12, 18 Vanessa Delatado

HELD:No. In the colonial history of the Philippines, colonials always believed that they have a great responsibility to civilize these less civilized
people of their colony, such That they impose upon themselves the duty to free them from obscurity of ignorance. They provided different
measures to bring these people to civilization. One of which is the implementation of the Act in question, implemented to secure these people
the blessings of peace and harmony, to facilitate and not to spoil their march towards civilization. Under the established principle of constitutional
law, the guaranty of equal protection is not violated by a legislation based on reasonable classification. Under the reasonable classification test,
the following requisitesshould notbe wanting.

provision.

In evaluating a due process claim, the first and foremost consideration must be whether life, liberty or
property interest exists. The bulk of jurisprudence is that a license authorizing a person to enjoy a
certain privilege is neither a property nor property right. In Tan vs. The Director of Forestry, we ruled that
a license is merely a permit or privilege to do what otherwise would be unlawful, and is not a contract
between the authority granting it and the person to whom it is granted; neither is it property or a

1 Must rest on substantial distinctions


2.Mustbe germane to the purposes of the law.
3.Mustnotbe limited to existing conditions only.

28

4.Must apply equallyto all members of the same class.

misled the legislators or the segment of the population affected. Lastly, it cannot be said to be void for
supposed conflict with treaty obligations because no treaty has actually been entered into on the
subject and the police power may not be curtailed or surrendered by any treaty or any other
conventional agreement.

Act No. 1639 satisfies these requirements. The classification rests on real or substantial, not merely
imaginary or whimsical, distinctions. It is not based upon accident of birth or parentage. The law, then,
does not seek to mark the non-Christian tribes as an inferior or less capable race. On the contrary, all
measures thus far adopted in the promotion of the public policy towards them rest upon a recognition of
their inherent right to equality in the enjoyment of those privileges now enjoyed by their Christian
brothers. But as there can be no true equality before the law, if there is, in fact, no equality in education,
the government has endeavored, by appropriate measures, to raise their culture and civilization and
secure for them the benefits of their progress, with the ultimate end in view of placing them with their
Christian brothers on the basis of true equality

Villegas v. Hiu Chiong Tsai Pao Ho 86 SCRA 270 Fernandez

FACTS An ordinance of the City of Manila prohibited the employment of aliens in any occupation or to
engage or participate in any position or occupation or business enumerated, whether permanent,
temporary or casual, without first securing an employment permit from the Mayor of Manila and paying
the permit fee of P50.00 except persons employed in the diplomatic or consular missions of foreign
countries, or in the technical assistance programs of both the Philippine Government and any foreign
government, and those working in their respective households, and members of religious orders or
congregations, sect or denomination, who are not paid monetarily or in kind. Respondent, an alien,
employed in Manila, brought suit and obtained judgment from the CFI declaring the ordinance null and
void.

Ichong v. Hernandez 101 PHIL. 1155 Tubo

Facts: Lao Ichong is an alien engaged in retail business in the Philippines. Along with other Chinese
alien doing business, they enjoy a monopoly. In 1954, the Congress passed RA No. 1180 where it
nationalizes the retail trade business. The main provisions of the Act are: (1) a prohibition against
persons, not citizens of the Philippines, and against associations, partnerships, or corporations the
capital of which are not wholly owned by citizens of the Philippines, from engaging directly or indirectly
in the retail trade.. Petitioner and on behalf of other alien residents corporations and partnerships
adversely affected by the provisions of RA No. 1180 brought this action sighting the said Act is
unconstitutional and enjoining the Secretary of Finance and all other persons acting under him,
particularly city and municipal treasurers, from enforcing its provisions. Petitioner contends that: (1) it
denies to alien residents the equal protection of the laws and deprives of their liberty and property
without due process of law ; (2) the Act violates international and treaty obligations of the Republic of
the Philippines. In answer, the Solicitor-General and the Fiscal of the City of Manila contend that: (1) the
Act was passed in the valid exercise of the police power of the State, which exercise is authorized in
the Constitution in the interest of national economic survival and no treaty or international obligations
are infringed.

HELD The contention that Ordinance No. 6537 is not a purely tax or revenue measure because its
principal purpose is regulatory in nature has no merit. While it is true that the first part which requires
that the alien shall secure an employment permit from the Mayor involves the exercise of discretion and
judgment in the processing and approval or disapproval of applications for employment permits and
therefore is regulatory in character the second part which requires the payment of P50.00 as
employee's fee is not regulatory but a revenue measure. There is no logic or justification in exacting
P50.00 from aliens who have been cleared for employment. It is obvious that the purpose of the
ordinance is to raise money under the guise of regulation.
Ordinance No. 6537 does not lay down any criterion or standard to guide the Mayor in the exercise of
his discretion. It has been held that where an ordinance of a municipality fails to state any policy or to
set up any standard to guide or limit the mayor's action, expresses no purpose to be attained by
requiring a permit, enumerates no conditions for its grant or refusal, and entirely lacks standard, thus
conferring upon the Mayor arbitrary and unrestricted power to grant or deny the issuance of building
permits, such ordinance is invalid, being an undefined and unlimited delegation of power to allow or
prevent an activity per se lawful.The ordinance in question violates the due process of law and equal
protection rule of the Constitution.

Issue: Whether or not the enactment falls within the scope of police power of the State?

Requiring a person before he can be employed to get a permit from the City Mayor of Manila who may
withhold or refuse it at will is tantamount to denying him the basic right of the people in the Philippines
to engage in a means of livelihood. While it is true that the Philippines as a State is not obliged to admit
aliens within its territory, once an alien is admitted, he cannot be deprived of life without due process of
law. This guarantee includes the means of livelihood. The shelter of protection under the due process
and equal protection clause is given to all persons, both aliens and citizens.

Decision:Yes, the law was enacted to remedy a real actual threat and danger to national economy
posed by alien dominance and control of the retail businesses and free citizens and country from such
dominance and control. It protects its own personality and insures its security and future. The law does
not violate the equal protection clause of the Constitution because sufficient grounds exist for the
distinction between alien and citizen in the exercise of the occupation regulated, nor the due process of
law clause, because the law is prospective in operation and recognizes the privilege of aliens already
engaged in the occupation and reasonably protects their privilege. The equal protection clause is not
infringed by legislation which applies only to those persons falling within a specified class, if it applies
alike to all persons within such class, and reasonable grounds exists for making a distinction between
those who fall within such class and those who do not. The wisdom and efficacy of the law to carry out
its objectives appear to be plainly evident as a matter of fact it seems not only appropriate but
actually necessary and that in any case such matter falls within the prerogative of the Legislature,
with whose power and discretion the Judicial department of the Government may not interfere. The
provisions of the law are clearly embraced in the title, and this suffers from no duplicity and has not

The trial court did not commit the errors assigned.


WHEREFORE, the decision appealed from is hereby affirmed, without pronouncement as to costs.
SO ORDERED.

29

(9) Constitutional Law @ 162 -- Equal Protection of Laws, Class Legislation and Uniformity of Operation
-- Bases of Classification. -- Cal. Const., art. XX, @ 18, does not admit of exceptions based on popular
notions of what is a proper, fitting, or moral occupation for persons of either sex.

Dumlao v. COMELEC 96 SCRA 392 San Luis

Facts: Petitioner questions the constitutionality of section 4 of Batas Pambansa Blg. 52 as


discriminatory and contrary to the equal protection and due process guarantees of the Constitution.

(10) Constitutional Law @ 162 -- Equal Protection of Laws, Class Legislation and Uniformity of
Operation -- Bases of Classification. -- Cal. Const., art. XX, @ 18, in no way prevents the Legislature
from dealing effectively with the evils [***4] and dangers inherent in selling and serving alcoholic
beverages, but merely precludes resort to legislation against women, rather than against the particular
evil sought to be curbed. (Overruling Ex Parte Hayes, 98 Cal. 555 [33 P. 337].)

Section 4 provided that any retired municipal or provincial city official that already received retirement
benefits and is 65 years of age shall not be qualified to run for the same local elective office from which
he has retired.
Issue: Whether or Not Sec. 4 of BP.52 is unconstitutional being contrary to the equal protection and
due process rights.

Facts:
A Michigan statute required that all bartenders hold licenses in cities with populations greater than

Held: No. The guarantee of equal protection is subject to rational classification based on reasonable
and real differentiations. In the present case, employees 65 years of age have been classified
differently from younger employees. The former are subject to compulsory retirement while the latter
are not.

50,000, but the statute also stated that a woman could not be issued a license unless she was "the wife
or daughter of the male owner" of a liquor establishment. Two female bartenders challenged the law,
requesting an injunction against its enforcement, on the ground that it violated the Equal Protection

Retirement is not a reasonable disqualification for elective local officials because there can be retirees
who are even younger and a 65year old retiree could be as good as a 65 year old official who is not a
retiree. But there is reason to disqualify a 65 year old elective official who is trying to run for
office because there is the need for new blood to assume relevance. When an official has
retired he has already declared himself tired and unavailable for the same government work.

Clause of the Fourteenth Amendment. The United States District Court for the Eastern District of
Michigan rejected the bartenders' claim.
Issue:
Whether the Michigan statute denying female bartenders access to licenses violate the Equal

WHEREFORE, the first paragraph of section 4 of Batas pambansa Bilang 52 is hereby declared valid.

Protection Clause of the Fourteenth Amendment


Held:
The US Supreme Court held that the Constitution "does not preclude the States from drawing a sharp
line between the sexes" or "to reflect sociological insight, or shifting social standards, any more than it
requires them to keep abreast of the latest scientific standards." The Court found that the Michigan

Goesart v. Cleary - 335 US 464 Hussin

legislature, in enacting the statute, could have determined that allowing women to bartend could "give
rise to moral and social problems against which it may devise preventive measures." The Court is in no
Facts: Finally, it is contended that section 25656 violates the equal protection clause of the Fourteenth
Amendment to the United States Constitution and article I, sections 11 and 21, of the California
Constitution n13 in that it prohibits women from tending bar unless they or their husbands hold the
liquor license; but does not impose a comparable limitation on men.

position to "cross-examine either actually or argumentatively the mind of Michigan legislators.

Issue: Whether or not equal protection clause is violated?


Held: Equal Protection of Laws, Class Legislation and Uniformity of Operation -- Bases of
Classification. -- Pursuant to Cal. Const., art. XX, @ 18, sex alone may not be used to exclude a person
from a vocation, profession or business.

[See Cal.Jur.2d, Constitutional Law, @ 283.]

Ormoc Sugar Central v. Ormoc City Feb. 7, 1968 Aripin

Facts: On January 29, 1964, the Municipal Board of Ormoc city passed an ordinance which
provides:

(8) Constitutional Law @ 162 -- Equal Protection of Laws, Class Legislation and Uniformity of Operation
-- Bases of Classification. -- Cal. Const., art. XX, @ 18, constitutes a restraint upon the law-making
power of the state, and renders legislative enactments contrary to its provisions void.

"There shall be paid to the City Treasurer on any and all productions of centrifugal sugar milled at the
Ormoc Sugar Company, Incorporated, in Ormoc City, a municipal tax equivalent to one per centum
(1%) per export sale to the United States of America and other foreign countries."

30

(Payments for said tax were made, under protest, by Ormoc Sugar Company, Inc. on March 20,
1964 for P7, 087.50 and on April 20, 1964 for P5, 000, or a total of P12, 087.50.)

constitution in that it legalizes PAGCORconducted gambling, while most other


forms of gambling are outlawed, together with prostitution, drug trafficking and
other vices;

Ormoc Sugar Company filed a complaint against the city of Ormoc, alleging that the afore-stated
ordinance is unconstitutional for being violative of the equal protection clause (Sec. 1[1], Art. III,
Constitution) and the rule of uniformity of taxation (Sec. 22[1], Art. VI, Constitution).

Issue: Whether or not PD 1869 violates the Equal Protection Caluse of the Constitution.

Answering, the defendants asserted that the tax ordinance was within defendant city's power to enact
under the Local Autonomy Act and that the same did not violate the afore-cited constitutional limitations.
After pre-trial and submission of the case on memoranda, the Court of First Instance, on August 6,
1964, rendered a decision that upheld the constitutionality of the ordinance and declared the taxing
power of defendant chartered city broadened by the Local Autonomy Act to include all other forms of
taxes,
licenses
or
fees
not
excluded
in
its
charter.

Held:The Court finds no valid ground to sustain this contention. The petitioners posture
ignores the well-accepted meaning of the clause equal protection of the laws.
The clause does not preclude classification of individuals who may be accorded
different treatment under the law as long as the classification is not
unreasonable or arbitrary (Itchong v. Hernandez, 101 Phil. 1155). A law does not
have to operate in equal force on all persons or things to be conformable to
Article III, Section 1 of the Constitution (DECS v. San Diego, G.R. No. 89572,
December 21, 1989)

ISSUE:W/N the ordinance violates the equal protection clause and the uniformity of taxation.
RULING: YES The equal protection clause applies only to persons or things identically situated and
does not bar a reasonable classification of the subject of legislation, and a classification is reasonable
where (1) it is based on substantial distinctions which make real differences; (2) these are germane to
the purpose of the law; (3) the classification applies not only to present conditions but also to future
conditions which are substantially identical to those of the present; (4) the classification applies only to
those who belong to the same class.

Republic v. Sandiganbayan 230 SCRA 711 Maria Fatima Orbecido


FACTS: Before 1986, the Landoil Group of Companies spearheaded by then Congressman Jose de
Venecia, Jr., was able to obtain foreign loans syndicated by various banks aggregating approximately
one hundred twenty million US dollars ($120M). These foreign loans were guaranted by
PHILGUARANTEE, whose Board of Directors was then composed of private respondents, Rosendo D.
Bondoc, Ruben Ancheta, Jaime C. Laya, Placido Mapa Jr., Roberto Ongpin, and Cesar Zalamea.
Congressman de Venecias group of companies was unable to seasonably service these foreign loans
and this compelled PHILGUARANTEE to assume its obligation as guarantor.
Moreover, the EDSA revolution in February 1986 swept the Marcoses out of power.The creation of
PCGG lead the difficult task of recovering the illegal wealth of Marcoses, their family, subordinates and
close associates. The Marcoses and their cronies had to face a flurry of both and criminal cases, all
designed to recover the Republics wealth allegedly plundered by them while in power. The petitioner in
this case filed a reconvayance,reversion,accounting,restitution and damages against Jose de Venecia,
et al.

A perusal of the requisites instantly shows that the questioned ordinance does not meet them, for it
taxes only centrifugal sugar produced and exported by the Ormoc Sugar Company, Inc. and none other.
At the time of the taxing ordinance's enactment, Ormoc Sugar Company, Inc., it is true, was the only
sugar central in the city of Ormoc. Still, the classification, to be reasonable, should be in terms
applicable to future conditions as well. The taxing ordinance should not be singular and exclusive as to
exclude any subsequently established sugar central, of the same class as plaintiff, for the coverage of
the tax. As it is now, even if later a similar company is set up, it cannot be subject to the tax because the
ordinance expressly points only to Ormoc City Sugar Company, Inc. as the entity to be levied upon
Appellant, however, is not entitled to interest; on the refund because the taxes were not arbitrarily
collected (Collector of Internal Revenue v. Binalbagan). At the time of collection, the ordinance provided
a sufficient basis to preclude arbitrariness, the same being then presumed constitutional until declared
otherwise.

ISSUE: Whether or not the dismissal of the complaint against de Venecia, Bondoc and company is
compelled by the equal protection clause of the constitution.

Wherefore, the decision appealed from is hereby reversed, the challenged ordinance is declared
unconstitutional and the defendants-appellees are hereby ordered to refund the P12,087.50 plaintiffappellant paid under protest. No costs.

HELD: The dismissal of the complaint against Bondoc and company is compelled by the equal
protection clause of the constitution. De Venecia Jr. And the respondents Bondoc and company are
similarly situated. Respondent Bondoc,et al. Were included in the complaint only because they
allegedly gave unwarranted favors to De Venecia,Jr. In guaranteeing the latters foreign loans. When
petitioner admitted that no undue favor was granted to De Venecia, Jr. In the grant of such guaranty
facilities and dismissed its complaint against him, petitioner cannot avoid its duty of dismissing its
complaint against respondents Bondoc and company.

Sison, Jr. v. PAGCOR May 14, 1991 Joan Solatorio

Facts: A TV ad proudly announces:


The new PAGCORresponding through responsible gaming.
But the petitioners think otherwise, that is why, they filed the instant petition
seeking to annul the Philippine Amusement and Gaming Corporation (PAGCOR)
CharterPD 1869, because it is allegedly contrary to morals, public policy and
order, and because, inter alia, It violates the equal protection clause of the

Himagan v. People 237 SCRA 538 Ombra


F: Petitioner, a policeman assigned w/ the medical co. of the PNP HQ at Camp Catitigan, Davao City
was implicated in the killing of Benjamin Machitar, Jr. and the attempted murder of Bernabe Machitar.

31

After the informations for murder and attempted murder were filed w/ the RTC, the trial court issued an
order suspending petitioner until termination of the case on the basis of Sec. 47 of RA 6975, w/c
provides:
Sec. 47.Preventive Suspension Pending Criminal Case. Upon the filing of a complaint or information
sufficient in form and substance against a member of the PNP for grave felonies where the penalty
imposed by law is six (6) years and one (1) day or more, the court shall immediately suspend the
accused from office until the case is terminated. Such case shall be subject to continuous trial and shall
be terminated within ninety (90) days from arraignment of the accused.
Petitioner filed a motion to lift the order for his suspension relying on Sec. 42 of PD 807, that his
suspension should be limited to 90 days and also, on our ruling in Deloso v. SB, and Layno v. SB. The
motion and the subsequent MFR were denied. Hence, this petition forcertiorar i andma nda mus.

Ruling: It does not constitute violation of the equal protection clause under the following reasons:
1.
2.

Telebap v. COMELEC 289 SCRA 337 - Danilo Dinopol

HELD: We find the petition to be devoid of merit.


(1) The language of the first sentence is clear, plain and free from ambiguity. xxx The second sentence
xx providing the trial must be terminated w/in 90 days from arraignment does not qualify or limit the first
sentence. The 2 can stand independently of each other. The first refers to the period of suspension.
The 2nd deals w/ the time frame w/in w/c the trial should be finished.
Suppose the trial is not terminated w/in the 90day period, should the suspension of accused be lifted?
Certainly no. While the law uses the mandatory word "shall" bef. the phrase "be terminated w/in 90
days," there is nothing in the law that suggests that the preventive suspension of the accused will be
lifted if the trial is not terminated w/in that period. But this is w/o prejudice to the administrative
sanctions, and, in appropriate cases where the facts so warrant, to criminal or civil liability of the judge.
Should the trial be unreasonably delayed w/o the fault of the accused, he may ask for the dismissal of
the case. Should this be refused, he can compel its dismissal bycer tiorari, prohibition orma nda mus, or
secure his liberty by
(2) Petitioner misapplies Sec. 42 of PD 807. A meticulous reading of the section clearly shows that it
refers to the lifting of the preventive suspension in pending admin. investigation, not in crim. cases, as
here. xxx Sec. 91 of RA 6975 w/c states that the CS law and its implementing rules shall apply to
members of the PNP insofar as the provisions, rules and regulations are not inconsistent w/ RA 6975.
(3) The petitioner's reliance on Layno and Deloso is misplaced. xxx Sec. 13 of RA 3019 upon w/c the
preventive suspension of the accused in Layno and Deloso was based was silent w/ respect to the
duration of the preventive suspension, such that the suspension of the accused therein for a prolonged
and unreasonable length of time raised a due process question. Not so in the instant case. Petitioner is
charged w/ murder under the RPC and it is undisputed that he falls squarely under Sec. 47 RA 6975
w/c categorically states that hissuspension shall last until the case is terminated.
(4) The deliberations of the Bicameral Conference Committee on National Defense relative to the bill
that became RA 6975 reveal the legislative intent to place on preventive suspension a member of the
PNP charged w/ grave felonies where the penalty imposed by law exceeds six yrs. of imprisonment and
w/c suspension continues until the case against him is terminated.

It is allowed by the Constitution itself.


That the Office of the Ombudsman is different from any other investigatory body
because those subjects to its jurisdiction or public officials who through official pressure
and influence can quash, delay or dismiss investigation.

Tiu v. CA GR 127410 Jan. 20, 1999 Camaingking

Facts:
On March 13, 1992, Congress with the approval of the President, pass into law RA 7227
entitled An Act Accelerating the Conversion of Military Reservations Into Other Productive Uses,
Creating the Bases Conversion and Development Authority for this Purpose, Providing Funds Therefor
and Other Purposes.
The President issued Executive Order No. 97-A, specifying the area within which the tax-and-duty-free
privilege was operative. On October 26, 1994, the petitioners challenged the EO 97-A allegedly being
violative of their right to equal protection of the law. The petition is denied for lack of merit. The assailed
decision and resolution are affirmed.
Issue:
Whether or not Executive Order No. 97-A violates the equal protection clause of the
constitution?
Ruling:It is well-settled that the equal-protection guarantee does not require territorial uniformity of
laws. As long as there are actual and material differences between territories, there is no violation of the
constitutional clause. And anyone including the petitioners, possessing the requisite investment capital
can always avail of the same benefits by channeling his or her resources or business operation into the
fenced-off free port zone. We believe that the classification set forth by the executive issuance does not
apply merely to existing conditions. As laid down in RA 7227, the objective is to establish a selfsustaining, industrial, commercial, financial and investment center in the area. There will therefore, a
long term difference between such investment center and the areas outside it. The Court holds that no
undue favor or privilege was extended. The classification occasioned by EO 97-A was not
unreasonable, capricious and unfounded. It was based rather on fair and substantive considerations
that were germane to the legislative purpose.

Almonte v. Vasquez 244 SCRA 286 Analyn Bongabong

Aguinaldo v. COMELEC GR 132774 June 21, 1999 Lynette Latip

Facts: petitioner is charged before the Office of the Ombudsman on the basis of an anonymous letter
sent to the Office of the Ombudsman alleging among others, illegal disbursement of funds, maintaining
ghost employees. Petitioner argued that he cannot be validly investigated and charged of the
allegations because the law requires that in all for complaints must be verified otherwise it would be a
violation of equal protection of the law.

FACTS: Petitioners, at the time of the filing of the petition, were incumbent provincial or municipal
officials in Cagayan seeking to prevent the COMELEC from enforcing during the 1998 elections Section
67 of the Omnibus Election Code (B.P. Blg. 881) which says that any elective official, whether national
or local, running for any office other than the one which he is holding in a permanent capacity, except
for President and Vice-President, shall be considered ipso facto resigned from his office upon the filing
of his certificate of candidacy.

Issue: Whether or not the act of the Ombudsman in proceeding with the investigation on the basis of
unsigned letter violates equal protection of the law.

Petitioners contend that Section 67 is violative of the equal protection clause of the
Constitution, as its classification of persons running for office does not apply equally to all members of

32

the same class.They argue that, Sec 67 gives the reelectionistan undue advantage since he is able to
use the resources, prestige, and influence of his position. The same is not available to one seeking an
office different from the one he is presently holding. This, according to petitioners, does not equalize
the playing field for all candidates.

FACTS: For automatic review by the Supreme Court is the decision dated July 22, 1994, of the
Regional Trial Court, Branch 156, Pasig, convicting accused-appellants SPO2 Elpidio
Mercado y Hernando and SPO1 Aurelio Acebron y Adora, of the Philippine National Police of
Tanay, Rizal, of wilfully, unlawfully, and feloniously kidnap one Richard Buama, a 17 year old
minor and boarded him in a Red car bearing License plate No. CGZ 835 against his will,
brought him to Tanay, Rizal in a safe house and there subjected him to extreme/ brutal
physical violence, and thereafter with abuse of superior strength and evident premeditation
hacked and bludgeoned/clubbed said Richard Buama who thereby sustained mortal wounds
which directly caused his death. The accused-appellants were then sentence to death, to
proportionately indemnify the heirs of the deceased Richard Buama, to pay expenses
incident to the burial; and as moral and exemplary damages, all without subsidiary
imprisonment in case of insolvency and to pay the costs. Then Republic Act No. 7659, An act
to impose the Death Penalty on certain heinous crimes, amending for that purpose the
revised penal laws, and for other purposes was still enforce. Accused-appellants argued that
Republic Act No. 7659 denies equality before the law.
ISSUE: Whether or not Republic Act No. 7659 denies equality before the law?
HELD: The court held that the constitutionality of Republic Act No. 7659 has already been settled in the
Courts 12-3 per curiam Resolution in People vs. Echegaray. The death penalty law makes no
distinction. It applies to all persons and to all classes of personsrich or poor, educated, or
uneducated, religious or non-religious. No particular person or classes of persons are
identified by the law against whom the death penalty shall be exclusively imposed.

ISSUE: Whether or not Section 67, of the Omnibus Election Code violates the equal protection clause
of the Constitution?
RULING: NO. The Supreme Court dismissed the petition and upheld the decision of the COMELEC
which asserted that the classification embodied in Section 67 is reasonable and based on substantial
distinction. Incumbents running for the same position are not considered resigned because the intention
of the law is to allow them to continue serving their constituents and avoid a disruption in the delivery of
essential services. Those running for different positions are considered resigned because they are
considered to have abandoned their present position by their act of running for other posts.
The SC citing the case of Dimaporo v Mitrasaid that Sec 67 seeks to ensure that such
officials serve out their entire term of office by discouraging them from running for another public office
and thereby cutting short their tenure by making it clear that should they fail in their candidacy, they
cannot go back to their former position. This is consonant with the constitutional edict that all public
officials must serve the people with utmost loyalty and not trifle with the mandate which they have
received from their constituents.Sec 67 was crafted with the intention of giving flesh to the constitutional
pronouncement that public service is a public trust.

De Guzman v. COMELEC 336 SCRA Arola


People v. Jalosjos 324 SCRA 689 Richard Atamosa
Facts: The accused-appellant, Romeo Jalosjos, is a full-fledged member of Congress who is confined
at the national penitentiary while his conviction for statutory rape and acts of lasciviousness is pending
appeal. The accused-appellant filed a motion asking that he be allowed to fully discharge the duties of a
Congressman, includingattendance at legislative sessions and committee meetings despite
his having been convicted in the first instance of a non-bailable offense on the basis of popular
sovereignty and the need for his constituents to be represented.

FACTS: R.A 8189 (otherwise known as The Voters Registration Act) was enacted on June
11, 1996 and approved by then President Fidel V. Ramos. Then after the COMELEC issued
several directives recessing the petitioners, who are either City or Municipality election
officers to different stations. Petitioners filed a petition for certiorari and prohibition with urgent
prayer for the issuance of a merit of preliminary injunction and temporary restraining order,
assailing the validity of Section 44 of R.A 8189. Petitioners theorize that Sec.44 of R.A 8189
is violative of the Equal Protection Clause of the 1987 Constitution because it singles out
the City and Municipal election officers of the COMELEC as prohibited from holding office in
the
same
City/
Municipality
for
more
than
four
years.

Issue: Whether or not accused-appellant should be allowed to discharge mandate as member


of House of Representatives
Held: Election is the expression of the sovereign power of the people. However, inspite of its
importance, the privileges and rights arising from having been elected may be enlarged or restricted by
law.

ISSUE:Whether or not Section 44 of R.A 8189 is violative of the Equal Protection Clause in
accordance
of
the
1987
Philippine
Constitution.

The immunity from arrest or detention of Senators and members of theHouse of Representatives arises
from a provision of the Constitution. The privilege has always been granted in a restrictive sense.
Theprovision granting an exemption as a special privilege cannot be extended beyond the ordinary
meaning of its terms. It may not be extended by intendment, implication or equitable considerations.

HELD:The court is not persuaded by petitioners arguments. The Equal Protection Clause of
the 1987 Constitution permits a valid classification under the following classifications. 1.) The
Classification must rest on substantial distinction, 2.) It must be germane to the purpose of
the law, 3.) it must not be limited to two existing conditions only. And, 4.) it must apply equally
to all members of the same class. Wherefore the petition is dismissed and the
constitutionality
of
section
44
of
RA
8189
is
upheld.

The accused-appellant has not given any reason why he should be exempted from the operation of
Sec. 11, Art. VI of the Constitution. The members of Congress cannot compel absent members to
attend sessions if the reason for the absence is a legitimate one. The confinement of a Congressman
charged with a crime punishable by imprisonment of more than six years is not merely authorized by
law, it has constitutional foundations. To allow accused-appellant to attend congressional sessions and
committee meetings for 5 days or more in a week will virtually make him a free man with all the
privileges appurtenant to his position. Such an aberrant situation not only elevates accused-appellants
status to that of a special class, it also would be a mockery of the purposes of the correction system.

People v. Mercado GR 116239, Nov. 29, 2000 Sherilyn Saladaga

33

1996, the DOLE Acting Secretary, Crescenciano B. Trajano, issued an Order resolving the parity and
representation issues in favor of the School. Then DOLE Secretary Leonardo A. Quisumbing
subsequently denied petitioners motion for reconsideration in an Order dated March 19, 1997.
Petitioner now seeks relief in this Court.

People v. Piedra 350 SCRA 163 Feliciano Beldad


FACTS: Accused-appellant Carol M. dela Piedra questions her conviction for illegal recruitment in
large scale and assails, as well, the constitutionality of the law defining and penalizing said crime.

Issue:
Whether or not foreign-hires should be included in the appropriate bargaining unit, eventually
caused a deadlock between the parties?

Petitioner was allegedly recruiting workers particularly nurses to be sent to Singapore. On January 30,
1994, she is briefing some people in the house of certain Jasmine Alejandro, included in her briefing is
the breakdown of fees involved such as P 30,000 for the Visa and round trip ticket, P5, 000.00placement fees, initial payment of P 2,000 while P 30,000 will be salary deduction. After the briefing
Maria Lourdes gave P 2,000 pesos to Jasmine as initial payment and alleged that the latter is
authorized by Carol dela Piedra to receive payment. After a while, operatives from Criminal
Investigation Service arrived and apprehended herein petitioner. Carol was charged and convicted by
the trial court of illegal recruitment and was sentenced Life Inprisonment. Upon appeal, accused
questions her conviction for illegal recruitment in large scale and assails, as well, the constitutionality
of the law defining and penalizing said crime.

Held:
No. To include foreign-hires in a bargaining unit with local-hires would not assure either group
the exercise of their respective collective bargaining rights. WHEREFORE, the petition is GIVEN DUE
COURSE. The petition is hereby GRANTED IN PART. The Orders of the Secretary of Labor and
Employment dated June 10, 1996 and March 19, 1997, are hereby REVERSED and SET ASIDE
insofar as they uphold the practice of respondent School of according foreign-hires higher salaries than
local-hires.
Note.The constitutional policy of providing full protection to labor is not intended to oppress
or destroy management. (Capili vs. National Labor Relations Commission, 270 SCRA 488 [1997])

ISSUES:(1) Whether or not sec. 13 (b) of P.D. 442, as amended, otherwise known as the illegal
recruitment law is unconstitutional as it violates the due process clause.

Central Bank Employees Assn. v. BSP 446 SCRA 299 Kinang

(2) Whether or not accused was denied equal protection and therefore should be exculpated.
HELD:(1) In the first issue, the court ruled in affirmative, Article 13 (b) of the Labor Code defining
recruitment and placement is void for vagueness and, thus, violates the due process clause.

FACTS On July 3, 1993, R.A. No. 7653 (the New Central Bank Act) took effect. It abolished the old
Central Bank of the Philippines, and created a new BSP.
On June 8, 2001, almost eight years after the effectivity of R.A. No. 7653, petitioner Central
Bank (now BSP) Employees Association, Inc., filed a petition for prohibition against BSP and the
Executive Secretary of the Office of the President, to restrain respondents from further implementing
the last proviso in Section 15(c), Article II of R.A. No. 7653, on the ground that it is unconstitutional.
The thrust of petitioners challenge is that the above proviso makes an unconstitutional cut
between two classes of employees in the BSP, viz: (1) the BSP officers or those exempted from the
coverage of the Salary Standardization Law (SSL) (exempt class); and (2) the rank-and-file (Salary
Grade [SG] 19 and below), or those not exempted from the coverage of the SSL (non-exempt class). It
is contended that this classification is a classic case of class legislation, allegedly not based on
substantial distinctions which make real differences, but solely on the SG of the BSP personnels
position. Petitioner also claims that it is not germane to the purposes of Section 15(c), Article II of R.A.
No. 7653, the most important of which is to establish professionalism and excellence at all levels in the
BSP.
In sum, petitioner posits that the classification is not reasonable but arbitrary and capricious,
and violates the equal protection clause of the Constitution. Petitioner also stresses: (a) that R.A. No.
7653 has a separability clause, which will allow the declaration of the unconstitutionality of the proviso
in question without affecting the other provisions; and (b) the urgency and propriety of the petition, as
some 2,994 BSP rank-and-file employees have been prejudiced since 1994 when the proviso was
implemented. Petitioner concludes that: (1) since the inequitable proviso has no force and effect of law,
respondents implementation of such amounts to lack of jurisdiction; and (2) it has no appeal nor any
other plain, speedy and adequate remedy in the ordinary course except through this petition for
prohibition, which this Court should take cognizance of, considering the transcendental importance of
the legal issue involved.

Due process requires that the terms of a penal statute must be sufficiently explicit to inform those who
are subject to it what conduct on their part will render them liable to its penalties.
(2) Anent the second issue, the court said no, the prosecution of one guilty person while others equally
guilty are not prosecuted, is not, by itself, a denial of the equal protection of the laws. The unlawful
administration by officers of a statute fair on its face, resulting in its unequal application to those who
are entitled to be treated alike, is not a denial of equal protection unless there is shown to be present
in it an element of intentional or purposeful discrimination. But a discriminatory purpose is not
presumed, there must be a showing of clear and intentional discrimination.

International School v. Quisumbing June 1, 2000 Allan Tamayao

Facts:
Private respondent International School, Inc. (the School, for short), pursuant to Presidential
Decree 732, is a domestic educational institution established primarily for dependents of foreign
diplomatic personnel and other temporary residents.1 To enable the School to continue carrying out its
educational program and improve its standard of instruction, Section 2(c) of the same decree
authorizes the School to employ its own teaching and management personnel selected by it either
locally or abroad, from Philippine or other nationalities, such personnel being exempt from otherwise
applicable laws and regulations attending their employment, except laws that have been or will be
enacted for the protection of employees. A collective bargaining agreement were held on June 1995,
petitioner International School Alliance of Educators, a legitimate labor union and the collective
bargaining representative of all faculty members4 of the School, contested the difference in salary
rates between foreign and local-hires. On September 7, 1995, petitioner filed a notice of strike. The
failure of the National Conciliation and Mediation Board to bring the parties to a compromise prompted
the Department of Labor and Employment (DOLE) to assume jurisdiction over the dispute. On June 10,

Respondent BSP, in its comment, contends that the provision does not violate the equal
protection clause and can stand the constitutional test, provided it is construed in harmony with other
provisions of the same law, such as fiscal and administrative autonomy of BSP, and the mandate of

34

the Monetary Board to establish professionalism and excellence at all levels in accordance with sound
principles of management.
The Solicitor General, on behalf of respondent Executive Secretary, also defends the validity
of the provision. Quite simplistically, he argues that the classification is based on actual and real
differentiation, even as it adheres to the enunciated policy of R.A. No. 7653 to establish professionalism
and excellence within the BSP subject to prevailing laws and policies of the national government.

to retain the articles seized, be declared illegal and set aside, and prays that all the articles in question
be returned to him.
Issue : Whether or not the seizure of evidence to use in an investigation is constitutional?
Held :Section 1, paragraph 3, of Article III of the Constitution, relative to the bill of rights, provides that
"The right of the people to be secure in their persons, houses, papers, and effects against
unreasonable searches and seizures shall not be violated, and no warrants shall issue but upon
probable cause, to be determined by the judge after examination under oath or affirmation of the
complainant and the witnesses he may produce, and particularly describing the place to be searched,
and the persons or things to be seized.

ISSUE
Whether the provision of law initially valid, becomes subsequently unconstitutional on the
ground that its continued operation would violate equal protection of the law?
HELDYes,with the passage of the subsequent laws amending the charter of 7 other governmental
institutions (GFIs) removing limitations on employees,the continued operation of the limitation on
Central Bank employees under section 15(c), Article II Central Bank law constitutes invidious
discrimination on the 2994 rank and file employees of the Bangko Central ngPilipinas.

Section 97 of General Orders, No. 58 provides that "A search warrant shall not issue except for
probable cause and upon application supported by oath particularly describing the place to be
searched and the person or thing to be seize."
It will be noted that both provisions require that there be not only probable cause before the issuance of
a search warrant but that the search warrant must be based upon an application supported by oath of
the applicant and the witnesses he may produce. In its broadest sense, an oath includes any form of
attestation by which a party signifies that he is bound in conscience to perform an act faithfully and
truthfully; and it is sometimes defined as an outward pledge given by the person taking it that his
attestation or promise is made under an immediate sense of his responsibility to God. The oath
required must refer to the truth of the facts within the personal knowledge of the petitioner or his
witnesses, because the purpose thereof is to convince the committing magistrate, not the individual
making the affidavit and seeking the issuance of the warrant, of the existence of probable cause.

II. Section 2
1.

Requisites of a Valid Warrant

People v. Veloso 48 Phil. 169 Jajurie

Facts: Jose M.a Veloso was a member of the House of Representatives of the Philippine Legislature:
He was also the manager of the club known as the Parliamentary Club. The police of Manila had
reliable information that the so-called Club was nothing more than a gambling house. On May 25, 1923,
Detective Andres Geronimo of the secret service of the City of Manila, applied for, and obtained a
search warrant from Judge Garduo of the municipal court. Thus provided, the police attempted to raid
the Club on the same day. Veloso told the police that they do not have the right to search the house

The true test of sufficiency of an affidavit to warrant issuance of a search warrant is whether it has been
drawn in such a manner that perjury could be charged thereon and affiant be held liable for damages.
In view of the foregoing and under the above-cited authorities, it appears that the affidavit, which served
as the exclusive basis of the search warrant, is insufficient and fatally defective by reason of the
manner in which the oath was made, and therefore, it is hereby held that the search warrant in question
and the subsequent seizure of the books, documents and other papers are illegal and do not in any
way warrant the deprivation to which the petitioner was subjected.

Issue: WON the search warrant is valid


Held: Yes. 1. A description of a place to be searched is sufficient if the officer with the warrant can, with
reasonable effort, ascertain and identify the place intended. 2. The warrant for the apprehension of an
unnamed party is void, "except in those cases where it contains a descriptio personae such as will
enable the officer to identify the accused." The description must be sufficient to indicate clearly the
proper person upon whom the warrant is to be served.

Stonehill v. Diokno June 19, 1967 Tanjusay

FACTS:The Respondents-Judgesissued, on different dates, a total of 42 search warrants against


petitioners directed to any peace officer, to search the persons or the premises of their offices,
warehouses and/or residences, and to seize and take possession of the following personal property to
wit:
"Books of accounts, financial records, vouchers, correspondence, receipts, ledgers, journals, portfolios,
credit journals, typewriters, and other documents and/or papers showing all business transactions
including disbursements receipts, balance sheets and profit and loss statements and Bobbins (cigarette
wrappers) as the subject of the offense; stolen or embezzled and proceeds or fruits of the offense," or
"used or intended to be used as the means of committing the offense," which is described in the
applications adverted to above as "violation of Central Bank Laws, Tariff and Customs Laws, Internal
Revenue (Code) and the Revised Penal Code."
Alleging that the aforementioned search warrants are null and void, as contravening the Constitution
and the Rules of Courtbecause, inter alia: (1) they do not describe with particularity the documents,

Alvarez v. CFI 64 Phil. 33 Engr. Sali

ORIGINAL ACTION in the Supreme Court. Mandamus.


Facts :The petitioner asks that the warrant of June 3, 1936, issued by the Court of First Instance of
Tayabas, ordering the search of his house and the seizure, at any time of the day or night, of certain
accounting books, documents and papers belonging to him in his residence situated in Infanta,
Province of Tayabas, as well as the order of a later date, authorizing the agents of the Anti-Usury Board

35

books and things to be seized; (2) cash money, not mentioned in the warrants, were actually seized; (3)
the warrants were issued to fish evidence against the aforementioned petitioners in deportation cases
filed against them; (4) the searches and seizures were made in an illegal manner; and (5) the
documents, papers and cash money seized were not delivered to the courts that issued the warrants,
to be disposed of in accordance with law. On March 20, 1962, said petitioners filed with the Supreme
Court this original action for certiorari, prohibition, mandamus and injunction, and prayed that, pending
final disposition of the present case, a writ of preliminary injunction be issued restraining Respondents
Prosecutors, their agents and/or representatives from using the effects seized as aforementioned, or
any copies thereof, in the deportation cases already adverted to, and that, in due course, thereafter,
decision be rendered quashing the contested search warrants and declaring the same null and void,
and commanding the respondents, their agents or representatives to return to petitioners herein, in
accordance with Section 3, Rule 67, of the Rules of Court, the documents, papers, things and cash
moneys seized or confiscated under the search warrants in question.

him her stenographic notes; and thereafter, J Ruiz asked respondent Logronio to take the oath and
warned him that if his deposition was found to be false and without legal basis, he could be charged for
perjury. J Ruiz signed de Leons application for search warrant and Logronios deposition. The search
was subsequently conducted.
ISSUE: Whether or not there had been a valid search warrant.
RULING:The SC ruled in favor of Bache on three grounds.
1. J Ruiz failed to personally examine the complainant and his witness.
Personal examination by the judge of the complainant and his witnesses is necessary to enable him to
determine the existence or non-existence of a probable cause.

ISSUE: Whether or not the following description stated in the facts contained in a search warrant is
sufficient

2. The search warrant was issued for more than one specific offense.

RULING: NO. The court ruled against the respondents issuing search warrants. None of those
essential requisites of a valid warrant has been complied with in the contested warrants. Indeed, the
same were issued upon applications stating that the natural and juridical persons therein named had
committed a "violation of Central Bank Laws, Tariff and Customs Laws, Internal Revenue (Code) and
Revised Penal Code." In other words, no specific offense had been alleged in said applications. The
averments thereof with respect to the offense committed were abstract. As a consequence, it was
impossible for the judges who issued the warrants to have found the existence of probable cause, for
the same presupposes the introduction of competent proof that the party against whom it is sought has
performed particular acts, or committed specific omissions, violating a given provision of our criminal
laws

The search warrant in question was issued for at least four distinct offenses under the Tax
Code. As ruled in Stonehill Such is the seriousness of the irregularities committed in connection with
the disputed search warrants, that this Court deemed it fit to amend Section 3 of Rule 122 of the former
Rules of Court that a search warrant shall not issue but upon probable cause in connection with one
specific offense. Not satisfied with this qualification, the Court added thereto a paragraph, directing that
no search warrant shall issue for more than one specific offense.
3. The search warrant does not particularly describe the things to be seized.

General search warrants are outlawed because the Supreme Court had already amended the Old
Rules of Court by providing in the Revised Rules of Court that No search warrant shall issue for more
than one specific offense. Search warrants authorizing the seizure of books of accounts and records
"showing all the business transactions" of certain persons, regardless of whether the transactions were
legal or illegal, contravene the explicit command of the Bill of Rights that the things to be seized should
be particularly described and defeat its major objective of eliminating general warrants.

The documents, papers and effects sought to be seized are described in the Search Warrant
Unregistered and private books of accounts (ledgers, journals, columnars, receipts and disbursements
books, customers ledgers); receipts for payments received; certificates of stocks and securities;
contracts, promissory notes and deeds of sale; telex and coded messages; business communications,
accounting and business records; checks and check stubs; records of bank deposits and withdrawals;
and records of foreign remittances, covering the years 1966 to 1970.
The description does not meet the requirement in Art III, Sec. 1, of the Constitution, and of Sec. 3, Rule
126 of the Revised Rules of Court, that the warrant should particularly describe the things to be seized.

Bache & Co. v Ruiz 37 SCRA 823 Casil

A search warrant may be said to particularly describe the things to be seized when the description
therein is as specific as the circumstances will ordinarily allow or when the description expresses a
conclusion of fact not of law by which the warrant officer may be guided in making the search and
seizure or when the things described are limited to those which bear direct relation to the offense for
which the warrant is being issued.

Search and Seizure Personal Examination of the Judge


FACTS:On 24 Feb 1970, Commissioner Vera of Internal Revenue, wrote a letter addressed to J Ruiz
requesting the issuance of a search warrant against petitioners for violation of Sec 46(a) of the NIRC, in
relation to all other pertinent provisions thereof, particularly Sects 53, 72, 73, 208 and 209, and
authorizing Revenue Examiner de Leon make and file the application for search warrant which was
attached to the letter. The next day, de Leon and his witnesses went to CFI Rizal to obtain the search
warrant. At that time J Ruiz was hearing a certain case; so, by means of a note, he instructed his
Deputy Clerk of Court to take the depositions of De Leon and Logronio. After the session had
adjourned, J Ruiz was informed that the depositions had already been taken. The stenographer read to

36

Placer v. Villanueva 126 SCRA 463 Marcial

Facts: Petitioners

filed information in

the

city

court

and

they

certified that

Preliminary Investigation and Examination had been conducted and that prima facie cases have been

Facts:Assailed in this petition for certiorari prohibition and mandamus with preliminary mandatory and
prohibitory injunction is the validity of two [2] search warrants issued on December 7, 1982 by Judge
Ernani Cruz-Pano of the then CFI of Rizal [Quezon City], under which the premises of the "Metropolitan
Mail" and "We Forum" newspapers, respectively, were searched, and office and printing machines,
equipment, paraphernalia, motor vehicles and other articles used in the printing, publication and
distribution of the said newspapers, as well as numerous papers, documents, books and other written
literature alleged to be in the possession and control of petitioner Jose Burgos, Jr. publisher-editor of
the
"We
Forum"
newspaper,
were
seized.
Petitioners further pray that a writ of preliminary mandatory and prohibitory injunction be issued for the
return of the seized articles, and that respondents be enjoined from using the articles thus seized as
evidence against petitioner Jose Burgos, Jr. and the other accused in Criminal Case No. Q- 022782 of
the Regional Trial Court of Quezon City, entitled People v. Jose Burgos, Jr. et al.

found. Upon receipt of said informations, respondent judge set the hearing of the criminal cases to
determine propriety of issuance of warrants of arrest. After the hearing, respondent issued an order
requiring petitioners to submit to the court affidavits of prosecution witnesses and other documentary
evidence in support of the informations to aid him in the exercise of his power of judicial review of the
findings of probable cause by petitioners. Petitioners petitioned for certiorari and mandamus to compel
respondent

to

issue

warrants

of arrest.

They

contended

that

the

Burgos, Sr. v. Chief of Staff, AFP 133 SCRA 800 Sarita

fiscals certification in

Issue: Whether or Not the 2 search warrants were validly issued and executed.

the informations of the existence of probable cause constitutes sufficient justification for the judge to
issue warrants of arrest.

Held: In regard to the quashal of warrants that petitioners should have initially filed to the lower court,
this Court takes cognizance of this petition in view of the seriousness and urgency of the constitutional
Issue raised, not to mention the public interest generated by the search of the "We Forum" offices
which was televised in Channel 7 and widely publicized in all metropolitan dailies. The existence of this
special circumstance justifies this Court to exercise its inherent power to suspend its rules. With the
contention pertaining to laches, the petitioners gave an explanation evidencing that they have
exhausted other extra-judicial efforts to remedy the situation, negating the presumption that they have
abandoned their right to the possession of the seized property.

Issue: Whether or Not respondent city judge may, for the purpose of issuing warrants of arrest, compel
the fiscal to submit to the court the supporting affidavits and other documentary evidence presented
during the preliminary investigation.

On the enumerated reasons:


1. This objection may properly be considered moot and academic, as petitioners themselves conceded
during the hearing on August 9, 1983, that an examination had indeed been conducted by respondent
judge of Col. Abadilla and his witnesses.

Held: Judge may rely upon the fiscals certification for the existence of probable cause and on the basis
thereof, issue a warrant of arrest. But, such certification does not bind the judge to come out with the
warrant. The issuance of a warrant is not a mere ministerial function; it calls for the exercise of judicial

2. The defect pointed out is obviously a typographical error. Precisely, two search warrants were
applied for and issued because the purpose and intent were to search two distinct premises. It would
be quite absurd and illogical for respondent judge to have issued two warrants intended for one and the
same place.

discretion on the part of issuing magistrate. Under Section 6 Rule 112 of the Rules of Court, the judge
must satisfy himself of the existence of probable cause before issuing a warrant of arrest. If on the face

3. Section 2, Rule 126, of the Rules of Court, does not require that the property to be seized should be
owned by the person against whom the search warrant is directed. It may or may not be owned by him.

of the information, the judge finds no probable cause, he may disregard the fiscals certification and

4. Petitioners do not claim to be the owners of the land and/or building on which the machineries were
placed. This being the case, the machineries in question, while in fact bolted to the ground, remain
movable property susceptible to seizure under a search warrant.

require submission of the affidavits of witnesses to aid him in arriving at the conclusion as to existence
of probable cause.

5. The broad statements in the application and joint affidavit are mere conclusions of law and does not
satisfy the requirements of probable cause. Deficient of such particulars as would justify a finding of the
existence of probable cause, said allegation cannot serve as basis for the issuance of a search warrant
and it was a grave error for respondent judge to have done so. In Alvarez v. Court of First Instance, this
Court ruled that "the oath required must refer to the truth of the facts within the personal knowledge of
the petitioner or his witnesses, because the purpose thereof is to convince the committing magistrate,
not the individual making the affidavit and seeking the issuance of the warrant, of the existence of

Petition dismissed.

37

probable cause." Another factor which makes the search warrants under consideration constitutionally
objectionable is that they are in the nature of general warrants. The description of the articles sought to
be seized under the search warrants in question are too general.

administrative remedies available under the law has lost factual support.
ISSUES: Whether or Not the constitutional rights of Beltran were violated when respondent RTC judge
issued a warrant for his arrest without personally examining the complainant and the witnesses, if any,
to determine probable cause?

With regard to the respondents invoking PD 885, there is an absence of any implementing rules and
regulations promulgated by the Minister of National Defense. Furthermore, President Marcos himself
denies the request of military authorities to sequester the property seized from petitioners. The closure
of the premises subjected to search and seizure is contrary to the freedom of the press as guaranteed
in our fundamental law. The search warrants are declared null and void.

HELD: No. what Art 3 Sec 2 of the Constitution underscores is the exclusive and personal responsibility
of the issuing judge to satisfy himself of the existence of probable cause. In doing so, the judge is not
required to personally examine the complainant and his witness. Following established doctrine and
procedure, he shall: (1) personally evaluate the report and the supporting documents submitted by the
fiscal regarding the existence of probable cause and, on the basis thereof, issue a warrant of arrest; or
(2) if on the basis thereof he finds no probable cause, he may disregard the fiscal's report and require
the submission of supporting affidavits of witnesses to aid him in arriving at a conclusion as to the
existence of probable cause.
Petition dismissed.

Corro v. Lising 137 SCRA 541 Sali / Marines

FACTS: Respondent Judge issued a search warrant for the seizure of articles allegedly used by
petitioner in committing the crime of sedition. Seized were printed copies of the Philippine Times,

Salazar v. Achacoso 183 SCRA 145 Allan R. Taga-oc

newspaper dummies, typewriters, mimeographing machines and tape recorders, video machines and

Facts: Pursuant to the powers vested by PD 1920 and EO 1022, POEA Administrator Achacoso
ordered the closure of the recruitment agency of Horty Salazar, having verified that she had no license
to operate a recruitment agency. He further ordered the seizure of the documents and paraphernalias,
being used or intended to be used as the means of commiting illegal recruitment. This order was
enforced on 26 January 1988. Petitioner filed this suit for prohibition.

tapes. The petitioner moved to quash the warrant but his motion was denied.

Issue: May the POEA (or the Sec. of Labor) validly issue warrants of serach and seizure?

ISSUE: Whether or not warrant valid?

HELD: NO. The provisions of PD 1920 and EO 1022, now embodied in Art. 38 of the Labor Code, are
the dying vestiges of authoritarian rule in its twilights moments. Under Art. III, Sec 2 of the 1987
Constitution, it is only judges and no other, who may issue warrants of arrest and search. The exception
is in cases of deportation of illegal and undesirable aliens, whom the President of the Commissioner of
Immigration may order arrested, following a final order of deportation, for the purpose of deportation.
The Sec. of Labor , not being a judge. may no longer issue search or arrest warrants. Hence, the authorities must go through the
judicial process. To that extent, we declare Art. 38, par. C of the Labor Code, unconstitutional and of no force and effect.&n bsp; a.
Existence of probable cause. Probable cause is such facts andcircumstances as would reasonably make a prudent man believe
that a crime have been committed and that the documents or things sought to be searched and seized are in the possession of
the person against whom the warrant is sought. Without probable cause, there can be no valid search warrant.

RULING: The statements made in the affidavits are mere conclusions of law and do not satisfy the
requirement of probable cause. The language used is all embracing as to include all conceivable words
and equipment of petitioner regardless of whether they are legal or illegal. The search warrant under
consideration was in the nature of a general warrant which is objectionable.

Board of Comm. (CID) v. De la Rosa 197 SCRA 853 Manalo

Soliven v. Makasiar 167 SCRA 394 Dalus

On July 6, 1960, Santiago Gatchalian, grandfather of William Gatchalian, was recognized by the BOI as
a native born Filipino citizen. Santiago Gatchalian testified that he has 5 children. On June 27, 1961,
William Gatchalian then a twelve year old minor arrived in Manila and sought admission as Filipino
citizen which was eventually granted by the board of special inquiry. However, the Secretary of Justice
issued a memorandum setting aside all decisions and directed the Board of Commissions to review all
cases where entry was allowed among which was that of William Gatchalian.

FACTS: In these consolidated cases, three principal issues were raised. Subsequent events have
rendered the first issue moot and academic. On March 30, 1988, the Secretary of Justice denied
petitioners' motion for reconsideration and upheld the resolution of the Undersecretary of Justice
sustaining the City Fiscal's finding of a prima facie case against petitioners. A second motion for
reconsideration filed by petitioner Beltran was denied by the Secretary of Justice on April 7, 1988. On
appeal, the President, through the Executive Secretary, affirmed the resolution of the Secretary of
Justice on May 2, 1988. The motion for reconsideration was denied by the Executive Secretary on May
16, 1988. With these developments, petitioners' contention that they have been denied the

ISSUE: Whether or not warrant of arrest valid?


HELD: Immigration Laws; Bureau of Immigration; Arrests; A warrant of arrest issued by the
Commissioner of Immigration for purposes of investigation only, as in the case at bar, is null and void

38

for being unconstitutional.Coming now to the contention of petitioners that the arrest of respondent
follows as a matter of consequence based on the warrant of exclusion issued on July 6, 1962, coupled
with the Arocha and Vivo cases (Rollo, pp. 33), the Court finds the same devoid of merit. Sec. 37 (a) of
Commonwealth Act No. 613, as amended, otherwise known as the Immigration Act of 1940, reads:
Sec. 37. (a) The following aliens shall be arrested upon the warrant of the Commissioner of
Immigration or of any other officer designated by him for the purpose and deported upon the warrant of
the Commissioner of Immigration after a determination by the Board of Commissioner of the existence
of the ground for deportation as charged against the alien. (Italics supplied) From a perusal of the
above provision, it is clear that in matters of implementing the Immigration Act insofar as deportation of
aliens are concerned, the Commissioner of Immigration may issue warrants of arrest only after a
determination by the Board of Commissioners of the existence of the ground for deportation as charged
against the alien. In other words, a warrant of arrest issued by the Commissioner of Immigration, to be
valid, must be for the sole purpose of executing a final order of deportation. A warrant of arrest issued
by the Commissioner of Immigration for purposes of investigation only, as in the case at bar, is null and
void for being unconstitutional.

Another motion for reconsideration was filed by the petitioner dated Sept. 1,1987, but was likewise
denied by Judge Cruz in an order dated Oct. 19, 1987. Hence, this special civil action for certiorari.
Issue: Whether or not issuance of Search Warrant No. 1 was tainted with irregularity?
Held: YES. The petition is granted. Search Warrant No. 1 is hereby declared null and void. Respondent
judge of RTC of Negros Oriental, Branch XXXIII is directed to return to petitioner Antonieta Silva the
amount of Php 1,231.00 which was earlier been seized from her by virtue of the Illegal Search Warrant.
The decision is immediately executory at no costs. In issuing a search warrant, the judge must strictly
comply with the constitutional and statutory requirement that he must determine the existence of
probable cause by personally examining the applicant and witnesses in form of searching questions
and answer. His failure to comply with this requirement constitutes grave abuse of discretion. The
officers implementing the search warrant clearly abused their authority when they seized the money of
Antonieta Silva. This is highly irregular considering that she was not even named as one of the
respondents and that the warrant did not indicate the seizure of money but only marijuana
leave, cigarettes and joints.
Allado v. Diokno 232 SCRA 192 Lagbas

Lim, Sr. v. Judge Felix Feb. 19, 1991 Guisadio

Facts: Petitioners Diosdado Jose Allado and Roberto L. Mendoza are partners of Law Firm of Salonga,
Hernandez, and Allado.on an alleged extrajudicial confession of a security guard, Escolastico Umbal, a
dischargee of the Philippine Constabulary, was implicating them as the brains behind the alleged
kidnapping and slaying of one Eugene Alexander Van Twest, a German National. In addition, the
petitioners are accused of the said heinous crime by the Presidential Anti-Crime Commission (PACC)
and is ordered arrested without bail by respondent judge, Hon Roberto C. Diokno.

Facts:
The herein petitioners are accused of the crime of multiple murder and frustrated murder in
connection with the airport incident, which occur on March 17, 1989 at about 7:30 o clock in the
morning at municipality of Masbate, province of Masbate. Respondent judge and his duly authorized
representatives or agents commits a grave abuse of discretion in enforcing and implementing the
warrant arrest without bail issued against the petitioner. The determination of probable cause was made
by Provincial Prosecutor. The constitutional requirements have not been satisfied.

The Petitioner Atty. Allado filed an appeal with the Secretary of Justice seeking review and

Issue
Whether or not a judge may issue a warrant of arrest without bail by simply relying on the
prosecutors certification and recommendation that a probable cause exists.

reversal of an undated resolution of the panel of prosecutors which they heard over the radio, finding a
prima facie case against them that information had already filed in court, which appeal was by petitioner

Ruling:
The respondent judge cannot ignore the clear words of the 1987 constitution which
requires. Probable cause to be personally determined by the judge. Not by any other officer or person.
There was no basis for the respondent judge to make his own personal determination regarding the
existence of a probable cause for the issuances of warrant of arrest as mandated by the constitution.

Mendoza. However, respondent judge issued the assailed warrant of arrest against petitioners. Hence,
the petitioners filed Regional Trial Court of Makati, Metro Manila the instant petition for certiorari and
prohibition with prayer for a temporary restraining order.

Silva v. Pres. Judge of RTC of Negros Or. 203 SCRA 140 Ottong
Issue: Whether or not the respondent judge acted with grave abuse of discretion in issuing the warrant
of warrant of arrest.

Facts In this special civil action for certiorari, petitioners seek the nullification of Search Warrant 1
issued by respondent judge as well as the return of money in the amount of P1,231.00 seized from
petitioner Antonieta Silva. The respondent judge issued Search Warrant for the petitioner upon
receiving an application along with the deposition of witness on June 13, 1986 for violation of Republic
act 6425, otherwise known as the Dangerous Drugs Act of 1972. In the search warrant items to be
search are cited such as marijuana dried leaves, cigarettes, and joint. In the course of the search,
the serving officers aside from the items listed also seized money belonging to Antonieta Silva in the
amount of Php 1,231.00. The latter then filed a motion for the return of the said amount on grounds that
search warrant only authorized seizure of marijuana dried leaves, cigarettes and joint. The herein
respondent issued an order holding in abeyance the disposition of said amount pending the filing of
appropriate charges in connection with the search warrant. Subsequently, petitioners filed a motion to
quash Search Warrant No. 1. However, their motion was denied due to lack of merit by respondent trial
court through Judge Cruz who then replaced the retire Judge Ontal.

Ruling: In the Order of respondent judge, it is expressly stated that "[t]his court aftercareful evaluation of
the evidence on record, believes and rules that probable causeexists; and therefore, a warrant ofarrest
should be issued. "However, we are unableto see how respondent judge arrived at such ruling. We
have painstakinglyexamined the records and we cannot find any support for his conclusion. On
thecontrary, we discern a number of reasons why we consider the evidence submittedto be insufficient
for a finding of probable cause against petitioners. T h e P A C C r e l i e s h e a v i l y o n t h e
s w o r n s t a t e m e n t o f S e c u r i t y G u a r d U m b a l w h o supposedly confessed his
participation in the alleged kidnapping and murder of Van T w e s t . F o r o n e , t h e r e i s
s e r i o u s d o u b t o n V a n T w e s t ' s r e p o r t e d d e a t h s i n c e the corpus
delicti has not been established, nor have his remains been recovered.U m b a l c l a i m s t h a t Van
T we s t w a s c o m p l e t e l y b u r n e d i n t o a s h e s w i t h t h e u s e o f gasoline and rubber tires from
around ten o'clock in the evening to six o'clock thenext morning. This is highly improbable, if not

39

ridiculous. A human body cannot bepulverized into ashes by simply burning it with the use of gasoline
and rubber tiresin an open field. Even crematoria use entirely closed incinerators where the corpseis
subjected to intense heat. Thereafter, the remains undergo a process where thebones are completely
ground to dust.Strangely, if not awkwardly, after Van Twest'sreported abduction which culminatedi n h i s
d e c i m a t i o n b y c r e m a t i o n , h i s c o u n s e l c o n t i n u e d t o r e p r e s e n t h i m b e f o r e judicial and
quasi-judicial proceedings. Hence, even Asst. Solicitor General Estoestabelieves that counsel of Van
Twest doubted the latter's death.Verily, respondent judge committed grave abuse of discretion in
issuing the warrantf o r t h e a r r e s t o f p e t i t i o n e r s i t a p p e a r i n g t h a t h e d i d n o t p e r s o n a l l y
examine theevidence nor did he call for the complainant and his witnesses in the
f a c e o f t h e i r incredible accounts. Instead, he merely relied on the certification of the
prosecutorst h a t p r o b a b l e c a u s e e x i s t e d . F o r , o t h e r w i s e , h e w o u l d h a v e
found out thattheevidence thus far presented was utterly
i n s u f f i c i e n t t o w a r r a n t t h e a r r e s t o f petitioners.

People v. Martinez 235 SCRA 171 amilbahar


Facts: On October 27,1989, after receiving information that appelant was engaged in tha sale of Shabu
at his residence at the Bureau of Air Transportation (BAT) Compound, Baliwasan Moret, Zamboanga
City,National Bureau of Investigation (NBI) Agent Bienvenido Salvo of the NBI Regional Office in
Zamboanga City Direct his asset or confidential informer Edgar Pelin to purchase from appellant
P200.00 worth of prohibited drug.
On November 6, 1989, the buy-bust operation was set up by the NBI Regional Office with
the assistance of Zamboanga City Police. a raid party composed of four separate teams was formed for
the buy-bust operation. The group arrived at the BAT Compound at around 2:00 oclock in the
afternoon of said date. Agent Salvo gave Pelin P200.00 in marked money for the purchase of the
shabu.pelin entered into the house of appellants and gave the marked P200.00 to appellant who turn
gave him a deck of shabu.
The search team led by Agent Salvo then rushed forward and entered appellants house
while tge rest if the raid party secured the area. Pelin turn over the deck of shabu to the NBI custodian.
Agent Salvo presented a copy of the research warrant to appellant and thereafter the search team
conducted a search premises.
The accused-appellant argues that he is not the person named in the search warrant issued
in connection with the buy-bust operation, his name being Abelardo Martinez and not Alexander
Martinez. That being the case, all things seized by virtue of said warrant are inadmissible in evidence.

Ortiz v. Palaypayon 234 SCRA 391 Abdurajak


FACTS: Judge Lucio Palaypayon of the Municipal Trial Court of Tinambac, Camarines Sur is
administratively charged with gross ignorance of the law tainted with vindictiveness and oppression
relative to a criminal case pending before respondent judge. The charged stemmed from a complaint
for damage to property thru reckless imprudence filed before the respondents sala on May 19,1993. It
appears that respondent judge issued an order for the arrest of accused Julianna Lu, Rodrigo Vasquez
and herein complainant, David Ortiz. Furthermore, it is alleged in the administrative complaint that
herein complainant was wrongfully included as one of the accused. Respondent judge, instead of filing
his comment on the complaint, filed a motion to dismiss alleging lack of factual or legal basis. He claims
that he personally examined in writing and under oath the private complainant and his witnesses by
asking the same questions propounded to them in their sworn statements and the same answers are
given. On September 3, 1993, respondent judge submitted his comment which merely reiterates the
arguments raised in his motion to dismiss. On May 19, 1993, Rosalinda Tanay and her husband,
Roberto Tanay executed sworn statements in question and answer form before the Tinambac Police
Station.

Issue: Whether or not true and correct name Abelardo Martinez and not Alexander Martinez cannot
overturn the Fact of his identity being issued in the search warrant.
Ruling: Search Warrants; Discrepancy regarding the name of the accused and that stated in the search
warrant cannot militate against his positive identification by the poseur-buyer.The discrepancy
regarding the name of accused-appellant and that stated in the search warrant cannot militate against
his positive identification by the poseur-buyer. It has been consistently held that greater weight is given
to the positive identification of the accused by the prosecution witnesses than accuseds denial
concerning the commission of the crime

ISSUE: Whether or not preliminary investigation is necessary in determination of probable cause?

In Criminal Case No. 9618, appellant was convicted for selling one(1) deck of shabu and was
accordingly sentenced to suffer life imprisonment and to P20.000.00 fine. In tha case of People vs.
Simon(G.R.No. 93028, July 29, 1994) this court gave retroactive application to the provisions of R.A.
6425 which was further amended by R.A. 7659 as follows:
sec. 20. Application of penalties, confiscation and forfeiture of the proceeds of instruments of the
crime.-the penalties for offensed/s under section 3, 4, 7, 8 and 9 of article II and section 14, 14-A , 15
and 16 of article III of this Act shall be applied if the dangerous drugs involves is in any of the following
quantities:
3. 200 grams or more of shabu or methylamphetamine hydrochloride.
Otherwise, if the quantity involves is less than the foregoing quantities, the penalty shall range from
prision correctional to reclusion perpetua depending upon the quantity.
given the fact that what is involved is less 200grams of shabu and there being no mitigating ir
aggravating circumtances, the proper imposible penalty to be imposed upon appellant is prisiom
correctional in its medium period without fine.
appellant is hereby sentenced to suffer imprisonment from Six (6) months of arresto mayor, as
minimum, two(2) years and four (4) months of prision correctional as maximum. The fine of P20.000.00
is deleted.

HELD: A preliminary investigation serves not only the purposes of the State. More important, it is a part
of the guarantees of freedom and fair play which are birthrights of all who live in our country. It is,
therefore, imperative upon the fiscal or the judge as the case may be, to relieve the accused from the
pain of going through a trial once it is ascertained that the evidence is insufficient to sustain a prima
facie case or that no probable cause exists to form a sufficient belief as to the guilt of the accused.
Although there is no general formula or fixed rule for the determination of probable cause since the
same must be decided in the light of the conditions obtaining in given situations and its existence
depends to a large degree upon the finding or opinion of the judge conducting the examination, such a
finding should not disregard the facts before the judge nor run counter to the clear dictates of reasons
It is evident that there was no preliminary investigation conducted. Respondent judge did not
personally examine the complainant and her witnesses by asking searching questions and answers to
satisfy himself of the existence of probable cause as mandated by law. A person presiding over a court
of law must not only apply the law but must live and abide by it and render justice at all times without
resorting to shortcuts clearly uncalled for.
For all the foregoing, we find respondent judge guilty of ignorance of the law. WHEREFORE the Court
resolves to hold respondent Judge Lucio P. Palaypayon administratively liable for gross ignorance of
the law and to accordingly impose on him the fine of Ten Thousand Pesos (10,000.00) with a STERN
WARNING that subsequent commission of the same or similar acts in the future will be dealt with more
severely.

40

Webb v. De Leon 247 SCRA 652 florendo


3. NO. There is no merit in this contention because petitioners were given all the opportunities
to be heard. The DOJ Panel precisely ed the parties to adduce more evidence in their behalf and for the
panel to study the evidence submitted more fully.

FACTS: On June 19, 1994, the National Bureau of Investigation (NBI) filed with the Department of
Justice a letter-complaint charging petitioners Hubert Webb, Michael Gatchalian, Antonio J. Lejano and
six (6) other persons with the crime of Rape and Homicide. Forthwith, the Department of Justice
formed a panel of prosecutors headed by Assistant Chief State Prosecutor Jovencio R. Zuo to conduct
the preliminary investigation of those charged with the rape and killing on June 30, 1991 of Carmela N.
Vizconde, her mother Estrellita Nicolas-Vizconde, and her sister Anne Marie Jennifer in their home at
Number
80
W.
Vinzons,
St.,
BF
Homes,
Paraaque,
Metro
Manila.

4. NO. Petitioners argument lacks appeal for it lies on the faulty assumption that the decision
whom to prosecute is a judicial function, the sole prerogative of the courts and beyond executive and
legislative interference. In truth, the prosecution of crimes appertains to the executive department of
government whose principal power and responsibility is to see that our laws are faithfully executed. A
necessary component of this power is the right to prosecute their violators (See R.A. No. 6981 and
section 9 of Rule 119 for legal basis).
The petitions are dismissed for lack of showing of grave abuse of discretion on the part of the
respondents.

On August 8, 1995, the DOJ Panel issued a 26-page Resolution finding probable cause to
hold respondents for trial and recommending that Information for rape with homicide be filed against
petitioners and their co-respondents. On the same date, it filed the corresponding Information against
petitioners and their co-accused with the Regional Trial Court of Paraaque.

People v. Woolcock 244 SCRA 235 bagasina

FACTS:In an information dated November 11, 1992, herein accused-appellants Deborah Woolcock,
George Williams, Jacqueline Ann Daughtry and Evans Asare were indicted for violating Section 4,
Article 11 of Republic Act No. 6425, otherwise known as the Dangerous Drugs Act of 1972, as
amended, before the Regional Trial Court of Pasay City, Branch 113 by allegedly conspiring with each
other in delivering, giving away, distributing, dispatching in transit or transporting approximately
3,328.58 grams* of heroin in Pasay City and other parts of Metro Manila, on or about the third week of
October, 1992, without authority of law, 1
On December 1, 1992, appellants filed a motion to quash on the ground of duplicity of offenses
charged, 2 namely, of delivering, giving away, distributing, dispatching in transit or transporting heroin
under- Section 4, and of conspiracy under Section 21 of the aforementioned law. The court a quo
denied the motion in an order dated December 15, 1992, ruling that there is but one offense charged,
but it nevertheless ordered the state prosecutors to amend the information. 3
At the arraignment on December 17, I992, appellants, duly assisted by counsel de parte, pleaded not
guilty to the offense charged, hence trial on the merits proceeded. 4
Subsequently, on February 26, 1993, appellants filed a motion to quash the search warrants before
Judge Antonio J. Fineza of the Regional Trial Court of Kalookan City, Branch 131, who had issued the
same, contending that said warrants were issued without probable cause and only for the purpose of
fishing for evidence. In an order dated March 24, 1993, Judge Fineza denied the motion, holding that
the same should be resolved by the Pasay court where the principal case was pending. 5 Under date of
March 26, 1993 appellants filed another motion to quash the warrants before the latter court on the
same grounds. 6
On May 31, 1993, the court below, through Judge Baltazar Relativo Dizon, without categorically ruling
on said motion rendered a judgment of conviction and sentenced each of the appellants to suffer
imprisonment for life and to pay a fine of P20,000.00. The trial court likewise ordered that the heroin be
confiscated in favor of the Government and to be ultimately destroyed. 7

Respondent Judge Raul de Leon and, later, respondent Judge Amelita Tolentino issued
warrants of arrest against them without conducting the required preliminary examination.
On August 11, 1995, petitioner Webb voluntarily surrendered to the police authorities at
Camp Ricardo Papa, Sr., in Bicutan, Taguig. Petitioners Gatchalian and Lejano likewise gave
themselves up to the authorities after filing their petitions before us.
Petitioners complain about the denial of their constitutional right to due process and violation of
their right to an impartial investigation. They also assail the prejudicial publicity that attended their
preliminary investigation.
ISSUES: 1. Whether or not the DOJ Panel likewise gravely abused its discretion in holding that there is
probable cause to charge them with the crime of rape and homicide
2. Whether or not respondent Judges de Leon and Tolentino gravely abused their discretion
when they failed to conduct a preliminary examination before issuing warrants of arrest against them
3. Whether or not the DOJ Panel denied them their constitutional right to due process during
their preliminary investigation
4. Whether or not the DOJ Panel unlawfully intruded into judicial prerogative when it failed to
charge Jessica Alfaro in the information as an accused.
HELD: 1. NO. The Court ruled that the DOJ Panel did not gravely abuse its discretion when it found
probable cause against the petitioners. A probable cause needs only to rest on evidence showing that
more likely than not, a crime has been committed and was committed by the suspects. Probable cause
need not be based on clear and convincing evidence of guilt, neither on evidence establishing guilt
beyond reasonable doubt and definitely, not on evidence establishing absolute certainty of guilt.

ISSUE:Whether or not the trial court erred


1. In not resolving appellants' motion to quash the search warrants;
2. In finding the existence of conspiracy among all appellants;
3. In exercising jurisdiction over the persons of appellants Daughtry and Asare although the confiscated
heroin was found in their respective hotel rooms at the Aloha Hotel and Manila Tourist Hotel both
located in Manila;
4. In not concluding that, appellant Woolcock was subjected to an illegal arrest when ownership of the
black bag which allegedly contained the confiscated heroin was not established to be hers; and
5. In according credence to the testimonies of the police officers. 25
Indeed the essence of appellants' grievances and the thrust of their arguments before this Court

2. NO. The Court ruled that respondent judges did not gravely abuse their discretion. In arrest
cases, there must be a probable cause that a crime has been committed and that the person to be
arrested committed it. Section 6 of Rule 112 simply provides that upon filing of information, the
Regional Trial Court may issue a warrant for the accused. Clearly the, our laws repudiate the
submission of petitioners that respondent judges should have conducted searching examination of
witnesses before issuing warrants of arrest against them.

41

revolve around or are directed at the foregoing questions as the materia in exitu, the resolution whereof
shall be the determinants of the primordial dispute

FACTS: Before us is a petition for review on certiorari of the decision of the Court of Appeals
promulgated on July 22, 1992 and its resolution of May 10, 1993 denying petitioners motion for
reconsideration, both of which sustained the order of the Regional Trial Court, Branch 133, Makati,
Metro Manila, dated November 22, 1988 for the quashal of Search Warrant No. 87-053 earlier issued
per its own order on September 5, 1988 for violation of Section 56 of Presidential Decree No. 49, as
amended, otherwise known as the Decree on the Protection of Intellectual Property.
Petitioner lodged a formal complaint with the NBI for violation of PD49, as amended. Agents
of the NBI made discreet surveillance on various video establishments in Metro Manila, including
private respondent. An application for search warrant was undertaken by NBI, whose affidavits
and depositions were taken. These were corroborated by two other witnesses. The search warrant
sought was duly issued and a search was conducted by the NBI and in the course of the search; they
found and seized various video tapes of duly copyrighted films of petitioner as well as equipment and
materials. A Motion to Lift Search Warrant was filed but was later denied. Thereafter, a motion
for reconsideration of the Order was filed. The court granted the said motion and justified it on the
ground that the master tapes of the copyrighted films were never presented. At appeal, the CA
sustained the decision of the trial court.

HELD:Essentially, the over-all testimony of the police officers clearly established the guilt of appellants
for the offense charged. The cross-examination conducted by appellants to filter (the) prosecution's
evidence failed to even punctuate any degree of unreliability and incredulity. All police officers who
testified during such time had remarkably shown all indications of regularity in their respective functions.
Moreover, even if the testimonies of police officers Eric Reyes, John Campos and Mellie Meligrito were
disregarded for their alledged unreliability and unworth, the same would not cause any serious dent on
the prosecution evidence. The testimonies of the remaining police officers would still sufficiently stand
to convict the appellants. Besides, the testimonies of police officers Reyes, Campos and Meligrito are
merely corroborative evidence.
With regard to the trial judge's alledged gross incompetence and ignorance of the law, suffice it to state
that whatever judgment he has rendered which caused such attribution to him is totally unconnected
with the present case. Appellants cannot visibly gain an advantage in this instance.
WHEREFORE, the appealed judgment of the Regional Trial Court, Branch 113, Pasay City in Criminal
Case No. 92-1912 is hereby AFFIRMED in toto, with costs against accused appellants.
SO ORDERED

ISSUE: What is needed in order to show probable cause of infringement of copyright by renting
agencies of cassette tapes and whether or not the presentation of master tapes required?

Tambasen vs. People 246 SCRA 184 Isnani


HELD: The court ruled that it in not at all the time, that the presentation of master tapes is not a sine
qua non requirement for the issuance of a search warrant in video piracy cases, provided that there is
no doubt as to the true nexus between the master tape and the pirated copies. It was held that the
presentation of the master tapes of the copyrighted films should, at most, be understood to merely
serve as a guidepost in determining the existence of probable cause in copyright cases.

Facts: In August 1988, P/Sgt. Natuel applied for issuance of search warrant alleging that he
received information that Petitioner had in his possession at his house M-16 Armalite rifles,
hand grenades, .45 Cal. pistols, dynamite sticks and subversive documents, which were used
or intended to be used for illegal purposes. The application was granted.
In September, a police team, searched the house of petitioner and seized 2 envelopes
containing P14000, handset with antennae, transceiver with antennae, regulator supply,
academy notebook and assorted papers and handset battery pack. In October, petitioner moved
that the search and seizure be declared illegal and that the seized articles be returned to him. In
December, MTCC, in its order, directed Lt. Col. Torres to return the money seized to petitioner ruling
that any seizure should be limited to the specified items covered thereby. SolGen petitioned with the
RTC for the annulment of the order of MTCC citing that pending the determination of legality of seizure
of the articles, they should remain in custogia legis. RTC granted the petition.

Facts
On 07 April 1988, the National Bureau of Investigation ("NBI"), through its Agent Lauro C.
Reyes,
filed
with
the
Regional
Trial
Court
of
Pasig
(Branch 159) three applications for search warrant against private respondents Tube Video Enterprises
and Edward C. Cham (ASW No. 95), the Blooming Rose Tape Center and Ma. Jajorie T. Uy (ASW No.
96),
and
the
Video
Channel and Lydia Nabong (ASW No. 97), charging said respondents with violation of Section 56 of
Presidential Decree ("P.D.") No. 49, otherwise known as the Decree on the Protection of Intellectual
Property, as amended by P.D. No. 1988

Issue: Whether or Not the seizure of the articles which were not mentioned in the search warrant was
legal.
Held: Section 2 Article III of the 1987 Constitution requires that a search warrant should particularly
describe the things to be seized. The police acts beyond the parameters of their authority if they seize
articles not described in the search warrants. The evident purpose and intent of the requirement is to
limit the things to be seized, to leave the officers of the law with no discretion; that unreasonable search
and seizure may not be made and that abuses may not be committed.

Issue: Whether or not the NBI had authority to file the application for search warrant and whether or
not Court observed due process of law before issuing the search warrants in question.
Ruling: The presentation of the master tapes of the copyrighted films from which the pirated films were
allegedly copied, was necessary for the validity of search warrants against those who have in their
possession the pirated films.In applying for the search warrants the NBI charged violation of the entire
provisions of Section 56 of P.D. No. 49 as amended by P.D.
No. 1988. This included not only the sale, lease or distribution of pirated tapes but also the transfer or
causing to be transferred of any sound recording or motion picture or other audio visual work.
With due respect to petitioners, the Court does not see a compelling reason to reexamine its previous
position on the issue.
WHEREFORE, in view of the foregoing, the instant petitions are hereby DENIED for lack of merit.
SO ORDERED.

Petition granted. People of the Philippines is ordered to return the money seized.

20th Century Fox Film v. CA 164 SCRA 655 Cabanlong

Columbia Pictures v. CA 262 SCRA 219 Ynawat

42

judge after examination under oath or affirmation of the complainant and the witnesses he may
produce, and particularly describing the place to be searched and the persons or things to be
seized." However, this provision applies to the issuance of arrest and search warrants, which
should be distinguished from a preliminary investigation. As already stated, the determination
of probable cause for the issuance of such orders is vested in the courts, but the conduct of
preliminary investigations is entrusted to the executive branch, with the exception of inferior
court judges. According to this Court: Judges and prosecutors alike should distinguish the
preliminary inquiry which determines probable cause for the issuance of a warrant of arrest
from the preliminary investigation proper which ascertains whether the offender should be held
for trial or released The determination of probable cause for the warrant of arrest is made by the
Judge. The preliminary investigation proper whether or not there is reasonable ground to
believe that the accused is guilty of the offense charged and, therefore, whether or not he
should be subjected to the expenses, rigors and embarrassment of trial is the function of the
prosecutor. We reiterate that preliminary investigation should be distinguished as to whether it
is an investigation for the determination of a sufficient ground for the filing of the information or
it is an investigation for the determination of a probable cause for the issuance of a warrant of
arrest. The first kind of preliminary investigation is executive in nature. It is part of the
prosecution's job. The second kind of preliminary investigation which is more properly called
preliminary examination is judicial in nature and is lodged with the judge.

Ho vs. People 280 SCRA 365 Hassan

Facts: Lo Ho Wing, Peter Lo , together with co-accused Lim Cheng Huat alias Antonio Lim and
Reynaldo Tia, were charged with a violation of the Dangerous Drugs Act, for the transport of
metamphetamine hydrochloride, otherwise known as "shabu". The drug was contained in tea bags
inside tin cans which were placed inside their luggages. Upon arrival from Hongkong, they boarded the
taxis at the airport which were apprehended by CIS operatives. Their luggages were subsequently
searched where the tea bags were opened and found to contain shabu. Only Lo and Lim were
convicted. Tia was discharged as a state witness, who turned out to be a " deep penetration agent" of
the CIS in its mission to bust the drug syndicate .
Issue: W/N the search and seizure was legal.
HELD: YES That search and seizure must be supported by a valid warrant is not an absolute rule. One
of the exceptions thereto is a search of a moving vehicle. The circumstance of the case clearly show
that the serach in question was made as regards a moving vehicle.

Gozos v. Tac-an GR 123191, Dec. 17, 1998 Larete

Flores v. Sumaljag 290 SCRA 568 Jimenez

Facts:
An Administrative case was filed against Judge Antonio C. Sumaljag, Acting Presiding Judge
of Branch 5 MTC of Baybay, Leyte for gross ignorance of the law in connection with the preliminary
investigation of three criminal cases during whom the complainant in the said criminal cases, Gualberto
Parmis and his witnesses testified in each case and thereafter respondent judge ordered the arrest of
the complainants who are members of the Sangguniang Pambarangay of Domingo C. Veloso in
Baybay, Leyte, charged with three counts of falsification of public documents as defined in Art. 171,
paragraphs 2, 4 and 8 of the Revised Penal Code on the grounds that :
1. the testimonies during the preliminary examinations failed to establish probable cause;

FACTS: The officials, teachers, and students of the Concepcion Aguila Memorial College
organized a school party. While the party was going on, the principal, Felizardo Aguila, was
informed that several men, who appeared to be drunk, were trying to force their way through the
main gate. One of the men seemed armed with a handgun. After calling the police, Aguila went
to the main gate, where he asked the men what their business was. At this point, private
respondents Blanco and Atienza arrived at the school. They were shortly joined by private
respondents Pedro Castillo, Sulit, and Ildefonso Castillo, who were all members of the
Philippine National Police of San Jose, Batangas. They demanded from the man armed with the
handgun, who later turned out to be the victim Gilbert Dyogi, that he surrender his gun and go
with them to the station. Gilbert Dyogi gave the handgun to the respondents, who then asked to
see his license. He produced a sheet of paper from his wallet which he handed to private
respondents. After allowing them to inspect the weapon and the alleged license, Gilbert Dyogi
asked the private respondents to give them back to him. However, private respondent Blanco
who had the handgun refused to do so. Before long the two were grappling for possession of
the gun. Apparently, Blanco pulled out his sidearm and fired at Gilbert Dyogi twice.
Subsequently respondent judge issued the questioned warrant against herein petitioners.

2.

that in Criminal Case Nos. R-3227-A and R-3227-B, the complainant was not personally
examined by respondent, the ones who testified being only complainants witnesses

3.

that there was absence of "searching questions and answers in Criminal Case No. R3231-A

4.

that there was no cause under the law to arrest them as the possibility of fleeing to escape
the hands of justice is remote as they are barangay officials.

Respondent in his answer maintained that he observed the pertinent legal procedures before
issuing the warrants of arrest against complainants and the quantum of evidence required in
preliminary investigation had likewise been observed. He contended that by posting bail, complainants
waived objection to any irregularities which might have been committed in the course of preliminary
investigation.

ISSUE:Whether or not the issuance of the warrant is valid?


HELD:It is evident that in this case, respondent judge conducted an inquiry, not only for the
purpose of determining whether there was probable cause to order the arrest of private
respondents Blanco, Pedro Castillo, Sulit, Atienza, and Ildefonso Castillo, but for the purpose of
determining whether there was sufficient evidence to prosecute them as well. Indeed, in his
questioned orders, respondent judge did not only determine the existence of probable cause for
the issuance of warrants of arrest, but also what the charge should be and who should be
charged. In so doing, respondent judge exceeded his authority. To justify his orders, respondent
judge invokes Art. III, Sec. 2 of the 1997 Constitution, which provides that "no search warrant or
warrant of arrest shall issue except upon probable cause to be determined personally by the

Issue: Whether or not the respondent judge followed the due process before issuing the warrants of
arrest against the complainants.
Held: With regards to the claim of the herein complainants that the respondent did not ask the
complainant and the witnesses searching questions but mainly questions designed to make them
merely affirm what they had previously stated in their affidavits, which was later noted by the Deputy
Court Administrator as leading questions, the court held that if it is the only way to make the witnesses

43

who were reticent to give details was to ask questions based on their complaints and affidavits it is of
no reason why this should be improper and that as far as the duty to ask searching questions is
concerned, respondent complied with his duty under the rules of court.
The complainants also aver that the testimonies of Gualberto Parmis and the witness during
the preliminary examination failed to establish probable cause but even if the barangay officials signed
the resolution by mistake, this would not show that they did not take part in the preparation of the
spurious minutes or excerpts thereof. For the purposes of the preliminary investigation, which was
summary in nature, it was enough that there was evidence showing that the crime had been committed
and that the accused were probably guilty thereof. Any error which might have been committed at this
stage could be corrected after the case was filed in the RTC.
With regards also to the third contention of the complainant that in Criminal Cases Nos. R3227-A and R-3228-A, only the witnesses were examined by the respondent judge; that the chief of
police was the complainant but was not examined is without merit. The chief of police signed the
complaints simply as prosecutor. The complainant in the two cases was Gualberto Parmis who testified
together with Diego Cala Jr.
Finally, it is contended that respondent ordered the arrest of complainants without the
justification of doing so in order to frustrate the ends of justice, as provided in Rule 112 is meritorious.
The rule now is that the investigating judges power to order the arrest of the accused is limited to
instances in which there is a necessity for placing him in custody in order not to frustrate the ends of
justice. It appears in this case that the respondent ordered the issuance of a warrant of arrest soley on
his finding of probable cause, totally omitting to consider whether it was necessary to do so in order not
to frustrate the ends of justice.
Since respondent has retired, the only appropriate penalty that could be imposed on him, in
light of what he failed to do in this case, is a fine of P5,000, to be deducted from whatever retirement
benefits he may be entitled to receive.

warrant.
The place to be searched, as set out in the warrant, cannot be amplified or modified by the officers'
own personal knowledge of the premises, or the evidence they adduced in support of their application
for the warrant. Such a change is proscribed by the Constitution which requires inter alia the search
warrant to particularly describe the place to be searched as well as the persons or things to be seized.
It would concede to police officers the power of choosing the place to be searched, even if it not be that
delineated in the warrant. It would open wide the door to abuse of the search process, and grant to
officers executing a search warrant that discretion which the Constitution has precisely removed from
them. The particularization of the description of the place to be searched may properly be done only by
the Judge, and only in the warrant itself; it cannot be left to the discretion of the police officers

Kho v. Makalintal GR 94902-06, April 21, 1999 Tubo

Facts:On May 15, 1990, NBI Agent Max B. Salvador applied for the issuance of search warrants by the
respondent Judge against Banjamin V. Kho in his residence at No. 45 Bb. Ramona Tirona St., BF
Homes, Phase I, Paranaque. On the same day, Eduardo T. Arugay, another NBI agent, applied with
the same court for the issuance of search warrants against the said petitioner in his house at No. 326
McDivitt St., Bgy. Moonwalk, Paranaque. The search warrants were applied for after teams of NBI
agents had conducted a personal surveillance and investigation in the two houses were being used as
storage centers for unlicensed firearms and chop-chop vehicles. Respondent NBI sought for the
issuance of search warrants in anticipation of criminal cases to be instituted against Kho. On the same
day, the respondent Judge conducted the necessary examination of the applicants and their witnesses,
after which he issued Search Warrant Nos. 90-11 to 90-15. On May 16, 1990, with Search Warrant Nos.
90-11 and 90-12, NBI agents searched subject premises at BF Homes, Paranaque, and they
recovered various high-powered firearms and hundreds of rounds of ammunition. On the other hand,
using Search Warrant Nos. 90-13, 90-14 and 90-15 another team of NBI agents search was conducted
at the house at No. 326 McDivitt St. Bgy. Moonwalk, Paranaque which yielded several high-powered
firearms with explosives and more than a thousand rounds of ammunition. Aside from firearms, various
radio and telecommunication equipment, two units of motor vehicles and one motorcycle were also
found. The NBI agents found out that no license has ever been issued for the confiscated firearms and
the motor vehicles were unlicensed and unregistered. On May 28, 1990, the petitioners presented a
Motion to Quash the said Search Warrants. Petitioners sought to restrain the NBI from using the
objects seized by virtue of such warrants in any case or cases filed or to be filed against them and to
return immediately the said items. On July 26, 1990, respondent Judge denied the Motion to Quash.

People v. CA 291 SCRA 400 delatado

FACTS A petition for certiorari has been filed to invalidate the order of Judge Casanova which quashed
search warrant issued by Judge Bacalla and declared inadmissible for any purpose the items seized
under
the
warrant.
>An application for a search warrant was made by S/Insp Brillantes against Mr. Azfar Hussain who had
allegedly in his possession firearms and explosives at Abigail Variety Store, Apt 1207 Area F. Bagon
Buhay Avenue, Sarang Palay, San Jose Del Monte, Bulacan. The following day Search Warrant No.
1068 was issued but was served not at Abigail Variety Store but at Apt. No. 1, immediately adjacent to
Abigail Variety Store resulting in the arrest of 4 Pakistani nationals and the seizure of a number of
different
explosives
and
firearms.

Issue: Whether or not the warrants issued by respondent Judge are valid? Whether or not respondent
Judge committed grave abuse of discretion on denying the Motion to Quash?

ISSUE:Whether or not a search warrant was validly issued as regard the apartment in which private
respondents were then actually residing, or more explicitly, Whether or not that particular apartment had
been
specifically
described
in
the
warrant.

Decision: Yes. Records show that the NBI agents who conducted the surveillance and investigation
testified unequivocably that they saw guns being carried to and unloaded at the two houses searched,
and motor vehicles and spare parts were stored therein. Respondent judge had the singular opportunity
to assess their testimonies and to find out their personal knowledge of facts and circumstances enough
to create a probable cause. The Judge was the one who personally examined the applicants and
witnesses and who asked searching questions vis-a-vis the applications for search warrants. He was
thus able to observe and determine whether subject applicants and their witnesses gave accurate
accounts of the surveillance and investigation they conducted at the premises to be searched. He
personally examined them under oath, and asked them searching questions on the facts and
circumstances personally known to them, in compliance with prescribed procedure and legal

HELD:The ambiguity lies outside the instrument, arising from the absence of a meeting of minds as to
the place to be searched between the applicants for the warrant and the Judge issuing the same; and
what was done was to substitute for the place that the Judge had written down in the warrant, the
premises that the executing officers had in their mind. This should not have been done. It is neither fair
nor licit to allow police officers to search a place different from that stated in the warrant on the claim
that the place actually searched although not that specified in the warrant is exactly what they had
in view when they applied for the warrant and had demarcated in their supporting evidence. What is
material in determining the validity of a search is the place stated in the warrant itself, not what the
applicants had in their thoughts, or had represented in the proofs they submitted to the court issuing the

44

requirements. It was within the discretion of the examining Judge to determine what questions to ask
the witnesses so long as the questions asked are germane to the pivot of inquiry - the existence or
absence of a probable cause.
Petitioners claim that subject search warrants are general warrants proscribed by the Constitution
which accordingly the things to be seized were not described and detailed out. The Court finds it that
the said warrants comply with Constitutional and statutory requirements. The law does not require that
the things to be seized must be described in precise and minute detail as to leave no room for doubt on
the part of the searching authorities. Otherwise, it would be virtually impossible for the applicants to
obtain a warrant as they would not know exactly what kind of things they are looking for.
The resolution of denying the Motion by the respondent Judge is not within the scope of Motion to
Quash for it assails the validity of the issuance of warrant.
The petition is hereby DISMISSED.

judge after examination under oath or affirmation of the complainant and the witnesses he may
produce, and particularly describing the place to be searched and the persons or things to be seized.
In view of the manifest objective of the constitutional safeguard against unreasonable search, the
Constitution and the Rules limit the place to be searched only to those described in the warrant. Thus,
this Court has held that this constitutional right is the embodiment of a spiritual concept: the belief that
to value the privacy of home and person and to afford its constitutional protection against the long reach
of government is no less than to value human dignity, and that his privacy must not be disturbed except
in case of overriding social need, and then only under stringent procedural safeguards.
What is material in determining the validity of a search is the place stated in the warrant itself,
not what the applicants had in their thoughts, or had represented in the proofs they submitted to the
court issuing the warrant. The place to be searched, as set out in the warrant, cannot be amplified or
modified by the officers own personal knowledge of the premises, or the evidence they adduced in
support of their application for the warrant. Such a change is proscribed by the Constitution which
requires inter alia the search warrant to particularly describe the place to be searched as well as the
persons or things to be seized. It would concede to police officers the power of choosing the place to
be searched, even if it not be that delineated in the warrant. It would open wide the door to abuse of
the search process, and grant to officers executing a search warrant that discretion which the
Constitution has precisely removed from them.

Paper Industries v. Asuncion GR 122092, May 19, 1999 Fernandez

WHEREFORE, the instant petition for certiorari and prohibition is hereby GRANTED and Search
Warrant No. 799 (95) accordingly declared NULL andVOID. The temporary restraining order issued by
this Court on October 23, 1995 is hereby MADE PERMANENT. No pronouncement as to costs.

FACTS On January 25, 1995, Police Chief Inspector Napoleon B. Pascua applied for a search warrant
before the said RTC of Quezon City, staring:
1. That the management of Paper Industries Corporation of the Philippines, located at PICOP
compound, Barangay Tabon, Bislig, Surigao del Sur, represented by its Sr. Vice President
Ricardo G. Santiago, is in possession or has in its control high powered firearms, ammunitions,
explosives, which are the subject of the offense, or used or intended to be used in committing
the offense, and which are being kept and concealed in the premises herein described.

Malalaon v. CA 232 SCRA 249 San Luis

On February 4, 1995, the police enforced the search warrant at the PICOP compound and seized some
firearms, ammunitions and explosives.

FACTS: The respondent Court of Appeals rendered judgment, in effect affirming that of the trail court,
by denying due course to the petition for certiorari and lifting the temporary restraining order it had
issued on November 29, 1990 in connection therewith. The judgment of respondent court is now
impugned in and sought to be reversed through the present recourse before us. The crime alleged is a
violation of PD 1866 (Illegal Possession of Firearms and Ammunitions). The officers applied for a
search warrant in Caloocan City. However, the place to be searched was in Quezon City. The officers
executed the search and seized several firearms. After the search and seizure was conducted, the
accused are now assailing the validity of the search warrant since they claim that it was void for lack of
jurisdiction. The accused buttresses their claim arguing that the criminal case was filed in the Quezon
City RTC, not in Caloocan City.

ISSUE: Whether or not the search warrant is valid.

ISSUE: Whether or not the search warrant was valid.

2. That a Search Warrant should be issued to enable any agent of the law to take possession
and bring to this Honorable Court.
After propounding several questions to Bacolod, Judge Maximiano C. Asuncion issued the contested
search warrant, the pertinent portion

HELD
No. The fundamental right against unreasonable searches and seizures and the basic
conditions for the issuance of a search warrant are laid down in Section 2, Article III of the 1987
Constitution, which reads:

HELD: Yes, the search warrant was valid. A warrant merely constitutes criminal process. It is not a
criminal action. The requisites, procedure and purpose for the issuance of a search warrant are
completely different from those for the institution of a criminal action. All grounds and objections then
available, existent or known shall be raised in the original or subsequent proceedings for the quashal of
the warrant, otherwise they shall be deemed waived. These guidelines shall likewise be observed
where the same criminal offense is charged in different informations or complaints and filed in two or
more courts with concurrent original jurisdiction over the criminal action. Moreover, no law or rule

The right of the people to be secure in their persons, houses, papers and effects against unreasonable
searches and seizures of whatever nature and for any purpose shall be inviolable, and no search
warrant or warrant of arrest shall issue except upon probable cause to be determined personally by the

45

imposes such a limitation on search warrants, in the same manner that no such restriction is provided
for warrants of arrest.

documents of UPC. A return of said search was duly made by Labaria with the RTC of Cebu. UPC filed
a motion to quash the warrants which was denied by the RTC. They appealed before the CA via
certiorari. The CA dismissed the appeal for a certiorari is not the proper remedy.

WHEREFORE, on the foregoing premises, the instant petition is DENIED and the assailed judgment of
respondent Court of Appeals herby AFFIRMED.

ISSUE: Whether or not there was a valid search warrant issued.

SO ORDERED.

2.

HELD: The SC ruled in favor of UPC and Uy in a way for it ordered the return of the seized items but
sustained the validity of the warrant. The SC ruled that the search warrant issued has not met some
basic requisites of validity. A search warrant must conform strictly to the requirements of the foregoing
constitutional and statutory provisions. These requirements, in outline form, are:

Particularity of Description
Yousef Al-Ghoul v. CA GR 126859, Sept. 4, 2001 Hussin

(1) the warrant must be issued upon probable cause;

Facts: Petitioners assail the decision dated September 30, 1996, of the Court of Appeals, which
affirmed the orders of the Regional Trial Court of Kalookan City, Branch 123, thereby dismissing
petitioners special civil action for certiorari.
On March 31, 1995, Judge Geronimo S. Mangay, presiding judge of the Regional Trial Court, National
Capital Judicial Region, Branch 125, Kalookan City, issued search warrants for the search and seizure
of certain items in Apartment No. 2 at 154 Obiniana Compound, Deparo Road, Kalookan City.
On April 1, 1995, the police searched Apartment No. 8, in the same compound and found one (1) .45
caliber pistol. Found in Apartment No. 2 were various guns, ammunitions and bomb pharapernalias.

(2) the probable cause must be determined by the judge himself and not by the applicant or any other
person;
(3) in the determination of probable cause, the judge must examine, under oath or affirmation, the
complainant and such witnesses as the latter may produce; and
(4) the warrant issued must particularly describe the place to be searched and persons or things to be
seized.

Issue: The issue that remains is whether the respondent court erred and gravely abused its discretion
when it ruled that the search and seizure orders in question are valid?

The SC noted that there has been inconsistencies in the description of the place to be searched as
indicated in the said warrants. Also the thing to be seized was not clearly defined by the judge. He used
generic itineraries. The warrants were also inconsistent as to who should be searched. One warrant
was directed only against Uy and the other was against Uy and UPC. The SC however noted that the
inconsistencies wered cured by the issuance of the latter warrant as it has revoked the two others.

Held: Searches and Seizure; The place to be searched cannot be changed, enlarged nor amplified by
the police.As held in PICOP v. Asuncion, the place to be searched cannot be changed, enlarged nor
amplified by the police. Policemen may not be restrained from pursuing their task with vigor, but in
doing so, care must be taken that constitutional and legal safeguards are not disregarded. Exclusion of
unlawfully seized evidence is the only practical means of enforcing the constitutional injunction against
unreasonable searches and seizures. Hence, we are constrained to declare that the search made at
Apartment No. 8 is illegal and the .45 caliber pistol taken thereat is inadmissible in evidence against
petitioners.

Section 2, Article III of the Constitution guarantees the right of the people against unreasonable
searches and seizures:
The right of the people to be secure in their persons, houses, papers, and effects against unreasonable
searches and seizures of whatever nature and for any purpose shall be inviolable, and no search
warrant or warrant of arrest shall issue except upon probable cause to be determined personally by the
judge after examination under oath or affirmation of the complainant and the witnesses he may
produce, and particularly describing the place to be searched and the persons or things to be seized.

Uy v. BIR 344 SCRA 36 - Aripin

Search and Seizure Requisites of a Valid Search Warrant


FACTS: In Sept 1993, Rodrigo Abos, a former employee of UPC reported to the BIR that Uy Chin Ho
aka Frank Uy, manager of UPC, was selling thousands of cartons of canned cartons without issuing a
report. This is a violation of Sec 253 & 263 of the Internal Revenue Code. In Oct 1993, the BIR
requested before RTC Cebu to issue a search warrant. Judge Gozo-Dadole issued a warrant on the
same day. A second warrant was issued which contains the same substance but has only one page, the
same was dated Oct 1st 2003. These warrants were issued for the alleged violation by Uy of Sec 253. A
third warrant was issued on the same day for the alleged violation of Uy of Sec 238 in relation to sec
263. On the strength of these warrants, agents of the BIR, accompanied by members of the PNP, on 2
Oct 1993, searched the premises of the UPC. They seized, among other things, the records and

3.

Warrantless Searches and Seizures


When is a search a search?

46

Valmonte v. General de Villa 178 SCRA 211 (Main) and 185 SCRA 655 (MR) Solatorio

banning, was dismissed.


Petitioners have filed the instant motion and supplemental motion for
reconsideration of said decision.

Facts:This is a petition for prohibition with preliminary injunction and/or


temporary restraining order, seeking the declaration of checkpoints in
Valenzuela, Metro Manila or elsewhere, as unconstitutional and the dismantling
and banning of the same or, in the alternative, to direct the respondents to
formulate guidelines in the implementation of checkpoints, for the protection of
the people.
Petitioner Ricardo C. Valmonte sues in his capacity as citizen of the Republic,
taxpayer, member of the Integrated Bar of the Philippines (IBP), and resident of
Valenzuela, Metro Manila; while petitioner Union of Lawyers and Advocates for
Peoples Rights (ULAP) sues in its capacity as an association whose members
are all members of the IBP.
On 20 January 1987, the National Capital Region District Command (NCRDC) was
activated pursuant to Letter of Instruction 02/87 of the Philippine General
Headquarters, AFP, with the mission of conducting security operations within its
area of responsibility and peripheral areas As part of its duty to maintain peace
and order, the NCRDC installed checkpoints in various parts of Valenzuela, Metro
Manila.
Petitioners aver that, because of the installation of said checkpoints, the
residents of Valenzuela are worried of being harassed and of their safety being
placed at the arbitrary, capricious and whimsical disposition of the military
manning the checkpoints, considering that their cars and vehicles are being
subjected to regular searches and check-ups, especially at night or at dawn,
without the benefit of a search warrant and/or court order. Their alleged fear for
their safety increased when, at dawn of 9 July 1988, Benjamin Parpon, a supply
officer of Municipality of Valenzuela, Bulacan, was gunned down allegedly in
cold blood by the members of the NCRDC manning the checkpoint along
McArthur Highway at Malinta, Valenzuela, for ignoring and/or refusing to submit
himself to the checkpoint and for continuing to speed off inspite of warning
shots fired in the air. Petitioner Valmonte also claims that, on several occasions,
he had gone thru these checkpoints where he was stopped and his car subjected
to search/check-up without a court order or search warrant.
Petitioners further contend that the said checkpoints give the respondents a
blanket authority to make searches and/or seizures without search warrant or
court order in violation of the Constitution;2 and, instances have occurred where
a citizen, while not killed, had been harassed.

Issue: Whether or not checkpoints are illegal.

Held:It should be stated, at the outset, that nowhere in the questioned decision did this Court
legalize all checkpoints, i.e. at all times and under all circumstances. What the
Court declared is, that checkpoints are not illegal per se. Thus, under
exceptional circumstances, as where the survival of organized government is on
the balance, or where the lives and safety of the people are in grave peril,
checkpoints may be allowed and installed by the government. Routine
inspection and a few questions do noty constitute unreasonable searches. If the
inspection becomes more thorough to the extent of becoming a search, this can
be done when there is deemed to be probable cause. In the latter situation, it is
justifiable as a warrantless search of a moving vehicle.

Guazon v. De Villa 181 SCRA 623 Sing


FACTS:This is a petition for prohibition with preliminary injunction to prohibit military represented by the
respondents from conducting target zoning or saturation drives in Metro Manila. The 41 petitioners
stated that they are of legal age, bona fide residents and taxpayers including leader of their respective
community.They maintain that they have a common or general interest in the preservation of the rule of
law, protection of their human rights and the reign of peace and order in their communities. In doing so
they claim that they are the representative of the citizens of Metro Manila. Who according to them
shares the same interest as they do. Petitioners claim that the saturation drives are in critical areas,
petitioners claim that saturation drives create human right violation. Respondents gave 2 reasons. First
is that drive has legal authority to do so and the petitioners accusations are lies. Respondent site
article VII , Sec 17 and 18 of the constitution which state the president shall have control of all the
executive departments, bureaus and offices. He shall ensure that the laws are faithfully observed. And
the president shall be the commander-in-chief of all and whenever it becomes necessary the president
make necessary actions. He can call out AFP to prevent or suppress the lawless violence or invasion
as a result of rebellion.

Issue: Whether or not checkpoints are constitutional.


ISSUE: Whether or not unreasonable search and seizures is valid?
HELD: As a general rule, a peace officer can not act unless he is possessed of the proper arrest or
search warrant. The exception is when a criminal offense is unfolding before him, in which case, action
is justified and necessary. The majority would have the exception to be simply, the general rule.

Held:Checkpoints during abnormal times, if conducted within reasonable limits, are


constitutional. Not all searches and seizures are prohibited. Those which are
reasonable are not forbidden. A reasonable search is not to be determined by
any fixed formula but is to be resolved according to the facts of each case.

Valmonte v. General de Villa - 185 SCRA 655 (MR)

The fact of the matter is that we are not here confronted by police officers on the beat or prowl cars on
patrol. What we haveand I suppose that everybody is agreed on itare lightning raids of homes,
arbitrary confiscation of effects, and summary arrests of persons, the very acts proscribed by the
Constitution. If this is a show of force, it certainly has no place in a constitutional democracy.

In the Courts decision dated 29 September 1989, petitioners petition for prohibition seeking the
declaration of the checkpoints as unconstitutional and their dismantling and/or

I find allusions to the last aborted coup detat inapt. In that case, our men in uniform had all the right to
act amidst crimes being committed in flagrante. The instant case is quite different. There are no
offenses being committed, but rather, police officers fishing for evidence of offenses that may have

47

been committed. As I said, in that event, a court warrant is indispensable. [Guazon vs. De Villa, 181
SCRA 623(1990)]

was not enough evidence to controvert the testimonies of respondents and the narration of claimant
Antonio del Rosario, the Collector of Customs issued his decision in the seizure cases on April 1, 1975
declaring that the seized articles including the car are not subject of forfeiture. Meanwhile, on March 14,
1974, after the requisite preliminary investigation, the City Fiscal of Quezon City, finding the existence
of a prima facie case against respondents Hope and Medina, filed Criminal Case No. Q-3781 in the
Court of First Instance of Rizal (Quezon City). Upon arraignment on April 23, 1974, respondents
pleaded not guilty. Trial commenced on January 28, 1975 and while the prosecution through its first
witness, Agent Macario Sabado, was adducing as evidence the pictures of the eleven (11) boxes
containing the assorted watches and watch bracelets, counsel for respondents objected to the
presentation of the pictures and the subject articles on the ground that they were seized without the
benefit of warrant, and therefore inadmissible in evidence under Section 4(2), Article IV of the New
Constitution. After the parties have argued their grounds in their respective memoranda, respondent
trial court issued the questioned order of August 20, 1975 as cited earlier. The prosecutions motion for
reconsideration was denied on September 30, 1975. Hence, this petition which was treated as a special
civil action in Our Resolution of May 5, 1976.

WHEREFORE, the petition is DISMISSED for lack of merit. No pronouncement as to costs.

People v. Saycon 236 SCRA 329 Jaafar

Facts: On or about 8 July 1992, at about 6:00 in the morning, the Coastguard personnel received
information from NARCOM agent Ruben Laddaran that a suspected "shabu" courier by the name of
Alvaro Saycon was on board the MV Doa Virginia, which was arriving at that moment in Dumaguete
City. Upon receipt of the information, the Coastguard chief officer CPO Tolin, instructed them to
intercept the suspect. A combined team of NARCOM agents and Philippine Coastguard personnel
consisting of CPO Tolin, a certain Miagme, and Senior Police Officers Ruben Laddaran and Winifredo
Noble of NARCOM posted themselves at the gate of Pier 1. The MV Doa Virginia docked at 6:00 a.m.
that same morning at Pier 1 in Dumaguete City. Alvaro Saycon alighted from the boat carrying a black
bag and went through the checkpoint manned by the Philippine Coastguard where he was identified by
police officer Winifredo Noble of NARCOM. Saycon was then invited to the Coastguard Headquarters at
the Pier area. He willingly went with them. At the headquarters, the coastguard asked Saycon to open
his bag, and the latter willingly obliged. In it were personal belongings and a maong wallet. Inside that
maong wallet, there was a Marlboro pack containing the suspected "shabu". When police officer
Winifredo Noble asked Saycon whether the Marlboro pack containing the suspected "shabu" was his,
Saycon merely bowed his head. Then Saycon, his bag and the suspected "shabu" were brought to the
NARCOM office for booking. When Alvaro Saycon was arrested, the NARCOM agents did not have a
warrant of arrest. The PNP's Forensic Analyst declared in court that she had conducted an examination
of the specimens and found out that the specimens weighed 4.2 grams in total, consisted of
methamphetamine
hydrochloride,
more
widely
known
as
"shabu."
Issue:

Whether

or

Not

the

warrantless

search

was

ISSUE: Whether or not warrantless search and seizures by customs valid?


HELD: Persons exercising authority under the customs law may effect search and seizure
without a search warrant.Aware of this delineation, the Court in that case expressed the
considered view that except in the case of the search of a dwelling house, persons exercising
police authority under the customs law may effect search and seizure without a search warrant
in the enforcement of customs laws. [People vs. CFI of Rizal, Br. IX, 101 SCRA 86(1980)]
The exclusive jurisdiction in seizure and forfeiture cases vested in the Collector of Customs precludes a
court of first instance from assuming authority over such matter. [People vs. CFI of Rizal, Br. IX, 101
SCRA 86(1980)]

Roan v. Gonzales 145 SCRA 687 - Falcatan

Facts: Petitioner claims he was the victim of an illegal search and seizure conducted by the military
authorities. Articles seized from him are to be used as evidence in his prosecution of illegal possession
of firearms. The challenged search warrant was issued by the judge on May 10, 1984. Petitioners
house was searched two days later but none of the articles listed in the warrant was discovered.

valid.

Issue: Whether or not the challenged search warrant issued by respondent judge violates the
Constitution.

Held: The warrantless search was valid, as the accused was a passenger of a motor vehicle. There
was probable cause to believe that the accused was carrying prohibited drugs. Three weeks earlier,
agents of the Narcotics Command bought methamine hydrochloride from him. An agent of the
Narcotics Command reported that the accused would be arriving on board the vessel and carrying
methamphetamine hydrochloride with him. Drug couriers do not go about their trade with some external
sign indicating that they are transporting prohibited drugs. This must be taken into account in
determining probable cause.

Held: Yes, respondent judge limited his inquiry on complainants affidavit only. All he did was question
Captain Quillosa on the contents of his affidavit only. He did not take the applicants deposition in writing
and attach them to the record, together with the affidavit presented to him. In other words, the applicant
was asking for the issuance of the search warrant on the basis of mere hearsay and not of information
personally known to him, as required by settled jurisprudence. His application, standing alone, was
insufficient to justify the issuance of the warrant sought. The pistol and bullets cannot, of course, be
used as evidence against the petitioner in the criminal action against him for illegal possession of
firearms. Wherefore, Search Warrant issued by the respondent judge on May 10, 1984, is hereby
declared null and void and accordingly set aside.

People v. CFI 101 SCRA 86 Cedric Alabata


FACTS: On February 9, 1974 at 6:45 am, the Regional Anti-Smuggling Action Center (RASAC)
apprehended Sgt. Jessie Hope and his girlfriend Monina Medina, aboard their Light blue Dodge car
base from an intel they received 1 week before that shipment of highly dutiable goods would be
transported to Manila from Angeles City on a blue Dodge car. The arrest was warrantless. Due to there

Nolasco v. Mago GR 27360 Sing

48

Facts:This is an original action for prohibition and certiorari, with preliminary injunction filed by Ricardo
Papa, Chief of Police of Manila; Juan once Enrile, Commissioner of Customs; Pedro Pacis, Collector of
Customs of the Port of Manila; and Martin Alagao, a patrolman of the Manila Police Department,
against RemediosMago and Hon. HilarionJarencio, Presiding Judge of Branch 23 of the Court of First
Instance of Manila, praying for the annulment of the order issued by respondent Judge in Civil Case
No. 67496 of the Court of First Instance of Manila under date of March 7, 1967, which authorized the
release under bond of certain goods which were seized and held by petitioners in connection with the
enforcement of the Tariff and Customs Code, but which were claimed by respondent RemediosMago,
and to prohibit respondent Judge from further proceeding in any manner whatsoever in said Civil Case
No. 67496. Pending the determination of this case this Court issued a writ of preliminary injunction
restraining the respondent Judge from executing, enforcing and/or implementing the questioned order
in Civil Case No. 67496 and from proceeding with said case.1

Held The bureau of custom has jurisdiction over it, they can do searches even without a warrant, it is
written in their job description. Like Martin Alagao, who was appointed as a custom agent therefore he
cannot he be charge for a warrantless search because as a custom agent they are exempted from
having so. The Bureau of Customs had acquired jurisdiction over the goods for the purpose of the
enforcement of the customs and tariff laws, to the exclusion of the Court of First Instance of Manila, We
have thus resolved the principal and decisive issue in the present case.
WHEREFORE, judgment is hereby rendered, as follows:Granting the writ of certiorari and
prohibition prayed for by petitioners 3

Martin Alagao, head of the counter intelligence of Manila Unit Department, on November 3,
1966 acted with reliable intelligence to the effect that a certain shipment of personal effect, which is
alleged not declared and would be released the following day in the custom zone at the port of Manila
Port, which was loaded to the two trucks.Under the leading of the Petitioner Ricardo Papa, chief of
Police Manila Department, and duly deputized agent of customs and was also participate in custom
operations.

People v. Lo Ho Wing 193 SCRA 122 Jaafar

Facts : Peter Lo , together with co-accused Lim Cheng Huat alias Antonio Lim and Reynaldo Tia, were
charged with a violation of the Dangerous Drugs Act, for the transport of metamphetamine
hydrochloride, otherwise known as "shabu". The drug was contained in tea bags inside tin cans which
were placed inside their luggages. Upon arrival from Hongkong, they boarded the taxis at the airport
which were apprehended by CIS operatives. Their luggages were subsequently searched where the tea
bags were opened and found to contain shabu. Only Lo and Lim were convicted. Tia was discharged as
a state witness, who turned out to be a " deep penetration agent" of the CIS in its mission to bust the
drug syndicate .

November 4, 1966 unit from counter- intelligence went after the truck and inspected it at
around 4:00pm in Agrifina Circle, Ermita, Manila. The two trucks was loaded with 9 bales of goods. It
was seized by order of the chief police. Upon investigation, a person claim ownership and showed
statement of a receipt of duties collected upon entry no. 147-1501. Issued by custom in the name of
BienvenidoNaguit.

Issue: W/N the search and seizure was legal.


HELD: YES, that search and seizure must be supported by a valid warrant is not an absolute rule. One
of the exceptions thereto is a search of a moving vehicle. The circumstance of the case clearly shows
that the search in question was made as regards a moving vehicle.

Mago and Lanopa in the court of First Instance filed for mandamus and restraining order as
preliminary injunction. Mago was owner of the goods seized, which she purchased at Sta. Monica
Grocery in San Fernando, Pampanga. According to her goods was seized without a search warrant.
November 10, 1966 Judge Jarencio order an ex parte restraining the Civil Case no. 6749

Therefore, a valid warrant was not necessary to effect the search on appellant and his co-accused. It
was firmly established from the factual findings of the court that the authorities had reasonable ground
to believe that appellant would attempt to bring in contraband and transport within the country. The
belief was based on intelligence reports gathered from surveillance activities on the suspected
syndicate, of which appellant was touted to be amember. Aside from this, they were also certain as to
the expected date and time of arrival of the accused from China via Hongkong. But such knowledge
was insufficient to enable them to fulfill the requirements for the issuance of a search warrant. Still and
all, the important thing is that there was probable cause to conduct the warrantless search, which must
still be present in the case.

December 9, 1966 the lower court with the conformity of the parties and order the inventory
of goods is made by its clerk of the court. By 23 of December 1966 Mago filed an ex parte motion to
release the goods upon agreement between parties.
March 7, 1967 the judge respondent issued as order to release the goods with the bonds
filed the amount of P 40, 000.00
March 13, 1967 Ricardo Papa, filed a writ for reconsideration on the order of release the
goods under bond. Without waiting the court action for the motion for reconsideration, and alleging that
they had no plain, speedy and adequate remedy in the ordinary course of law, herein petitioners filed
the present action for prohibition and certiorari with preliminary injunction before this Court. In their
petition petitioners alleged, among others,that the respondent Judge acted without jurisdiction in
ordering the release to respondent RemediosMago of the disputed goods. In due time the respondent
also filed for his answer on the petion for prohibition and certiorari.

People v. Malmstedt 198 SCRA 401 Cedric Alabata


FACTS: Accused is a Swedish national arrested for carrying Hashish, a form of marijuana during a
NARCOM inspection. He was tried and found guilty in violation of Dangerous Drugs Act. He contends
that the arrest was illegal without the search warrant.

Issue:Whether or not the Bureau of Custom has jurisdiction to seize over the goods?
ISSUE: W/ N the personal effects of Malmstedt may be searched without an issued warrant.

49

day the petitioners house was search. Thinking that the gunmen took refuge at the dwelling place of
the petitioner and its outcome turns out to be fruitless.

HELD: The Constitution guarantees the right of the people to be secure in their persons, houses,
papers and effects against unreasonable searches and seizures. However, where the search is made
pursuant to a lawful arrest, there is no need to obtain a search warrant. A lawful arrest without a warrant
may be made by a peace officer or a private person under the following circumstances. Section 5
provides that a peace officer or a private person may, without a warrant, arrest a person (a) When, in
his presence, the person to be arrested has committed, is actually committing, or is attempting to
commit an offense; (b) When an offense has in fact just been committed, and he has personal
knowledge of facts indicating that the person to be arrested has committed it; and (c) When the person
to be arrested is a prisoner who has escaped from a penal establishment or place where he is serving
final judgment or temporarily confined while his case is pending, or has escaped while being transferred
from one confinement to another. In cases falling under paragraphs (a) and (b) hereof, the person
arrested without a warrant shall be forthwith delivered to the nearest police station or jail, and he shall
be proceeded against in accordance with Rule 112, Section 7. Herein, Malmstedt was caught in
flagrante delicto, when he was transporting prohibited drugs. Thus, the search made upon his personal
effects falls squarely under paragraph (1) of the foregoing provisions of law, which allow a warrantless
search incident to a lawful arrest.

2 days after the incident without warrant, together with his men from Philippine Constabulary,
Capt. JulitoRoxas, seized the petitioner's motorcycle and took it to the PC headquarters in Masbate.
Motorcycle was taken and impounded with the suspicion that it was what the gunmen used during the
assassination.After investigation, the petitioner and several others were charged with multiple murder
and frustrated murder for the killing of Espinosa and three of his bodyguards and the wounding of
another person.
July 21, 1989 the petitioner filed charges against Capt. JulitoRoxas and his men from
Philippine Constabulary and filed for petition for replevin for seeking the return of the petitioners
motorcycle and other damages of P 55, 000, with civil case number 3818 at the Regional Trial Court of
Masbate on the court of Hon. Bulatid. On which it was transfer to Criminal case under Judge Gil
Fernandez.
October 1990, it was dismissed by Judge Fernandez on the ground of lack of jurisdiction
including the reconsideration, now the petitioner seek the higher court.

Posadas v. CA 188 SCRA 288 Falcatan

Issue: Whether or not Warrantless Searches and Seizure of said motorcycle is proper?
Facts: On October 16, 1986, Pat. Urcisio Ungab and Pat. Umbra Umpar, both members of the
Integrated National Police (INP) of Davao, assigned with The Intelligence Task Force, were conducting
a surveillance. They spotted petitioner carrying a buri bag and they noticed him to be acting
suspiciously. They approached and identified themselves and petitioner attempted to flee but his
attempt was thwarted by resistance of the two. They checked the bag of petitioner where they found
one caliber .38 smith &Wesson revolver with two rounds of live ammunitions for a caliber .38 gun, a
smoke (tear gas) grenade and two live ammunitions for a .22 caliber gun. Petitioner was not able to
show any necessary license or authority to possess firearms and ammunitions found in his possession.
He was prosecuted for illegal possession of firearms and ammunitions in the RTC where he was found
guilty. Appealed to court of Appeals, court rendered decision affirming in toto decision of RTC.

Held: The necessity for the immediate seizure of the motorcycle without the prior obtention of a warrant
has not been established.The mere mobility of the motorcycle did not make the search warrant
redundant for it is not denied that the vehicle remained with the petitioner until it was forcibly taken from
him. The fear that it would be dismantled or hidden was mere speculation that was not borne out by the
facts. The extraordinary events cited in People v. Court of First Instance of Rizal are not present in the
case now before us. The necessity for the immediate seizure of the motorcycle without the prior
obtention of a warrant has not been established.
The warrantless seizure of the motorcycle was unquestionably violative of the right to be let alone by
the authorities as guaranteed by the Constitution.The warrantless seizure of the motorcycle was
unquestionably violative of the right to be let alone by the authorities as guaranteed by the
Constitution. The vehicle cannot even be detained on the ground that it is a prohibited article the mere
possession of which is unlawful.

Issue: W/N the warrantless search on the petitioner is valid.


Held: An arrest without a warrant may be effected by a peace officer or private person, among others,
when in his presence the person to be arrested has committed, is actually committing, or is attempting
to commit an offense, or when an offense has in fact just been committed, and he has personal
knowledge of the facts indicating that the person arrested has committed it. It is reasonable for an
officer rather than simply to shrug his shoulder and allow a crime to occur, to stop a suspicious
individual briefly in order to determine his identity or maintain the status quo while obtaining more
information. The constitutional guarantee against unreasonable searches has not been violated.

People v. Cuachon 238 SCRA 540 Jaafar

Bagalihog v. Fernandez 198 SCRA 614 Sing


Facts:
On November 20, 1988 the accused-appellant Romeo Cuachon Soler was charged in Crim
case no. 1118 together with Ezra King, Alberto Marniego, Jaime Sylvestre, Danilo Mumar, carols and
Michael Mcmurray with having violated sec. 27, Art. IV of R.A. 6425 ( the dangerous drug act of 1972)
as amended, before the trial court of Makati. That the accused , in conspiracy with one another willfully,
unlawfully and feloniously smoked and sniffed methyl amphetamine hydrochloride (commonly known as

Warrantless Searches and Seizures (replevin action by claimant to recover goods)


Facts: Rep. Moises Espinosa was shot dead last March 17, 1989 after just embarking at Masbate
Airport. According to the eyewitnesses the suspect/ gunmen was on board a motorcycle. That same

50

shabu) . In crim case no. 1119, accused-appellant and Ezra king were charged with violating Sec. 15 ,
Art. III of the same law. That an alleged on the same date and place the accused in conspiracy with one
another, willfully, unlawfully and feloniously sold, gave away, distributed and delivered to another methyl
amphetamine hydrochloride without corresponding license. The two informations filed at the trial court
which consequently issued warrant of arrest to all accused, but only the accused appellant was
arrested. At the arraignment on October 20, 1989 the latter pleaded not guilty to both charges,
thereafter a joint trial was held on August 16, 1991. The trail court adjudged him guilty as charged. In
crim case no. 1118 he was imposed a prison term of twelve (12) years and ordered to pay a fine of
(12,000) while in crim case no. 1119 he was sentenced to life imprisonment and fined 30,000.00.
Accused appellant pleaded for reversal of his conviction, on his version that an error subject
to the credibility of the prosecution witnesses, vis--vis on the incident which transpired on November
20, 1988 that while PFC Norman Reyes claims to have confiscated as plastic bag containing shabu
from him, this was never corroborated by any witnesses. Neither was the claim of Pat. Eduardo
Ugaddan that he bought shabu from him and Ezra king worth P150.00 corroborated by any
witnesses. There was no testimony about P50.00 change. Pat. Ugaddan was even unable to identify on
the witness stand the buy-bust money and the shabu. The policemen admitted having no warrant of
arrest or search warrant when they arrested accused-appellant and search his room. That no lawyer
assisted him during the custodial investigation especially when he was made to sign the receipt of the
property seized.

appellant. They were then made to sign an inventory, written in Tagalog, of the explosives and
ammunition confiscated by the raiding team. No search warrant was secured by the raiding team.
Accused was found guilty of illegal possession of firearms. That judgment of conviction is now
challenged before us in this appeal.

Issue/s: Whether or not the arrest of the accused and search on his house is lawful even
without warrant of arrest and search warrant?

of automobiles. The presence of an unusual quantity of high-powered firearms and explosives could not

Held: Buy-Bust Operations; In a continuing buy-bust operation, an arrest may be made even
without warrant.We find the explanation of Pat. Ugaddan acceptable if not satisfactory. In buybust
operations where the illegal sale can be witnessed by the other members of the team from strategic
positions, usually the poseur-buyer gives a pre-arranged signal to arrest the seller. In these cases,
considering that the operation took place inside the house of appellant, understandably, Pat. Ugaddan
and his informant had to get out of the house and inform their back up that the sale had been
consummated. Furthermore, a weighty consideration is the fact that, as previously stated, Pat.
Ugaddan immediately gave the shabu to Pfc. Reyes who was only a block away and after the latter had
determined that it was shabu, which only took a few minutes, they all immediately proceeded to the
house of appellant and arrested him. It was a continuing buy-bust operation which, as the phrase
connotes, commenced with buying shabu and culminated in his arrest. Since his arrest was lawful, it
follows that the incidental search was also valid. [People vs. Cuachon, 238 SCRA 540(1994)]

because of simultaneous and intense firing within the vicinity of the office and in the nearby Camp

ISSUE: W/N there was a valid search and seizure in this case.
HELD: YES It is admitted that the military operatives who raided the Eurocar Sales Office were not
armed with a search warrant at that time. The raid was actually precipitated by intelligence reports that
said office was being used as headquarters by the RAM. Prior to the raid, there was a surveillance
conducted on the premises wherein the surveillance team was fired at by a group of men coming from
the Eurocar building. When the military operatives raided the place, the occupants thereof refused to
open the door despite requests for them to do so, thereby compelling the former to break into the office.
The Eurocar Sales Office is obviously not a gun store and it is definitely not an armory or arsenal which
are the usual depositories for explosives and ammunition. It is primarily and solely engaged in the sale
be justifiably or even colorably explained. In addition, there was general chaos and disorder at that time
Aguinaldo which was under attack by rebel forces. The courts in the surrounding areas were obviously
closed and, for that matter, the building and houses therein were deserted. Under the foregoing
circumstances, it is our considered opinion that the instant case falls under one of the exceptions to the
prohibition against a warrantless search. In the first place, the military operatives, taking into account
the facts obtaining in this case, had reasonable ground to believe that a crime was being committed.
There was consequently more than sufficient probable cause to warrant their action. Furthermore,
under the situation then prevailing, the raiding team had no opportunity to apply for and secure a
search warrant from the courts. Under such urgency and exigency of the moment, a search warrant
could lawfully be dispensed with.
People v. De Lara 236 SCRA 291 Ebno Maruhom
Facts: After a surveillance was conducted by National Criminal Investigation Service (NCIS) of the
Western Police District (WPD) in the vicinity of Garrido and Zamora Streets at Sta. Ana, Manila, they
confirmed the reported drug-pushing activities in that area by the group of appellant.
On January 8, 1987, Malaya and Peoples Tonight reported that there were rampant, drug-pushing
activities in the vicinity of Garrido and Zamora Streets in Sta. Ana, Manila, prompting Gen. Alfredo Lim,
then WPD Superintendent, to reprimand the NCIS office.
On January 9, a buy-bust operation was conducted and herein appellant was arrested in his house
because of selling of Marijuana. Police confronted appellant, who admitted that he kept prohibited
drugs in his house. Appellant showed the arresting officers a blue plastic bag with white lining
containing prohibited drugs.

People v. De Garcia 233 SCRA 540 Alabata


FACTS: The incidents involved in this case took place at the height of the coup d''etat staged in
December, 1989. Accused-appellant Rolando de Gracia was charged in two separate informations for
illegal possession of ammunition and explosives in furtherance of rebellion, and for attempted homicide.
Appellant was convicted for illegal possession of firearms in furtherance of rebellion, but was acquitted
of attempted homicide. Surveillance was undertaken by the military along EDSA because of intelligence
reports about a coup. Members of the team were engaged by rebels in gunfire killing one member of
the team. A searching team raided the Eurocar Sales Office. They were able to find and confiscate six
cartons of M-16 ammunition, five bundles of C-4 dynamites, M-shells of different calibers, and "molotov"
bombs inside one of the rooms belonging to a certain Col. Matillano. De Gracia was seen inside the
office of Col. Matillano, holding a C-4 and suspiciously peeping through a door. The team arrested

51

Issue: Whether or not the arrest of the accused (appellant) and the seizure of prohibited drugs found
inside his house violated the constitutional right of the herein appellant?

People v. Barros 231 SCRA 557 Ebno Maruhom


Facts:Bonifacio Barros was charged with violating Section 4 of R.A. No. 6425, as amended (known as
the Dangerous Drugs Act of 1972), in an information which read as follows:
That on September 6, 1987, M/Sgt. Francis Yag-as and S/Sgt. James Ayan, both members of the P.C.
Mountain Province Command, rode the Dangwa Bus bearing Plate No. ABZ-242 bound for Sabangan,
Mountain Province. Upon reaching Chackchakan, Bontoc, Mountain Province, the bus stopped and
both M/Sgt. Yag-as and S/Sgt. Ayan, who were seated at the back, saw accused carrying a carton,
board the bus and seated himself on seat No. 18 after putting the carton under his seat. Thereafter, the
bus continued and upon reaching Sabangan, M/ Sgt. Yag-as and S/Sgt. Ayan before they alighted, it
being their station, called C2C [Fernando] Bongyao to inspect the carton under seat No. 18. After C2C
Bongyao inspected the carton, he found out that it contained marijuana and he asked the passengers
[who] the owner of the carton [was] but nobody answered. Thereafter, C2C Bongyao alighted with the
carton and S/Sgt. Ayan and C2C Bongyao invited the herein accused to the detachment for questioning
as
accused
was
the
suspected
owner
of
the
carton
containing
marijuana

Held: As to the legality of appellants arrest, we find that the police operatives acted within the bounds
of law.
In the case at bench, appellant was caught red-handed in delivering two tin foils of marijuana to Pat.
Orolfo, Jr., the poseur-buyer. Applying the aforementioned provision of law, appellants arrest was
lawfully effected without need of a warrant of arrest. Having caught the appellant in flagrante as a
result of the buy-bust operation, the policemen were not only authorized but were also under obligation
to apprehend the drug pusher even without a warrant of arrest.
Appellant, however, asseverates that his arrest was precipitated only by newspaper publications about
the rampant sale of drugs along Garrido and Zamora Streets, Sta. Ana, Manila . A surveillance on the
illegal activities of the appellant was already conducted by the police as early as December 15 and 17,
1986. The newspaper reports concerning the illegal drug activities came out only on January 8 and 14,
1987, long after the police knew of the said illegal activities. Appellants eventual arrest on January 9,
1987 was the result of the surveillance conducted and the buy-bust operation.
The policemens entry into the house of appellant without a search warrant was in hot-pursuit of a
person caught committing an offense in flagrante. The arrest that followed the hot-pursuit was valid.
We also find as valid the seizure of the plastic bag of prohibited drugs found inside appellants house.
The seizure of the plastic bag containing prohibited drugs was the result of appellants arrest inside his
house. A contemporaneous search may be conducted upon the person of the arrestee and the
immediate vicinity where the arrest was made.

Issue: Whether or not the constitutional right of the accused against unreasonable searches and
seizures had been violated by the police authorities?
Held:In the case at bar, we have been unable to find in the record of this case any circumstance which
constituted or could have reasonably constituted probable cause for the peace officers to search the
carton box allegedly owned by appellant Barros. The carrying of such a box by appellant onto a
passenger bus could not, by itself, have convinced M/Sgt. Francis Yag-as and S/ Sgt. James Ayan
either that the appellant was a law violator or the contents of the box were instruments or the subject
matter or proceeds of some criminal offense. The carrying of carton boxes is a common practice among
our people, especially those coming from the rural areas since such boxes constitute the most
economical kind of luggage possible. The peace officers here involved had not received any information
or tip-off from an informer; nor such a tip-off was alleged by the police officers before or during the
trial. The police officers also did not contend that they had detected the odor of dried marijuana, or
appellant Barros had acted suspiciously in the course of boarding the bus and taking a seat during the
trip to Sabangan, nor in the course of being asked whether he owned the carton box later ascertained
to contain four (4) kilos of marijuana. The testimony of the law enforcement officers who had
apprehended the accused (M/ Sgt. Francis Yag-as and S/Sgt. James Ayan), and who had searched the
box in his possession, (C2C Fernando Bongyao), simply did not suggest or indicate the presence of
any
such
probable
cause.
So far as the record itself is concerned, therefore, it would appear that there existed no circumstance
which might reasonably have excited the suspicion of the two (2) police officers riding in the same bus
as appellant Barros. They asked the police officers at the checkpoint at Sabangan to inspect the box
allegedly carried by appellant Barros apparently on a mere guess that appellant Barros might be
carrying something in the nature of contraband goods. There was, in other words, nothing to show that
appellant Barros was then in the process of actually committing or attempting to commit a crime.21
There was, moreover, nothing on the record that could have reasonably led the two (2) police officers to
believe that an offense [had] in fact just been committed when appellant Barros boarded the bus at
Chakchakan or when he was asked whether he owned the box here involved at the checkpoint in
Sabangan. The two (2) police officers, according to the record, had no personable knowledge of facts
indicating that the person to be arrested (appellant Barros) had committed it. There was, in brief, no
basis for a valid warrantless arrest. Accordingly, the search and seizure of the carton box was equally
non-permissible and invalid.22 The fruits of the invalid search and seizurei.e., the four (4) kilos of
marijuanashould therefore not have been admitted in evidence against appellant Barros.

Aniag v. COMELEC 237 SCRA 424 Ebno Maruhom


Facts:Due to the Gun Ban, herein petitioner who was then Congressman of the 1st District of Bulacan
immediately instructed his driver, Ernesto Arellano, to pick up the firearms from petitioners house at
Valle Verde and return them to Congress.
About thirty minutes later, the policemen manning the outpost flagged down the car driven by Arellano
as it approached the checkpoint. They searched the car and found the firearms neatly packed in their
gun cases and placed in a bag in the trunk of the car.
On 6 April 1992, upon recommendation of its Law Department, COMELEC issued Resolution No. 920829 directing the filing of information against petitioner and Arellano for violation of Sec. 261, par. (q),
of B.P. Blg. 881 otherwise known as the Omnibus Election Code, in relation to Sec. 32 of R.A. No.
7166.
Issue: Whether or not the appellant can be validly prosecuted for instructing his driver to return to the
Sergeant-at-Arms of the House of Representatives the two firearms issued to him on the basis of the
evidence gathered from the warrantless search of his car.
Held:There was no evidence to show that the policemen were impelled to do the search because of a
confidential report leading them to reasonably believe that certain motorists matching the description
furnished by their informant were engaged in gunrunning, transporting firearms or in organizing special
strike forces. Nor, as adverted to earlier, was there any indication from the package or behavior of
Arellano that could have triggered the suspicion of the policemen. Absent such justifying circumstances
specifically pointing to the culpability of petitioner and Arellano, the search could not be valid. The
action then of the policemen unreasonably intruded into petitioners privacy and the security of his
property, in violation of Sec. 2, Art. III, of the Constitution. Consequently, the firearms obtained in
violation of petitioners right against warrantless search cannot be admitted for any purpose in any
proceeding.

52

People v. Tabar 222 SCRA 144 Jennifer oliveros

FACTS: On February 8, 1989 at around 3:00 in the afternoon, Police Officer Josephus Trangia and his
team held a buy bust operation for marijuana pusher in Punta Prinsesa, Cebu City. Their informant
approached a young boy named Rommel Tabar which he handed him the marked money P5.00 bill and
in returned the boy handed him the cigarette. Thereafter, the informant signal by scratching his head
with his right hand and they approached the boy and the informant introduced them as police officers,
they immediately proceeded to the scene. The informant gave the three (3) sticks of marijuana after
buying the same from the boy and that they confiscate the marked money from the boy. After such, they
proceeded to the shanty and met Carmelina Tabar and saw her holding white pants which they found
other marijuana sticks in cigarette packs which they also confiscated and brought her to Fuente Police
Station for Investigation. Thereafter, case filed against the person of Rommel Tabar and Carmelina
Tabar for violation of Sec. 4, Art. 11 of RA 6425 The Dangerous Act of 1972.

HELD: The court dismissed the case, the complainant himself admitted that the search and seizure was
conducted in the absence of warrant and it was produced by him and was issued it after the search and
seizure took place. That complainant cannot justify warrantless search and seizure by invoking circular
no. 130 of the Office of the President. The circular pertains to the procedure in the confiscation of fish
caught through the use of explosives. Such confiscation may be exercised only by the Commissioner of
Fisheries or his duly representatives, who can take only a sample of the fish not to exceed 1 kilo for
testing if the fish were indeed caught through the use of explosives. It is only upon the determination
that the fish were caught through the use of explosives when the seizure of the entire catch may be
authorized. Thereafter, an appraisal of the value of the fish caught shall be made, which shall be paid to
the accused should he be subsequently acquitted in the criminal case filed against him.

ISSUE:
WHETHER OR NOT THE POLICE OFFICER COULD LAWFULLY ARREST THE
ACCUSED WITHOUT CORRESPONDING ARREST AND SEARCH WARRANT.

HELD:
The court held upon Carmelina voluntarily unrolled the pair of pants with package
containing the packs of marijuana sticks, exposing the same to the Police team could be lawfully
arrested and searched without warrant. Pursuant to Section 5, Rule 113 and Sec. 12, Rule 126 of the
Revised Rules of Court Carmelina Tabar could be lawfully be arrested and searched for anything which
may be used as proof of commission of an offense without the corresponding arrest and search
warrants.

People v. Leangsiri 252 SCRA 213- Jennifer Oliveros

FACTS:Leangsiri was arrested in the Ninoy Aquino International Airport by bringing into the country approximately 8,225.31
grams of heroin. He informed the arresting officers that the heroin is meant to deliver to three other people in Las Palmas Hotel in
Manila. Thereafter, the NARCOM formed a group for a follow up operation in the said Hotel. With the Leangsiri cooperation, he
checked in to Room 504, where the others will meet him to give the drugs. At around 10 in the evening, Amidu and two other
co-appellants entered Room 504 and Leagsiri gave them the drugs, before the appellants leave the room, the NARCOM officers
barged in and arrested the appellants. Amidu, told the officers that he is staying in Rm 413 and that the two others are in Royal
Palm Hotel. The officers then went to the room of Amidu, searching for evidence and subsequently confiscated a telephone
address bearing the name of Leangsiri, other possessions and documents of Amidu were also confiscated. In the case of the two
other, the police confiscated a suit case and masking tape and empty transparent bag, allegedly will be use in transporting the
drugs. The appellants were charged and were convicted in conspiring to transport heroin violation of RA 6425.

Manlavi v. Gacott 244 SCRA 50 Jennifer oliveros

ISSUE:
FACTS:
That on January 18, 1991, the accused together with John Doe, Peter
Doe, and William Doe which identities and whereabouts are still unknown and Virgilio Laguna, a military
officer were conspiring and confederating and willfully, unlawfully and feloniously possess illegal fishing
with the use of explosives weighing 8,000 kilos. The accused moved to quash the case on the ground
of warrantless and illegal search and seizure. But the complainants contention that it is legal under
Circular No.130.

WHETHER OR NOT THE WARRANTLESS SEARCH IS LEGAL AND THE EVIDENCE FOUND CAN BE
ADMISSIBLE AS EVIDENCE.

HELD:
The court held warrantless search is illegal and the piece of paper bearing Leangsiri's name cannot be admitted as
evidence against appellants what can be admitted are evidences seized within the direct premise where the accused has an
immediate control which should only be Rm. 504. In the case at bar, appellants were arrested in Room 504 of the Las Palmas
Hotel. The piece of paper bearing Leangsiri's name was obtained through a warrantless search of Room 413 of the same hotel,
and found tucked within the pages of appellant Amidu's telephone and address book. Clearly, The inadmissibility of this evidence
will not, however, exculpate appellants. Its exclusion does not destroy the prosecution's case against appellants. The remaining
evidence still established their guilt beyond reasonable doubt.

ISSUE: WHETHER OR NOT THE ABSENCE OF A SEARCH WARRANT IS ALLOWED UNDER


CIRCULAR NO. 130.

53

You might also like